Как найти эдс индукции в контуре: ЭДС индукции в контуре определяется…

Содержание

ЭДС индукции, наводимая в контуре

Здесь рассмотрены задачи, от простых до сложных, на расчет возникающей в контуре ЭДС индукции при изменении потока. Потребуется знание производной, в том числе производной сложной функции.


Задача 1. За время мс  в соленоиде, содержащем витков, магнитный поток равномерно убывает от мВб до значения мВб. Найти величину ЭДС индукции в соленоиде.

   

Ответ: 400 В.

Задача 2. Соленоид, состоящий из витков и имеющий диаметр см, находится в однородном магнитном поле, индукция которого Тл. Соленоид поворачивают на угол в течение с. Найти среднее значение ЭДС индукции соленоида, если его ось до и после поворота параллельна линиям магнитной индукции.

Изменение потока вызывает появление ЭДС индукции. При повороте на поток меняется на .

   

Ответ: 0,24 В.

Задача 3. Контур площадью м расположен перпендикулярно к линиям магнитной индукции. Магнитная индукция однородного магнитного поля изменяется по закону . Определить зависимость магнитного потока и ЭДС индукции от времени. Определить мгновенное значение потока и ЭДС индукции в конце пятой секунды.

   

   

   

   

Ответ: , , Вб, В.

 

Задача 4. Кольцевой виток находится в переменном магнитном поле, индукция которого изменяется по закону и перпендикулярна плоскости витка. Виток, не перекрещивая, превратили в восьмерку, составленную из двух разных колец. Во сколько раз при этом изменилась амплитуда тока в витке? Индуктивностью витка пренебречь.

Поток в данном случае изменяется в связи с изменением площади. Посмотрим, как изменилась площадь. Первоначально площадь витка равна , длина витка . После изменения формы, так как колец два, то длина каждого из них . То есть . Тогда площадь такого витка

   

Так как витка два, то их площадь суммарно равна , следовательно, первоначальная площадь изменилась вдвое, и поток тогда тоже изменился вдвое. Следовательно, вдвое меньше станет ЭДС индукции и вдвое меньше станет ток.

Ответ: в два раза меньше.

Задача 5. Квадратную рамку из проводника вращают равномерно в перпендикулярном оси рамки переменном магнитном поле, изменяющемся по закону . Сторона рамки см. В начальный момент времени угол между плоскостью рамки и направлением индукции магнитного поля , угловая скорость вращения рамки рад/с.  Найти зависимость ЭДС индукции, которая возникает в рамке, от времени.

Сначала рамка ориентирована перпендикулярно полю, и поток максимален. Следовательно, изменение площади начнется с максимального значения – а это значит, по закону косинуса. Тогда площадь рамки

   

Поток через рамку будет равен

   

Определяем ЭДС:

   

Ответ: .

 

Эдс индукции в замкнутом контуре это

Электромагнитная индукция (индукция значит наведение) это явление, при котором в замкнутом контуре возникает электрический ток при изменении магнитного потока, пронизывающего его.

Явление

электромагнитной индукции было обнаружено в 1831 г.

М. Фарадеем. Ток, возникающий при электромагнитной индукции называют индукционным.

Закон электромагнитной индукцииЭДС индукции в контуре равна скорости изменения магнитного поля сквозь поверхность, ограниченную контуром.

Электромагнитная индукция
1831 г. – М. Фарадей обнаружил, что в замкнутом проводящем контуре при изменении магнитного поля возникает так называемый индукционный ток. (Индукция, в данном случае, – появление, возникновение).
Индукционный ток в катушке возникает при
перемещении постоянного магнита относительно катушки; при перемещении электромагнита относительно катушки; при перемещении сердечника относительно электромагнита, вставленного в катушку; при регулировании тока в цепи электромагнита; при замыкании и размыкании цепи
Появление тока в замкнутом контуре при изменении магнит­ного поля, пронизывающего контур, свидетельствует о действии в контуре сторонних сил (или о возникно­вении ЭДС индукции). Явление возникновения ЭДС в замкнутом проводящем контуре при изменении магнитного поля (потока), пронизывающего контур, назы­вается электромагнитной индукцией. Или:явление возникновения электрического поля при изменении магнитного поля (потока), называется электромагнитной индукцией.
Закон электромагнитной индукции При всяком изменении магнитного потока через проводящий замкнутый контур в этом контуре возникает электрический ток. I зависит от свойств контура (сопротивление): . e не зависит от свойств контура: . ЭДС индукции в замкнутом контуре прямо пропорциональна скорости изменения магнитного потока через площадь, ограниченную этим контуром.
Основные применения электромагнитной индукции: генерирование тока (индукционные генераторы на всех электростанциях, динамомашины), трансформаторы.
Правило Ленца Возникновение индукционного тока – следствие закона сохранения энергии! В случае 1: При приближении магнита, увеличении тока, замыкании цепи: ; Магнитный поток Ф­ → ΔФ>0.Чтобы компенсировать это изменение (увеличение) внешнего поля, необходимо магнитное поле, направленное в сторону, противоположную внешнему полю: , где – т.н. индукционное магнитное поле. В случае 2: при удалении магнита, уменьшении тока, размыкании цепи: . Магнитный поток Ф → ΔФ 0). Ток в контуре имеет положительное направление ( ), если совпа­дает с , (т.е. ΔΦ

Не нашли то, что искали? Воспользуйтесь поиском:

Явление электромагнитной индукции

Электромагнитная индукция – явление возникновения тока в замкнутом проводящем контуре при изменении магнитного потока, пронизывающего его.

Явление электромагнитной индукции было открыто М. Фарадеем.

  • На одну непроводящую основу были намотаны две катушки: витки первой катушки были расположены между витками второй. Витки одной катушки были замкнуты на гальванометр, а второй – подключены к источнику тока. При замыкании ключа и протекании тока по второй катушке в первой возникал импульс тока. При размыкании ключа также наблюдался импульс тока, но ток через гальванометр тек в противоположном направлении.
  • Первая катушка была подключена к источнику тока, вторая, подключенная к гальванометру, перемещалась относительно нее. При приближении или удалении катушки фиксировался ток.
  • Катушка замкнута на гальванометр, а магнит движется – вдвигается (выдвигается) – относительно катушки.

Опыты показали, что индукционный ток возникает только при изменении линий магнитной индукции. Направление тока будет различно при увеличении числа линий и при их уменьшении.

Сила индукционного тока зависит от скорости изменения магнитного потока. Может изменяться само поле, или контур может перемещаться в неоднородном магнитном поле.

Объяснения возникновения индукционного тока

Ток в цепи может существовать, когда на свободные заряды действуют сторонние силы. Работа этих сил по перемещению единичного положительного заряда вдоль замкнутого контура равна ЭДС. Значит, при изменении числа магнитных линий через поверхность, ограниченную контуром, в нем появляется ЭДС, которую называют ЭДС индукции.

Электроны в неподвижном проводнике могут приводиться в движение только электрическим полем. Это электрическое поле порождается изменяющимся во времени магнитным полем. Его называют вихревым электрическим полем. Представление о вихревом электрическом поле было введено в физику великим английским физиком Дж. Максвеллом в 1861 году.

Свойства вихревого электрического поля:

  • источник – переменное магнитное поле;
  • обнаруживается по действию на заряд;
  • не является потенциальным;
  • линии поля замкнутые.

Работа этого поля при перемещении единичного положительного заряда по замкнутому контуру равна ЭДС индукции в неподвижном проводнике.

Магнитный поток

Магнитным потоком через площадь ​ ( S ) ​ контура называют скалярную физическую величину, равную произведению модуля вектора магнитной индукции ​ ( B ) ​, площади поверхности ​ ( S ) ​, пронизываемой данным потоком, и косинуса угла ​ ( alpha ) ​ между направлением вектора магнитной индукции и вектора нормали (перпендикуляра к плоскости данной поверхности):

Обозначение – ​ ( Phi ) ​, единица измерения в СИ – вебер (Вб).

Магнитный поток в 1 вебер создается однородным магнитным полем с индукцией 1 Тл через поверхность площадью 1 м 2 , расположенную перпендикулярно вектору магнитной индукции:

Магнитный поток можно наглядно представить как величину, пропорциональную числу магнитных линий, проходящих через данную площадь.

В зависимости от угла ​ ( alpha ) ​ магнитный поток может быть положительным ( ( alpha ) ( alpha ) > 90°). Если ( alpha ) = 90°, то магнитный поток равен 0.

Изменить магнитный поток можно меняя площадь контура, модуль индукции поля или расположение контура в магнитном поле (поворачивая его).

В случае неоднородного магнитного поля и неплоского контура магнитный поток находят как сумму магнитных потоков, пронизывающих площадь каждого из участков, на которые можно разбить данную поверхность.

Закон электромагнитной индукции Фарадея

Закон электромагнитной индукции (закон Фарадея):

ЭДС индукции в замкнутом контуре равна и противоположна по знаку скорости изменения магнитного потока через поверхность, ограниченную контуром:

Знак «–» в формуле позволяет учесть направление индукционного тока. Индукционный ток в замкнутом контуре имеет всегда такое направление, чтобы магнитный поток поля, созданного этим током сквозь поверхность, ограниченную контуром, уменьшал бы те изменения поля, которые вызвали появление индукционного тока.

Если контур состоит из ​ ( N ) ​ витков, то ЭДС индукции:

Сила индукционного тока в замкнутом проводящем контуре с сопротивлением ​ ( R ) ​:

При движении проводника длиной ​ ( l ) ​ со скоростью ​ ( v ) ​ в постоянном однородном магнитном поле с индукцией ​ ( vec ) ​ ЭДС электромагнитной индукции равна:

где ​ ( alpha ) ​ – угол между векторами ​ ( vec ) ​ и ( vec ) .

Возникновение ЭДС индукции в движущемся в магнитном поле проводнике объясняется действием силы Лоренца на свободные заряды в движущихся проводниках. Сила Лоренца играет в этом случае роль сторонней силы.

Движущийся в магнитном поле проводник, по которому протекает индукционный ток, испытывает магнитное торможение. Полная работа силы Лоренца равна нулю.

Количество теплоты в контуре выделяется либо за счет работы внешней силы, которая поддерживает скорость проводника неизменной, либо за счет уменьшения кинетической энергии проводника.

Важно!
Изменение магнитного потока, пронизывающего замкнутый контур, может происходить по двум причинам:

  • магнитный поток изменяется вследствие перемещения контура или его частей в постоянном во времени магнитном поле. Это случай, когда проводники, а вместе с ними и свободные носители заряда, движутся в магнитном поле;
  • вторая причина изменения магнитного потока, пронизывающего контур, – изменение во времени магнитного поля при неподвижном контуре. В этом случае возникновение ЭДС индукции уже нельзя объяснить действием силы Лоренца. Явление электромагнитной индукции в неподвижных проводниках, возникающее при изменении окружающего магнитного поля, также описывается формулой Фарадея.

Таким образом, явления индукции в движущихся и неподвижных проводниках протекают одинаково, но физическая причина возникновения индукционного тока оказывается в этих двух случаях различной:

  • в случае движущихся проводников ЭДС индукции обусловлена силой Лоренца;
  • в случае неподвижных проводников ЭДС индукции является следствием действия на свободные заряды вихревого электрического поля, возникающего при изменении магнитного поля.

Правило Ленца

Направление индукционного тока определяется по правилу Ленца: индукционный ток, возбуждаемый в замкнутом контуре при изменении магнитного потока, всегда направлен так, что создаваемое им магнитное поле препятствует изменению магнитного потока, вызывающего индукционный ток.

Алгоритм решения задач с использованием правила Ленца:

  • определить направление линий магнитной индукции внешнего магнитного поля;
  • выяснить, как изменяется магнитный поток;
  • определить направление линий магнитной индукции магнитного поля индукционного тока: если магнитный поток уменьшается, то они сонаправлены с линиями внешнего магнитного поля; если магнитный поток увеличивается, – противоположно направлению линий магнитной индукции внешнего поля;
  • по правилу буравчика, зная направление линий индукции магнитного поля индукционного тока, определить направление индукционного тока.

Правило Ленца имеет глубокий физический смысл – оно выражает закон сохранения энергии.

Самоиндукция

Самоиндукция – это явление возникновения ЭДС индукции в проводнике в результате изменения тока в нем.

При изменении силы тока в катушке происходит изменение магнитного потока, создаваемого этим током. Изменение магнитного потока, пронизывающего катушку, должно вызывать появление ЭДС индукции в катушке.

В соответствии с правилом Ленца ЭДС самоиндукции препятствует нарастанию силы тока при включении и убыванию силы тока при выключении цепи.

Это приводит к тому, что при замыкании цепи, в которой есть источник тока с постоянной ЭДС, сила тока устанавливается через некоторое время.

При отключении источника ток также не прекращается мгновенно. Возникающая при этом ЭДС самоиндукции может превышать ЭДС источника.

Явление самоиндукции можно наблюдать, собрав электрическую цепь из катушки с большой индуктивностью, резистора, двух одинаковых ламп накаливания и источника тока. Резистор должен иметь такое же электрическое сопротивление, как и провод катушки.

Опыт показывает, что при замыкании цепи электрическая лампа, включенная последовательно с катушкой, загорается несколько позже, чем лампа, включенная последовательно с резистором. Нарастанию тока в цепи катушки при замыкании препятствует ЭДС самоиндукции, возникающая при возрастании магнитного потока в катушке.

При отключении источника тока вспыхивают обе лампы. В этом случае ток в цепи поддерживается ЭДС самоиндукции, возникающей при убывании магнитного потока в катушке.

ЭДС самоиндукции ​ ( varepsilon_ ) ​, возникающая в катушке с индуктивностью ​ ( L ) ​, по закону электромагнитной индукции равна:

ЭДС самоиндукции прямо пропорциональна индуктивности катушки и скорости изменения силы тока в катушке.

Индуктивность

Электрический ток, проходящий по проводнику, создает вокруг него магнитное поле. Магнитный поток ​ ( Phi ) ​ через контур из этого проводника пропорционален модулю индукции ​ ( vec ) ​ магнитного поля внутри контура, а индукция магнитного поля, в свою очередь, пропорциональна силе тока в проводнике.

Следовательно, магнитный поток через контур прямо пропорционален силе тока в контуре:

Индуктивность – коэффициент пропорциональности ​ ( L ) ​ между силой тока ​ ( I ) ​ в контуре и магнитным потоком ​ ( Phi ) ​, создаваемым этим током:

Индуктивность зависит от размеров и формы проводника, от магнитных свойств среды, в которой находится проводник.

Единица индуктивности в СИ – генри (Гн). Индуктивность контура равна 1 генри, если при силе постоянного тока 1 ампер магнитный поток через контур равен 1 вебер:

Можно дать второе определение единицы индуктивности: элемент электрической цепи обладает индуктивностью в 1 Гн, если при равномерном изменении силы тока в цепи на 1 ампер за 1 с в нем возникает ЭДС самоиндукции 1 вольт.

Энергия магнитного поля

При отключении катушки индуктивности от источника тока лампа накаливания, включенная параллельно катушке, дает кратковременную вспышку. Ток в цепи возникает под действием ЭДС самоиндукции.

Источником энергии, выделяющейся при этом в электрической цепи, является магнитное поле катушки.

Для создания тока в контуре с индуктивностью необходимо совершить работу на преодоление ЭДС самоиндукции. Энергия магнитного поля тока вычисляется по формуле:

Основные формулы раздела «Электромагнитная индукция»

Алгоритм решения задач по теме «Электромагнитная индукция»:

1. Внимательно прочитать условие задачи. Установить причины изменения магнитного потока, пронизывающего контур.

2. Записать формулу:

  • закона электромагнитной индукции;
  • ЭДС индукции в движущемся проводнике, если в задаче рассматривается поступательно движущийся проводник; если в задаче рассматривается электрическая цепь, содержащая источник тока, и возникающая на одном из участков ЭДС индукции, вызванная движением проводника в магнитном поле, то сначала нужно определить величину и направление ЭДС индукции. После этого задача решается по аналогии с задачами на расчет цепи постоянного тока с несколькими источниками.

3. Записать выражение для изменения магнитного потока и подставить в формулу закона электромагнитной индукции.

4. Записать математически все дополнительные условия (чаще всего это формулы закона Ома для полной цепи, силы Ампера или силы Лоренца, формулы кинематики и динамики).

5. Решить полученную систему уравнений относительно искомой величины.

Причиной электродвижущей силы может стать изменение магнитного поля в окружающем пространстве. Это явление называетсяэлектромагнитной индукцией. Величина ЭДС индукции в контуре определяется выражением

где — поток магнитного поля через замкнутую поверхность , ограниченную контуром. Знак «−» перед выражением показывает, что индукционный ток, созданный ЭДС индукции, препятствует изменению магнитного потока в контуре (см. правило Ленца).

41. Индуктивность, ее единица СИ. Индуктивность длинного соленоида.

Индукти́вность (или коэффициент самоиндукции) — коэффициент пропорциональности между электрическим током, текущим в каком-либо замкнутом контуре, и магнитным потоком, создаваемым этим током через поверхность [1] , краем которой является этот контур. [2][3][4] .

— магнитный поток, — ток в контуре, — индуктивность.

Нередко говорят об индуктивности прямого длинного провода(см.). В этом случае и других (особенно – в не отвечающих квазистационарному приближению) случаях, когда замкнутый контур непросто адекватно и однозначно указать, приведенное выше определение требует особых уточнений; отчасти полезным для этого оказывается подход (упоминаемый ниже), связывающий индуктивность с энергией магнитного поля.

Через индуктивность выражается ЭДС самоиндукции в контуре, возникающая при изменении в нём тока [4] :

.

Из этой формулы следует, что индуктивность численно равна ЭДС самоиндукции, возникающей в контуре при изменении силы тока на 1 А за 1 с.

При заданной силе тока индуктивность определяет энергию магнитного поля, создаваемого этим током [4] :

.

Обозначение и единицы измерения

В системе единиц СИ индуктивность измеряется в генри [7] , сокращенно Гн, в системе СГС — в сантиметрах (1 Гн = 10 9 см) [4] . Контур обладает индуктивностью в один генри, если при изменении тока на один ампер в секунду на выводах контура будет возникать напряжение в один вольт. Реальный, не сверхпроводящий, контур обладает омическим сопротивлением R, поэтому на нём будет дополнительно возникать напряжение U=I*R, где I — сила тока, протекающего по контуру в данное мгновение времени.

Символ , используемый для обозначения индуктивности, был взят в честь Ленца Эмилия Христиановича (Heinrich Friedrich Emil Lenz) [ источник не указан 1017 дней ] . Единица измерения индуктивности названа в честь Джозефа Генри (Joseph Henry) [8] . Сам термин индуктивность был предложен Оливером Хевисайдом (Oliver Heaviside) в феврале 1886 года [ источник не указан 1017 дней ] .

Электрический ток, который течет в замкнутом контуре, создает вокруг себя магнитное поле, индукция которого, согласно закону Био-Савара-Лапласа, пропорциональна току. Сцепленный с контуром магнитный поток Ф поэтому прямо пропорционален току I в контуре: (1) где коэффициент пропорциональности L называетсяиндуктивностью контура. При изменении в контуре силы тока будет также изменяться и сцепленный с ним магнитный поток; значит, в контуре будет индуцироваться э.д.с. Возникновение э.д.с. индукции в проводящем контуре при изменении в нем силы тока называетсясамоиндукцией. Из выражения (1) задается единица индуктивности генри (Гн): 1 Гн — индуктивность контура, магнитный поток самоиндукции которого при токе в 1 А равен 1 Вб: 1 Гн = 1 Вб/с = 1 В

Вычислим индуктивность бесконечно длинного соленоида. Полный магнитный поток сквозь соленоид (потокосцепление) равен μμ(N 2 I/l)S . Подставив в (1), найдем (2) т. е. индуктивность соленоида зависит от длиныl солениода, числа его витков N, его , площади S и магнитной проницаемости μ вещества, из которого изготовлен сердечник соленоида. Доказано, что индуктивность контура зависит в общем случае только от геометрической формы контура, его размеров и магнитной проницаемости среды, в которой он расположен, и можно провести аналог индуктивности контура с электрической емкостью уединенного проводника, которая также зависит только от формы проводника, его размеров и диэлектрической проницаемости среды. Найдем, применяя к явлению самоиндукции закон Фарадея, что э.д.с. самоиндукции равна Если контур не претерпевает деформаций и магнитная проницаемость среды остается неизменной (в дальнейшем будет показано, что последнее условие выполняется не всегда), то L = const и(3) где знак минус, определяемый правилом Ленца, говорит о том, чтоналичие индуктивности в контуре приводит к замедлению изменения тока в нем. Если ток со временем увеличивается, то (dI/dt 0 т. е. ток самоиндукции направлен навстречу току, обусловленному внешним источником, и замедляет его увеличение. Если ток со временем уменьшается, то (dI/dt>0) и ξs >1), обладающей боль­шой индуктивностью, э.д.с. самоиндукции может во много раз превышать э.д.с. источника тока, включенного в цепь. Таким образом, необходимо учитывать, что контур, содержащий индуктивность, нельзя резко размыкать, так как это (возникнове­ние значительных э.д.с. самоиндукции) может привести к пробою изоляции и выводу из строя измерительных приборов. Если в контур сопротивление вводить постепенно, то э.д.с. самоиндукции не достигнет больших значений.

43. Явление взаимной индукции. Трансформатор.

Рассмотрим два неподвижных контура (1 и 2), которые расположены достаточно близко друг от друга (рис. 1). Если в контуре 1 протекает ток I1, то магнитный поток, который создавается этим током (поле, создающее этот поток, на рисунке изображено сплошными линиями), прямо пропорционален I1. Обозначим через Ф21 часть потока,пронизывающая контур 2. Тогда (1) где L21 — коэффициент пропорциональности.

Если ток I1 меняет свое значение, то в контуре 2 индуцируется э.д.с. ξi2 , которая по закону Фарадея будет равна и противоположна по знаку скорости изменения магнитного потока Ф21, который создается током в первом контуре и пронизыващет второй: Аналогичным образом, при протекании в контуре 2 тока I2 магнитный поток (его поле изображено на рис. 1 штрихами) пронизывает первый контур. Если Ф12 — часть этого потока, который пронизывает контур 1, то Если ток I2 меняет свое значение, то в контуре 1 индуцируется э.д.с. ξi1 , которая равна и противоположна по знаку скорости изменения магнитного потока Ф12, который создается током во втором контуре и пронизывает первый: Явление возникновения э.д.с. в одном из контуров при изменении силы тока в другом называется взаимной индукцией. Коэффициенты пропорциональности L21 и L12 называются взаимной индуктивностью контуров. Расчеты, которые подтверждены опытом, показывают, что L21 и L12 равны друг другу, т. е. (2) Коэффициенты пропорциональности L12 и L21 зависят от размеров, геометрической формы, взаимного расположения контуров и от магнитной проницаемости среды, окружающей контуры. Единица взаимной индуктивности та же, что и для индуктивности, — генри (Гн). Найдем взаимную индуктивность двух катушек, которые намотаны на общий тороидальный сердечник. Этот случай имеет большое практическое значение (рис. 2). Магнитная индукция поля, которое создавается первой катушкой с числом витков N1, током I1 и магнитной проницаемостью μ сердечника, B = μμ(N1I1/l) где l — длина сердечника по средней линии. Магнитный поток сквозь один виток второй катушки Ф2 = BS = μμ(N1I1/l)S

Значит, полный магнитный поток (потокосцепление) сквозь вторичную обмотку, которая содержит N2 витков, Поток Ψ создается током I1, поэтому, используя (1), найдем (3) Если рассчитать магнитный поток, который создавается катушкой 2 сквозь катушку 1, то для L12 получим выражение в соответствии с формулой (3). Значит, взаимная индуктивность двух катушек, которые намотаны на общий тороидальный сердечник,

Трансформа́тор (от лат. transformo — преобразовывать) — это статическое электромагнитное устройство, имеющее две или более индуктивно связанных обмоток на каком-либо магнитопроводе и предназначенное для преобразования посредствомэлектромагнитной индукции одной или нескольких систем (напряжений) переменного тока в одну или несколько других систем (напряжений) переменного тока без изменения частоты системы (напряжения) переменного тока

Электродвижущая сила индукции

Электрические токи порождают вокруг себя магнитные поля. Данная связь дала толчок к многочисленным попыткам создать электрический ток в контуре при помощи магнитного поля.

Данную задачу решил М. Фарадей в 1831 году. Ученый открыл явление электромагнитной индукции.

Электромагнитная индукция

Явление электромагнитной индукции заключается в том, что в замкнутом проводящем контуре, если изменяется поток магнитной индукции, который рассматриваемый контур охватывает, появляется электрический ток. Возникающий электрический ток называют током индукции.

Анализируя свои множественные эксперименты, М. Фарадей пришел к выводу о том, что:

  1. Индукционный ток появляется всегда при изменении магнитного потока, который охватывает проводящий контур. Так, если в однородном магнитном поле проводящий замкнутый контур повернуть, то в момент разворота в нем будет течь ток индукции. В этом случае индукция магнитного поля постоянна около проводящего контура, переменным является только поток магнитной индукции, который изменяется за счет изменения площади контура.
  2. Величина тока индукции не связана со способом изменения магнитного потока. Она определена только скоростью его изменения. Сила тока индукции тем больше, чем больше скорость перемещения магнита, или быстрота изменения силы тока, или скорость перемещения катушек.

Электромагнитная индукция подтверждает связь между электрическими и магнитными явлениями.

Закон Фарадея

Анализируя данные своих экспериментов, М. Фарадей предложил количественный закон, описывающий электромагнитную индукцию. Ученый доказал, что каждый раз при изменении потока магнитной индукции, который сцеплен с проводящим контуром, в проводнике появляется ток индукции. Наличие индукционного тока означает то, что в цепи присутствует электродвижущая сила (ЭДС), которую в данном случае называют электродвижущей силой электромагнитной индукции ($Ɛ_i$).

Величина тока индукции, а значит, и величина $Ɛ_i$ зависит от скорости изменения магнитного потока:

$\left|Ɛ_{i} \right|=\frac{dФ}{dt}\left( 1 \right)$.

где $Ф$ — поток магнитной индукции.

Определимся со знаком ЭДС индукции. Знак потока магнитной индукции связан с выбором положительной нормали к рассматриваемому проводящему контуру. А направление силы тока и направление нормали связывает правило правого буравчика (винта). Получается, что фиксируя направление нормали, мы устанавливаем знак магнитного потока, направление тока и $Ɛ_i$ в контуре.

Готовые работы на аналогичную тему

Сформулируем закон электромагнитной индукции Фарадея в окончательном виде:

Определение 1

Не зависимо от причины изменения магнитного потока, который охватывает замкнутый проводящий контур, электродвижущая сила индукции, появляющаяся в этом контуре равна:

$Ɛ_{i}=-\frac{dФ}{dt}\left( 2 \right)$.

где под $\frac{dФ}{dt}$ понимают полную скорость изменения потока магнитной индукции, охватываемого проводником.

Минус в формуле (2) указывает на то, что:

  • При росте потока магнитной индукции (скорость изменения магнитного потока больше нуля) ($\frac{dФ}{dt}>0)$, ЭДС индукции меньше нуля ($Ɛ_i
  • При уменьшении потока магнитной индукции (скорость изменения магнитного потока меньше нуля), ЭДС индукции больше нуля ($Ɛ_i>0$). Что значит, направление потока и направление поля тока индукции совпадают.

Замечание 1

Знак минус в формуле (2) – это математическое отображение правила Ленца, которое используют для того, чтобы найти направление тока индукции.

Закон Фарадея справедлив при:

  1. произвольных перемещениях замкнутого проводящего контура;
  2. при любых его деформациях;
  3. изменениях магнитного поля.

ЭДС индукции измеряется с Международной системе единиц (СИ) в вольтах (В).

$\left[ Ɛ_{i} \right]=\left[ \frac{dФ}{dt} \right]=\frac{Вб}{с}$=В.

Значение закона Фарадея

Закон Фарадея выражает новое физическое явление, в котором переменное магнитное поле порождает электрическое поле. Отсюда делается вывод о том, что электрическое поле может порождаться не только электрическими зарядами, но и изменяющимся магнитным полем.

Электромагнитная индукция – это всеобщий фундаментальный закон природы, реализующий связь между электрическими и магнитными полями.

Природа электродвижущей силы индукции

Если проводник перемещается в магнитном поле, то на свободные электроны его вещества действуют силы Лоренца. Эти электроны под воздействием названной силы приходят в движение относительно проводника, что означает: в проводнике появляется ток.

Рисунок 1. Проводники. Автор24 — интернет-биржа студенческих работ

Рассмотрим прямой участок $DG$ проводника на рис.1. Этот участок перемещается со скоростью $\vec v$ по проводникам $CK$ и $AL$, как по направляющим. При этом контур $AGDCA$ постоянно замкнут. Вектор индукции внешнего магнитного поля нормален плоскости рассматриваемого контура. Магнитное поле будем считать однородным. На заряды, которые перемещаются вместе с проводником, действует сила Лоренца, равная:

$\vec{F}_{L}=q\left( \vec{v}\times \vec{B} \right)\left( 3 \right)$.

где$ \vec{B}$– индукция внешнего магнитного поля. Под воздействием силы Лоренца свободные электроны проводника приходят в движение и образуют электрический ток.D {vBdl=vBl\, \left( 6 \right).} $

На не движущихся участках замкнутого контура, который мы рассматриваем, ЭДС не возникает. Следовательно, ЭДС контура равна ЭДС подвижного проводника $DG$, перемещающейся в магнитном поле.

$Ɛ_{i}=\int\limits_{AGDCA} {\vec{E}d\vec{l}=vBl\, \left( 7 \right).} $

Скорость перемещения проводника выразим как:

$v=\frac{dx}{dt}\left( 8 \right)$.

где $x$ — координата контактов проводника в точках $D$ и $G$ направляющими проводниками:

$Ɛ_{i}=Bl\frac{dx}{dt}\left( 9 \right)$.

Учитывая, что:

$Ф=-xlB$ (10),

где $Ф$ — поток магнитной индукции через поверхность, которую ограничивает контур $AGDCA$. Знак минус указывает на то, что направления векторов $\vec B$ и $ d\vec S$ противоположны,

окончательно имеем:

$Ɛ_{i}=-\frac{dФ}{dt}\left( 11 \right)$.

Выражение (11) мы получили, рассматривая движение части проводника. При перемещении нескольких участков проводника, ЭДС индукции находят как алгебраическую сумму ЭДС индукции, появляющихся на каждом участке.

Электромагнитная индукция — материалы для подготовки к ЕГЭ по Физике

Автор статьи — профессиональный репетитор, автор учебных пособий для подготовки к ЕГЭ Игорь Вячеславович Яковлев

Темы кодификатора ЕГЭ: явление электромагнитной индукции, магнитный поток, закон электромагнитной индукции Фарадея, правило Ленца.

Опыт Эрстеда показал, что электрический ток создаёт в окружающем пространстве магнитное поле. Майкл Фарадей пришёл к мысли, что может существовать и обратный эффект: магнитное поле, в свою очередь, порождает электрический ток.

Иными словами, пусть в магнитном поле находится замкнутый проводник; не будет ли в этом проводнике возникать электрический ток под действием магнитного поля?

Через десять лет поисков и экспериментов Фарадею наконец удалось этот эффект обнаружить. В 1831 году он поставил следующие опыты.

1. На одну и ту же деревянную основу были намотаны две катушки; витки второй катушки были проложены между витками первой и изолированы. Выводы первой катушки подключались к источнику тока, выводы второй катушки — к гальванометру (гальванометр — чувствительный прибор для измерения малых токов). Таким образом, получались два контура: «источник тока — первая катушка» и «вторая катушка — гальванометр».

Электрического контакта между контурами не было, только лишь магнитное поле первой катушки пронизывало вторую катушку.

При замыкании цепи первой катушки гальванометр регистрировал короткий и слабый импульс тока во второй катушке.

Когда по первой катушке протекал постоянный ток, никакого тока во второй катушке не возникало.

При размыкании цепи первой катушки снова возникал короткий и слабый импульс тока во второй катушке, но на сей раз в обратном направлении по сравнению с током при замыкании цепи.

Вывод.

Меняющееся во времени магнитное поле первой катушки порождает (или, как говорят, индуцирует) электрический ток во второй катушке. Этот ток называется индукционным током.

Если магнитное поле первой катушки увеличивается (в момент нарастания тока при замыкании цепи), то индукционный ток во второй катушке течёт в одном направлении.

Если магнитное поле первой катушки уменьшается (в момент убывания тока при размыкании цепи), то индукционный ток во второй катушке течёт в другом направлении.

Если магнитное поле первой катушки не меняется (постоянный ток через неё), то индукционного тока во второй катушке нет.

Обнаруженное явление Фарадей назвал электромагнитной индукцией (т. е. «наведение электричества магнетизмом»).

2. Для подтверждения догадки о том, что индукционный ток порождается переменным магнитным полем, Фарадей перемещал катушки друг относительно друга. Цепь первой катушки всё время оставалась замкнутой, по ней протекал постоянный ток, но за счёт перемещения (сближения или удаления) вторая катушка оказывалась в переменном магнитном поле первой катушки.

Гальванометр снова фиксировал ток во второй катушке. Индукционный ток имел одно направление при сближении катушек, и другое — при их удалении. При этом сила индукционного тока была тем больше, чем быстрее перемещались катушки.

3. Первая катушка была заменена постоянным магнитом. При внесении магнита внутрь второй катушки возникал индукционный ток. При выдвигании магнита снова появлялся ток, но в другом направлении. И опять-таки сила индукционного тока была тем больше, чем быстрее двигался магнит.

Эти и последующие опыты показали, что индукционный ток в проводящем контуре возникает во всех тех случаях, когда меняется «количество линий» магнитного поля, пронизывающих контур. Сила индукционного тока оказывается тем больше, чем быстрее меняется это количество линий. Направление тока будет одним при увеличении количества линий сквозь контур, и другим — при их уменьшении.

Замечательно, что для величины силы тока в данном контуре важна лишь скорость изменения количества линий. Что конкретно при этом происходит, роли не играет — меняется ли само поле, пронизывающее неподвижный контур, или же контур перемещается из области с одной густотой линий в область с другой густотой.

Такова суть закона электромагнитной индукции. Но, чтобы написать формулу и производить расчёты, нужно чётко формализовать расплывчатое понятие «количество линий поля сквозь контур».

Магнитный поток

Понятие магнитного потока как раз и является характеристикой количества линий магнитного поля, пронизывающих контур.

Для простоты мы ограничиваемся случаем однородного магнитного поля. Рассмотрим контур площади , находящийся в магнитном поле с индукцией .

Пусть сначала магнитное поле перпендикулярно плоскости контура (рис. 1).

Рис. 1.

В этом случае магнитный поток определяется очень просто — как произведение индукции магнитного поля на площадь контура:

(1)

Теперь рассмотрим общий случай, когда вектор образует угол с нормалью к плоскости контура (рис. 2).

Рис. 2.

Мы видим, что теперь сквозь контур «протекает» лишь перпендикулярная составляющая вектора магнитной индукции (а та составляющая, которая параллельна контуру, не «течёт» сквозь него). Поэтому, согласно формуле (1), имеем . Но , поэтому

(2)

Это и есть общее определение магнитного потока в случае однородного магнитного поля. Обратите внимание, что если вектор параллелен плоскости контура (то есть ), то магнитный поток становится равным нулю.

А как определить магнитный поток, если поле не является однородным? Укажем лишь идею. Поверхность контура разбивается на очень большое число очень маленьких площадок, в пределах которых поле можно считать однородным. Для каждой площадки вычисляем свой маленький магнитный поток по формуле (2), а затем все эти магнитные потоки суммируем.

Единицей измерения магнитного потока является вебер (Вб). Как видим,

Вб = Тл · м = В · с. (3)

Почему же магнитный поток характеризует «количество линий» магнитного поля, пронизывающих контур? Очень просто. «Количество линий» определяется их густотой (а значит, величиной — ведь чем больше индукция, тем гуще линии) и «эффективной» площадью, пронизываемой полем (а это есть не что иное, как ). Но множители и как раз и образуют магнитный поток!

Теперь мы можем дать более чёткое определение явления электромагнитной индукции, открытого Фарадеем.

Электромагнитная индукция — это явление возникновения электрического тока в замкнутом проводящем контуре при изменении магнитного потока, пронизывающего контур.

ЭДС индукции

Каков механизм возникновения индукционного тока? Это мы обсудим позже. Пока ясно одно: при изменении магнитного потока, проходящего через контур, на свободные заряды в контуре действуют некоторые силы — сторонние силы, вызывающие движение зарядов.

Как мы знаем, работа сторонних сил по перемещению единичного положительного заряда вокруг контура называется электродвижущей силой (ЭДС): . В нашем случае, когда меняется магнитный поток сквозь контур, соответствующая ЭДС называется ЭДС индукции и обозначается .

Итак, ЭДС индукции — это работа сторонних сил, возникающих при изменении магнитного потока через контур, по перемещению единичного положительного заряда вокруг контура.

Природу сторонних сил, возникающих в данном случае в контуре, мы скоро выясним.

Закон электромагнитной индукции Фарадея

Сила индукционного тока в опытах Фарадея оказывалась тем больше, чем быстрее менялся магнитный поток через контур.

Если за малое время изменение магнитного потока равно , то скорость изменения магнитного потока — это дробь (или, что тоже самое, производная магнитного потока по времени).

Опыты показали, что сила индукционного тока прямо пропорциональна модулю скорости изменения магнитного потока:

Модуль поставлен для того, чтобы не связываться пока с отрицательными величинами (ведь при убывании магнитного потока будет ). Впоследствии мы это модуль снимем.

Из закона Ома для полной цепи мы в то же время имеем: . Поэтому ЭДС индукции прямо пропорциональна скорости изменения магнитного потока:

(4)

ЭДС измеряется в вольтах. Но и скорость изменения магнитного потока также измеряется в вольтах! Действительно, из (3) мы видим, что Вб/с = В. Стало быть, единицы измерения обеих частей пропорциональности (4) совпадают, поэтому коэффициент пропорциональности — величина безразмерная. В системе СИ она полагается равной единице, и мы получаем:

(5)

Это и есть закон электромагнитной индукции или закон Фарадея. Дадим его словесную формулировку.

Закон электромагнитной индукции Фарадея. При изменении магнитного потока, пронизывающего контур, в этом контуре возникает ЭДС индукции, равная модулю скорости изменения магнитного потока.

Правило Ленца

Магнитный поток, изменение которого приводит к появлению индукционного тока в контуре, мы будем называть внешним магнитным потоком. А само магнитное поле, которое создаёт этот магнитный поток, мы будем называть внешним магнитным полем.

Зачем нам эти термины? Дело в том, что индукционный ток, возникающий в контуре, создаёт своё собственное магнитное поле, которое по принципу суперпозиции складывается с внешним магнитным полем.

Соответственно, наряду с внешним магнитным потоком через контур будет проходить собственный магнитный поток, создаваемый магнитным полем индукционного тока.

Оказывается, эти два магнитных потока — собственный и внешний — связаны между собой строго определённым образом.

Правило Ленца . Индукционный ток всегда имеет такое направление, что собственный магнитный поток препятствует изменению внешнего магнитного потока .

Правило Ленца позволяет находить направление индукционного тока в любой ситуации.

Рассмотрим некоторые примеры применения правила Ленца.

Предположим, что контур пронизывается магнитным полем, которое возрастает со временем (рис. (3)). Например, мы приближаем снизу к контуру магнит, северный полюс которого направлен в данном случае вверх, к контуру.

Магнитный поток через контур увеличивается. Индукционный ток будет иметь такое направление, чтобы создаваемый им магнитный поток препятствовал увеличению внешнего магнитного потока. Для этого магнитное поле, создаваемое индукционным током, должно быть направлено против внешнего магнитного поля.

Индукционный ток течёт против часовой стрелки, если смотреть со стороны создаваемого им магнитного поля. В данном случае ток будет направлен по часовой стрелке, если смотреть сверху, со стороны внешнего магнитного поля, как и показано на (рис. (3)).

Рис. 3. Магнитный поток возрастает

Теперь предположим, что магнитное поле, пронизывающее контур, уменьшается со временем (рис. 4). Например, мы удаляем магнит вниз от контура, а северный полюс магнита направлен на контур.

Рис. 4. Магнитный поток убывает

Магнитный поток через контур уменьшается. Индукционный ток будет иметь такое направление, чтобы его собственный магнитный поток поддерживал внешний магнитный поток, препятствуя его убыванию. Для этого магнитное поле индукционного тока должно быть направлено в ту же сторону , что и внешнее магнитное поле.

В этом случае индукционный ток потечёт против часовой стрелки, если смотреть сверху, со стороны обоих магнитных полей.

Взаимодействие магнита с контуром

Итак, приближение или удаление магнита приводит к появлению в контуре индукционного тока, направление которого определяется правилом Ленца. Но ведь магнитное поле действует на ток! Появится сила Ампера, действующая на контур со стороны поля магнита. Куда будет направлена эта сила?

Если вы хотите хорошо разобраться в правиле Ленца и в определении направления силы Ампера, попробуйте ответить на данный вопрос самостоятельно. Это не очень простое упражнение и отличная задача для С1 на ЕГЭ. Рассмотрите четыре возможных случая.

1. Магнит приближаем к контуру, северный полюс направлен на контур.
2. Магнит удаляем от контура, северный полюс направлен на контур.
3. Магнит приближаем к контуру, южный полюс направлен на контур.
4. Магнит удаляем от контура, южный полюс направлен на контур.

Не забывайте, что поле магнита не однородно: линии поля расходятся от северного полюса и сходятся к южному. Это очень существенно для определения результирующей силы Ампера. Результат получается следующий.

Если приближать магнит, то контур отталкивается от магнита. Если удалять магнит, то контур притягивается к магниту. Таким образом, если контур подвешен на нити, то он всегда будет отклоняться в сторону движения магнита, словно следуя за ним. Расположение полюсов магнита при этом роли не играет .

Уж во всяком случае вы должны запомнить этот факт — вдруг такой вопрос попадётся в части А1

Результат этот можно объяснить и из совершенно общих соображений — при помощи закона сохранения энергии.

Допустим, мы приближаем магнит к контуру. В контуре появляется индукционный ток. Но для создания тока надо совершить работу! Кто её совершает? В конечном счёте — мы, перемещая магнит. Мы совершаем положительную механическую работу, которая преобразуется в положительную работу возникающих в контуре сторонних сил, создающих индукционный ток.

Итак, наша работа по перемещению магнита должна быть положительна . Это значит, что мы, приближая магнит, должны преодолевать силу взаимодействия магнита с контуром, которая, стало быть, является силой отталкивания .

Теперь удаляем магнит. Повторите, пожалуйста, эти рассуждения и убедитесь, что между магнитом и контуром должна возникнуть сила притяжения.

Закон Фарадея + Правило Ленца = Снятие модуля

Выше мы обещали снять модуль в законе Фарадея (5). Правило Ленца позволяет это сделать. Но сначала нам нужно будет договориться о знаке ЭДС индукции — ведь без модуля, стоящего в правой части (5), величина ЭДС может получаться как положительной, так и отрицательной.

Прежде всего, фиксируется одно из двух возможных направлений обхода контура. Это направление объявляется положительным . Противоположное направление обхода контура называется, соответственно, отрицательным . Какое именно направление обхода мы берём в качестве положительного, роли не играет — важно лишь сделать этот выбор.

Магнитный поток через контур считается положительным , если магнитное поле, пронизывающее контур, направлено туда, глядя откуда обход контура в положительном направлении совершается против часовой стрелки. Если же с конца вектора магнитной индукции положительное направление обхода видится по часовой стрелке, то магнитный поток считается отрицательным .

ЭДС индукции считается положительной , если индукционный ток течёт в положительном направлении. В этом случае направление сторонних сил, возникающих в контуре при изменении магнитного потока через него, совпадает с положительным направлением обхода контура.

Наоборот, ЭДС индукции считается отрицательной , если индукционный ток течёт в отрицательном направлении. Сторонние силы в данном случае также будут действовать вдоль отрицательного направления обхода контура.

Итак, пусть контур находится в магнитном поле . Фиксируем направление положительного обхода контура. Предположим, что магнитное поле направлено туда, глядя откуда положительный обход совершается против часовой стрелки. Тогда магнитный поток положителен: .

Предположим, далее, что магнитный поток увеличивается . Согласно правилу Ленца индукционный ток потечёт в отрицательном направлении (рис. 5).

Рис. 5. Магнитный поток возрастает

Стало быть, в данном случае имеем . Знак ЭДС индукции оказался противоположен знаку скорости изменения магнитного потока. Проверим это в другой ситуации.

А именно, предположим теперь, что магнитный поток убывает . По правилу Ленца индукционный ток потечёт в положительном направлении. Стало быть, (рис. 6).

Рис. 6. Магнитный поток возрастает

Таков в действительности общий факт: при нашей договорённости о знаках правило Ленца всегда приводит к тому, что знак ЭДС индукции противоположен знаку скорости изменения магнитного потока :

(6)

Тем самым ликвидирован знак модуля в законе электромагнитной индукции Фарадея.

Вихревое электрическое поле

Рассмотрим неподвижный контур, находящийся в переменном магнитном поле. Каков же механизм возникновения индукционного тока в контуре? А именно, какие силы вызывают движение свободных зарядов, какова природа этих сторонних сил?

Пытаясь ответить на эти вопросы, великий английский физик Максвелл открыл фундаментальное свойство природы: меняющееся во времени магнитное поле порождает поле электрическое . Именно это электрическое поле и действует на свободные заряды, вызывая индукционный ток.

Линии возникающего электрического поля оказываются замкнутыми, в связи с чем оно было названо вихревым электрическим полем . Линии вихревого электрического поля идут вокруг линий магнитного поля и направлены следующим образом.

Пусть магнитное поле увеличивается. Если в нём находится проводящий контур, то индукционный ток потечёт в соответствии с правилом Ленца — по часовой стрелке, если смотреть с конца вектора . Значит, туда же направлена и сила, действующая со стороны вихревого электрического поля на положительные свободные заряды контура; значит, именно туда направлен вектор напряжённости вихревого электрического поля.

Итак, линии напряжённости вихревого электрического поля направлены в данном случае по часовой стрелке (смотрим с конца вектора , (рис. 7).

Рис. 7. Вихревое электрическое поле при увеличении магнитного поля

Наоборот, если магнитное поле убывает, то линии напряжённости вихревого электрического поля направлены против часовой стрелки (рис. 8).

Рис. 8. Вихревое электрическое поле при уменьшении магнитного поля

Теперь мы можем глубже понять явление электромагнитной индукции. Суть его состоит именно в том, что переменное магнитное поле порождает вихревое электрическое поле. Данный эффект не зависит от того, присутствует ли в магнитном поле замкнутый проводящий контур или нет; с помощью контура мы лишь обнаруживаем это явление, наблюдая индукционный ток.

Вихревое электрическое поле по некоторым свойствам отличается от уже известных нам электрических полей: электростатического поля и стационарного поля зарядов, образующих постоянный ток.

1. Линии вихревого поля замкнуты, тогда как линии электростатического и стационарного полей начинаются на положительных зарядах и оканчиваются на отрицательных.
2. Вихревое поле непотенциально: его работа перемещению заряда по замкнутому контуру не равна нулю. Иначе вихревое поле не могло бы создавать электрический ток! В то же время, как мы знаем, электростатическое и стационарное поля являются потенциальными.

Итак, ЭДС индукции в неподвижном контуре — это работа вихревого электрического поля по перемещению единичного положительного заряда вокруг контура .

Пусть, например, контур является кольцом радиуса и пронизывается однородным переменным магнитным полем. Тогда напряжённость вихревого электрического поля одинакова во всех точках кольца. Работа силы , с которой вихревое поле действует на заряд , равна:

Следовательно, для ЭДС индукции получаем:

ЭДС индукции в движущемся проводнике

Если проводник перемещается в постоянном магнитном поле, то в нём также появляется ЭДС индукции. Однако причиной теперь служит не вихревое электрическое поле (оно не возникает — ведь магнитное поле постоянно), а действие силы Лоренца на свободные заряды проводника.

Рассмотрим ситуацию, которая часто встречается в задачах. В горизонтальной плоскости расположены параллельные рельсы, расстояние между которыми равно . Рельсы находятся в вертикальном однородном магнитном поле . По рельсам движется тонкий проводящий стержень со скоростью ; он всё время остаётся перпендикулярным рельсам (рис. 9).

Рис. 9. Движение проводника в магнитном поле

Возьмём внутри стержня положительный свободный заряд . Вследствие движения этого заряда вместе со стержнем со скоростью на заряд будет действовать сила Лоренца:

Направлена эта сила вдоль оси стержня, как показано на рисунке (убедитесь в этом сами — не забывайте правило часовой стрелки или левой руки!).

Сила Лоренца играет в данном случае роль сторонней силы: она приводит в движение свободные заряды стержня. При перемещении заряда от точки к точке наша сторонняя сила совершит работу:

(Длину стержня мы также считаем равной .) Стало быть, ЭДС индукции в стержне окажется равной:

(7)

Таким образом, стержень аналогичен источнику тока с положительной клеммой и отрицательной клеммой . Внутри стержня за счёт действия сторонней силы Лоренца происходит разделение зарядов: положительные заряды двигаются к точке , отрицательные — к точке .

Допустим сначала,что рельсы непроводят ток.Тогда движение зарядов в стержне постепенно прекратится. Ведь по мере накопления положительных зарядов на торце и отрицательных зарядов на торце будет возрастать кулоновская сила, с которой положительный свободный заряд отталкивается от и притягивается к — и в какой-то момент эта кулоновская сила уравновесит силу Лоренца. Между концами стержня установится разность потенциалов, равная ЭДС индукции (7).

Теперь предположим, что рельсы и перемычка являются проводящими. Тогда в цепи возникнет индукционный ток; он пойдёт в направлении (от «плюса источника» к «минусу» N). Предположим, что сопротивление стержня равно (это аналог внутреннего сопротивления источника тока), а сопротивление участка равно (сопротивление внешней цепи). Тогда сила индукционного тока найдётся по закону Ома для полной цепи:

Замечательно, что выражение (7) для ЭДС индукции можно получить также с помощью закона Фарадея. Сделаем это.
За время наш стержень проходит путь и занимает положение (рис. 9). Площадь контура возрастает на величину площади прямоугольника :

Магнитный поток через контур увеличивается. Приращение магнитного потока равно:

Скорость изменения магнитного потока положительна и равна ЭДС индукции:

Мы получили тот же самый результат, что и в (7). Направление индукционного тока, заметим, подчиняется правилу Ленца. Действительно, раз ток течёт в направлении , то его магнитное поле направлено противоположно внешнему полю и, стало быть, препятствует возрастанию магнитного потока через контур.

На этом примере мы видим, что в ситуациях, когда проводник движется в магнитном поле, можно действовать двояко: либо с привлечением силы Лоренца как сторонней силы, либо с помощью закона Фарадея. Результаты будут получаться одинаковые.

Электромагнитная индукция – FIZI4KA

Явление электромагнитной индукции

Электромагнитная индукция – явление возникновения тока в замкнутом проводящем контуре при изменении магнитного потока, пронизывающего его.

Явление электромагнитной индукции было открыто М. Фарадеем.

Опыты Фарадея

  • На одну непроводящую основу были намотаны две катушки: витки первой катушки были расположены между витками второй. Витки одной катушки были замкнуты на гальванометр, а второй – подключены к источнику тока. При замыкании ключа и протекании тока по второй катушке в первой возникал импульс тока. При размыкании ключа также наблюдался импульс тока, но ток через гальванометр тек в противоположном направлении.
  • Первая катушка была подключена к источнику тока, вторая, подключенная к гальванометру, перемещалась относительно нее. При приближении или удалении катушки фиксировался ток.
  • Катушка замкнута на гальванометр, а магнит движется – вдвигается (выдвигается) – относительно катушки.

Опыты показали, что индукционный ток возникает только при изменении линий магнитной индукции. Направление тока будет различно при увеличении числа линий и при их уменьшении.

Сила индукционного тока зависит от скорости изменения магнитного потока. Может изменяться само поле, или контур может перемещаться в неоднородном магнитном поле.

Объяснения возникновения индукционного тока

Ток в цепи может существовать, когда на свободные заряды действуют сторонние силы. Работа этих сил по перемещению единичного положительного заряда вдоль замкнутого контура равна ЭДС. Значит, при изменении числа магнитных линий через поверхность, ограниченную контуром, в нем появляется ЭДС, которую называют ЭДС индукции.

Электроны в неподвижном проводнике могут приводиться в движение только электрическим полем. Это электрическое поле порождается изменяющимся во времени магнитным полем. Его называют вихревым электрическим полем. Представление о вихревом электрическом поле было введено в физику великим английским физиком Дж. Максвеллом в 1861 году.

Свойства вихревого электрического поля:

  • источник – переменное магнитное поле;
  • обнаруживается по действию на заряд;
  • не является потенциальным;
  • линии поля замкнутые.

Работа этого поля при перемещении единичного положительного заряда по замкнутому контуру равна ЭДС индукции в неподвижном проводнике.

Магнитный поток

Магнитным потоком через площадь ​\( S \)​ контура называют скалярную физическую величину, равную произведению модуля вектора магнитной индукции ​\( B \)​, площади поверхности ​\( S \)​, пронизываемой данным потоком, и косинуса угла ​\( \alpha \)​ между направлением вектора магнитной индукции и вектора нормали (перпендикуляра к плоскости данной поверхности):

Обозначение – ​\( \Phi \)​, единица измерения в СИ – вебер (Вб).

Магнитный поток в 1 вебер создается однородным магнитным полем с индукцией 1 Тл через поверхность площадью 1 м2, расположенную перпендикулярно вектору магнитной индукции:

Магнитный поток можно наглядно представить как величину, пропорциональную числу магнитных линий, проходящих через данную площадь.

В зависимости от угла ​\( \alpha \)​ магнитный поток может быть положительным (\( \alpha \) < 90°) или отрицательным (\( \alpha \) > 90°). Если \( \alpha \) = 90°, то магнитный поток равен 0.

Изменить магнитный поток можно меняя площадь контура, модуль индукции поля или расположение контура в магнитном поле (поворачивая его).

В случае неоднородного магнитного поля и неплоского контура магнитный поток находят как сумму магнитных потоков, пронизывающих площадь каждого из участков, на которые можно разбить данную поверхность.

Закон электромагнитной индукции Фарадея

Закон электромагнитной индукции (закон Фарадея):

ЭДС индукции в замкнутом контуре равна и противоположна по знаку скорости изменения магнитного потока через поверхность, ограниченную контуром:

Знак «–» в формуле позволяет учесть направление индукционного тока. Индукционный ток в замкнутом контуре имеет всегда такое направление, чтобы магнитный поток поля, созданного этим током сквозь поверхность, ограниченную контуром, уменьшал бы те изменения поля, которые вызвали появление индукционного тока.

Если контур состоит из ​\( N \)​ витков, то ЭДС индукции:

Сила индукционного тока в замкнутом проводящем контуре с сопротивлением ​\( R \)​:

При движении проводника длиной ​\( l \)​ со скоростью ​\( v \)​ в постоянном однородном магнитном поле с индукцией ​\( \vec{B} \)​ ЭДС электромагнитной индукции равна:

где ​\( \alpha \)​ – угол между векторами ​\( \vec{B} \)​ и \( \vec{v} \).

Возникновение ЭДС индукции в движущемся в магнитном поле проводнике объясняется действием силы Лоренца на свободные заряды в движущихся проводниках. Сила Лоренца играет в этом случае роль сторонней силы.

Движущийся в магнитном поле проводник, по которому протекает индукционный ток, испытывает магнитное торможение. Полная работа силы Лоренца равна нулю.

Количество теплоты в контуре выделяется либо за счет работы внешней силы, которая поддерживает скорость проводника неизменной, либо за счет уменьшения кинетической энергии проводника.

Важно!
Изменение магнитного потока, пронизывающего замкнутый контур, может происходить по двум причинам:

  • магнитный поток изменяется вследствие перемещения контура или его частей в постоянном во времени магнитном поле. Это случай, когда проводники, а вместе с ними и свободные носители заряда, движутся в магнитном поле;
  • вторая причина изменения магнитного потока, пронизывающего контур, – изменение во времени магнитного поля при неподвижном контуре. В этом случае возникновение ЭДС индукции уже нельзя объяснить действием силы Лоренца. Явление электромагнитной индукции в неподвижных проводниках, возникающее при изменении окружающего магнитного поля, также описывается формулой Фарадея.

Таким образом, явления индукции в движущихся и неподвижных проводниках протекают одинаково, но физическая причина возникновения индукционного тока оказывается в этих двух случаях различной:

  • в случае движущихся проводников ЭДС индукции обусловлена силой Лоренца;
  • в случае неподвижных проводников ЭДС индукции является следствием действия на свободные заряды вихревого электрического поля, возникающего при изменении магнитного поля.

Правило Ленца

Направление индукционного тока определяется по правилу Ленца: индукционный ток, возбуждаемый в замкнутом контуре при изменении магнитного потока, всегда направлен так, что создаваемое им магнитное поле препятствует изменению магнитного потока, вызывающего индукционный ток.

Алгоритм решения задач с использованием правила Ленца:

  • определить направление линий магнитной индукции внешнего магнитного поля;
  • выяснить, как изменяется магнитный поток;
  • определить направление линий магнитной индукции магнитного поля индукционного тока: если магнитный поток уменьшается, то они сонаправлены с линиями внешнего магнитного поля; если магнитный поток увеличивается, – противоположно направлению линий магнитной индукции внешнего поля;
  • по правилу буравчика, зная направление линий индукции магнитного поля индукционного тока, определить направление индукционного тока.

Правило Ленца имеет глубокий физический смысл – оно выражает закон сохранения энергии.

Самоиндукция

Самоиндукция – это явление возникновения ЭДС индукции в проводнике в результате изменения тока в нем.

При изменении силы тока в катушке происходит изменение магнитного потока, создаваемого этим током. Изменение магнитного потока, пронизывающего катушку, должно вызывать появление ЭДС индукции в катушке.

В соответствии с правилом Ленца ЭДС самоиндукции препятствует нарастанию силы тока при включении и убыванию силы тока при выключении цепи.

Это приводит к тому, что при замыкании цепи, в которой есть источник тока с постоянной ЭДС, сила тока устанавливается через некоторое время.

При отключении источника ток также не прекращается мгновенно. Возникающая при этом ЭДС самоиндукции может превышать ЭДС источника.

Явление самоиндукции можно наблюдать, собрав электрическую цепь из катушки с большой индуктивностью, резистора, двух одинаковых ламп накаливания и источника тока. Резистор должен иметь такое же электрическое сопротивление, как и провод катушки.

Опыт показывает, что при замыкании цепи электрическая лампа, включенная последовательно с катушкой, загорается несколько позже, чем лампа, включенная последовательно с резистором. Нарастанию тока в цепи катушки при замыкании препятствует ЭДС самоиндукции, возникающая при возрастании магнитного потока в катушке.

При отключении источника тока вспыхивают обе лампы. В этом случае ток в цепи поддерживается ЭДС самоиндукции, возникающей при убывании магнитного потока в катушке.

ЭДС самоиндукции ​\( \varepsilon_{is} \)​, возникающая в катушке с индуктивностью ​\( L \)​, по закону электромагнитной индукции равна:

ЭДС самоиндукции прямо пропорциональна индуктивности катушки и скорости изменения силы тока в катушке.

Индуктивность

Электрический ток, проходящий по проводнику, создает вокруг него магнитное поле. Магнитный поток ​\( \Phi \)​ через контур из этого проводника пропорционален модулю индукции ​\( \vec{B} \)​ магнитного поля внутри контура, а индукция магнитного поля, в свою очередь, пропорциональна силе тока в проводнике.

Следовательно, магнитный поток через контур прямо пропорционален силе тока в контуре:

Индуктивность – коэффициент пропорциональности ​\( L \)​ между силой тока ​\( I \)​ в контуре и магнитным потоком ​\( \Phi \)​, создаваемым этим током:

Индуктивность зависит от размеров и формы проводника, от магнитных свойств среды, в которой находится проводник.

Единица индуктивности в СИ – генри (Гн). Индуктивность контура равна 1 генри, если при силе постоянного тока 1 ампер магнитный поток через контур равен 1 вебер:

Можно дать второе определение единицы индуктивности: элемент электрической цепи обладает индуктивностью в 1 Гн, если при равномерном изменении силы тока в цепи на 1 ампер за 1 с в нем возникает ЭДС самоиндукции 1 вольт.

Энергия магнитного поля

При отключении катушки индуктивности от источника тока лампа накаливания, включенная параллельно катушке, дает кратковременную вспышку. Ток в цепи возникает под действием ЭДС самоиндукции.

Источником энергии, выделяющейся при этом в электрической цепи, является магнитное поле катушки.

Для создания тока в контуре с индуктивностью необходимо совершить работу на преодоление ЭДС самоиндукции. Энергия магнитного поля тока вычисляется по формуле:

Основные формулы раздела «Электромагнитная индукция»

Алгоритм решения задач по теме «Электромагнитная индукция»:

1. Внимательно прочитать условие задачи. Установить причины изменения магнитного потока, пронизывающего контур.

2. Записать формулу:

  • закона электромагнитной индукции;
  • ЭДС индукции в движущемся проводнике, если в задаче рассматривается поступательно движущийся проводник; если в задаче рассматривается электрическая цепь, содержащая источник тока, и возникающая на одном из участков ЭДС индукции, вызванная движением проводника в магнитном поле, то сначала нужно определить величину и направление ЭДС индукции. После этого задача решается по аналогии с задачами на расчет цепи постоянного тока с несколькими источниками.

3. Записать выражение для изменения магнитного потока и подставить в формулу закона электромагнитной индукции.

4. Записать математически все дополнительные условия (чаще всего это формулы закона Ома для полной цепи, силы Ампера или силы Лоренца, формулы кинематики и динамики).

5. Решить полученную систему уравнений относительно искомой величины.

6. Решение проверить.

Электромагнитная индукция

3.1 (62.38%) 42 votes

Способы определения ЭДС индукции в движущихся проводниках

 

Нахождение ЭДС индукции через силу Лоренца

Магнитный поток через контур может изменяться по следующим причинам:

В обоих этих случаях будет выполняться закон электромагнитной индукции. При этом происхождение электродвижущей силы в этих случаях различное. Рассмотрим подробнее второй из этих случаев

В данном случае проводник движется в магнитном поле. Вместе с проводником совершают движение и все заряды, которые находятся внутри проводника. На каждый из таких зарядов со стороны магнитного поля будет действовать сила Лоренца. Она и будет способствовать перемещению зарядов внутри проводника.

  • ЭДС индукции в данном случае будет иметь магнитное происхождение.

Рассмотрим следующий опыт: магнитный контур, у которого одна сторона подвижная, помещают в однородное магнитное поле. Подвижная сторона длиной l начинает скользить вдоль сторон MD и NC с постоянной скоростью V. При этом она постоянно остаётся параллельной стороне СD. Вектор магнитной индукции поля будет перпендикулярен проводнику и составлять угол а с направлением его скорости. На следующем рисунке представлена лабораторная установка для этого опыта:

Сила Лоренца, действующая на движущуюся частицу, вычисляется по следующей формуле:

Fл = |q|*V*B*sin(a).

Сила Лоренца будет направлена вдоль отрезка MN. Рассчитаем работу силы Лоренца:

A = Fл*l = |q|*V*B*l*sin(a).

ЭДС индукции — это отношение работы, совершаемой силой при перемещении единичного положительного заряда, к величине этого заряда. Следовательно, имеем:

Ei = A/|q| = V*B*l*sin(a).

Эта формула будет справедлива для любого проводника, движущегося в с постоянной скоростью в магнитном поле. ЭДС индукции будет только в этом проводнике, так как остальные проводники контура остаются неподвижными. Очевидно, что ЭДС индукции во всем контуре будет равняться ЭДС индукции в подвижном проводнике.

ЭДС из закона электромагнитной индукции 

Магнитный поток через тот же контур, что и в примере выше, будет равняться: 

Ф = B*S*cos(90-a) = B*S*sin(a).

Здесь угол (90-а) = угол между вектором магнитной индукции и нормалью к поверхности контура. За некоторое время ∆t площадь контура будет изменяться на ∆S = -l*V*∆t. Знак «минус» показывает, что площадь уменьшается. При этом за это время магнитный поток изменится:

∆Ф = -B*l*V*sin(a).

Тогда ЭДС индукции равна:

Ei = -∆Ф/∆t = B*l*V*sin(a).

Если весь контур будет двигаться внутри однородного магнитного поля с постоянной скоростью, то ЭДС индукции будет равняться нулю, так как будет отсутствовать изменение магнитного потока.

  • ЭДС индукции будет возникать и при повороте рамки внутри магнитного поля.

Нужна помощь в учебе?



Предыдущая тема: Закон электромагнитной индукции:магнитный поток и электродвижущая сила
Следующая тема:&nbsp&nbsp&nbspЭлектродинамический микрофон: самоиндукция

Задачи на эдс индукции

Методика решения задач на применение закона электромагнитной индукции
будет полезна как учащимся, так и абитуриентам

Решая задачи на закон электромагнитной индукции, удобно пользоваться следующими рекомендациями.

    Анализируя условия задачи, необходимо прежде, всего установить причины изменения магнитного потока, связанного с контуром, и определить, какая из величин В,S или α, входящих в выражение для магнитного потока Ф, изменяется с течением времени. После этого нужно записать закон электромагнитной индукции Фарадея для одного витка или для нескольких витков
    .
    Если в задаче речь идет о поступательном движении прямого проводника, то э.д.с. индукции определяют по формуле
    ,
    вытекающей из закона электромагнитной индукции.

Затем выражение для Ф надо представить в развернутом виде. Для этого выбирают два момента времени t1 и t2 и для каждого из них определяют потоки Ф1 и Ф2, связанные с данным контуром. Изменение магнитного потока за время Δt = t2t1 в зависимости от условия задачи, будет равно или

если изменяется магнитная индукция поля, в котором находится контур, или

если изменяется положение рамки в поле, или, наконец,

где ΔS — площадь, описанная в пространстве движущимся проводником.

  • Далее надо подставить выражение для ΔФ в исходную формулу закона электромагнитной индукции и, записав дополнительные условия, решить полученные уравнения совместно относительно искомой величины.
    Наибольшие затруднения возникают обычно при расчете электрических цепей, содержащих аккумуляторы, когда на одном из участков цепи возникает э.д.с. индукции, вызванная движением проводника в магнитном поле.
    Решение в этом случае нужно начинать с определения величины и направления этой э.д.с, после чего задача сведется к расчету обычной цепи постоянного тока с несколькими источниками э.д.с.(см. п.3е в методике решения задач электродинамики), соединенными между собой последовательно или параллельно.
  • —————————————————————————————————

    вернуться на стр. «ЭМ-индукция»«Физика»вернуться к методике решения задач

    Закон ЭМ индукции Фарадея.

    1. Магнитный поток внутри катушки с числом витков равным 400, за 0,2 с изменился от 0,1 Вб до 0,9 Вб. Определить ЭДС, индуцируемую в катушке.

    2. Определить магнитный поток, проходящий через прямоугольную площадку со сторонами 20х40 см, если она помещена в однородное магнитное поле с индукцией в 5 Тл под углом 60° к линиям магнитной индукции поля.

    3. Сколько витков должна иметь катушка, чтобы при изменении магнитного потока внутри нее от 0,024 до 0,056 Вб за 0,32 с в ней создавалась средняя э.д.с. 10 В?

    ЭДС индукции в движущихся проводниках.

    1. Определить ЭДС индукции на концах крыльев самолета Ан-2, имеющих длину 12,4 м, если скорость самолёта при горизонтальном полёте 180 км/ч, а вертикальная составляющая вектора индукции магнитного поля Земли 0,5·10 -4 Тл.

    2. Найти ЭДС индукции на крыльях самолета Ту-204, имеющих длину 42 м, летящего горизонтально со скоростью 850 км/ч, если вертикальная составляющая вектора индукции магнитного поля Земли 5·10 -5 Тл.

    ЭДС самоиндукции

    1. В катушке возникает магнитный поток 0,015 Вб, когда по ее виткам проходит ток 5,0 А. Сколько витков содержит катушка, если ее индуктивность 60 мГ?

    2. Во сколько раз изменится индуктивность катушки без сердечника, если число витков в ней увеличить в два раза?

    3. Какая э.д.с. самоиндукции возникнет в катушке с индуктивностью 68 мГ, если ток 3,8 А исчезнет в ней за 0,012 с?

    4. Определить индуктивность катушки, если при ослаблении в ней тока на 2,8 А за 62 мс в катушке появляется средняя э.д.с. самоиндукции 14 В.

    5. За сколько времени в катушке с индуктивностью 240 мГ происходит нарастание тока от нуля до 11,4 А, если при этом возникает средняя э.д.с. самоиндукции 30 В?

    Энергия электромагнитного поля

    1. По катушке с индуктивностью 0,6 Гн течет ток силой 20 А. Какова энергия магнитного поля катушки? Как изменится эта энергия при возпастании силы тока в 2 раза? в 3 раза?

    2. Какой силы ток нужно пропускать по обмотке дросселя с индуктивностью 0,5 Гн, чтобы энергия поля оказалась равной 100 Дж?

    3. Энергия магнитного поля какой катушки больше и во сколько раз, если первая имеет характеристики: I1=10A, L1=20 Гн, вторая: I2=20A, L2=10 Гн?

    4. Определить энергию магнитного поля катушки, в которой при токе 7,5 А магнитный поток равен 2,3·10 -3 Вб. Число витков в катушке 120.

    5. Определить индуктивность катушки, если при токе 6,2 А ее магнитное поле обладает энергией 0,32 Дж.

    6. Магнитное поле катушки с индуктивностью 95 мГ обладает энергией 0,19 Дж. Чему равна сила токав катушке?

    вернуться на стр. «ЭМ-индукция»«Физика»вернуться к методике решения задач

    Балаш В.А. «Задачи по физике и методы их решения». Пособие для учителей. М., «Просвещение», 1974.
    Мартынов И.М., Хозяинова Э.М. «Дидактический материал по физике 9 кл.» М., «Просвещение», 1978.
    Марон А.Е., Мякишев Г.Я. «Физика». Учебное пособие для 11 кл. вечерней (заоч.) средн. шк. и самообразования. М., «Просвещение», 1992.
    Гладкова Р.А., Добронравов В.Е., Жданов Л.С., Цодиков Ф.С. «Сборник задач и вопросов по физике» для сред. спец. уч. заведений М., «Наука», 1975.

    На рисунке 100 представлены различные случаи электромагнитной индукции. Сформулировать и решить задачу для каждого случая
    РЕШЕНИЕ

    Будет ли в рамке ABCD (рис. 101) возникать индукционный ток, если рамку: а) вращать относительно неподвижного проводника с током ОО , как показано на рисунке; б) вращать вокруг стороны АВ; в) вращать вокруг стороны ВС; г) двигать поступательно в вертикальном направлении; д) двигать поступательно в горизонтальном направлении
    РЕШЕНИЕ

    Три одинаковых полосовых магнита падают в вертикальном положении одновременно с одной высоты. Первый падает свободно, второй во время падения проходит сквозь незамкнутый соленоид, третий — сквозь замкнутый соленоид. Сравнить время падения магнитов. Ответы обосновать на основании правила Ленца и закона сохранения энергии
    РЕШЕНИЕ

    Определить направление индукционного тока, возникающего в витке В (рис. 102), если в цепи витка А ключ замыкают и если этот ключ размыкают. Указать также направление индукционного тока, если при замкнутом ключе скользящий контакт реостата передвигают вправо или его передвигают влево
    РЕШЕНИЕ

    Если вращать магнит (рис. 103), то замкнутый виток проволоки, укрепленный на оси, начинает вращаться. Объяснить явление и определить направление вращения витка
    РЕШЕНИЕ

    Если клеммы двух демонстрационных гальванометров соединить проводами и затем покачиванием одного из приборов вызвать колебание его стрелки, то и у другого прибора стрелка тоже начнет колебаться. Объяснить опыт и при возможности проверить
    РЕШЕНИЕ

    Почему колебания стрелки компаса быстрее затухают, если корпус прибора латунный, и медленнее, если корпус прибора пластмассовый
    РЕШЕНИЕ

    Объяснить принцип торможения трамвая, когда водитель, отключив двигатель от контактной сети (рис. 104), переводит его в режим генератора (ключ переводится из положения 1 в положение 2). Как зависит ускорение (быстрота торможения) трамвая: а) от нагрузки (сопротивления резистора) при данной скорости движения трамвая; б) от скорости трамвая при данной нагрузке
    РЕШЕНИЕ

    По какому закону должен изменяться магнитный поток в зависимости от времени, чтобы ЭДС индукции, возникающая в контуре, оставалась постоянной
    РЕШЕНИЕ

    За 5 мс магнитный поток, пронизывающий контур, убывает с 9 до 4 мВб. Найти ЭДС индукции в контуре
    РЕШЕНИЕ

    Найти скорость изменения магнитного потока в соленоиде из 2000 витков при возбуждении в нем ЭДС индукции 120 В
    РЕШЕНИЕ

    Сколько витков должна содержать катушка с площадью поперечного сечения 50 см2, чтобы при изменении магнитной индукции от 0,2 до 0,3 Тл в течение 4 мс в ней возбуждалась ЭДС 10 В
    РЕШЕНИЕ

    Внутри витка радиусом 5 см магнитный поток изменился на 18,6 мВб за 5,9 мс. Найти напряженность вихревого электрического поля в витке
    РЕШЕНИЕ

    Какой заряд q пройдет через поперечное сечение витка, сопротивление которого R = 0,03 Ом, при уменьшении магнитного потока внутри витка на ΔФ =12 мВб
    РЕШЕНИЕ

    В магнитное поле индукцией В = 0,1 Тл помещен контур, выполненный в форме кругового витка радиусом R = 3,4 см. Виток сделан из медной проволоки, площадь поперечного сечения которой s = 1 мм2. Нормаль к плоскости витка совпадает с линиями индукции поля. Какой заряд пройдет через поперечное сечение витка при исчезновении поля
    РЕШЕНИЕ

    В витке, выполненном из алюминиевого провода длиной 10 см и площадью поперечного сечения 1,4 мм2, скорость изменения магнитного потока 10 мВб/c. Найти силу индукционного тока
    РЕШЕНИЕ

    Найти ЭДС индукции в проводнике с длиной активной части 0,25 м, перемещающемся в однородном магнитном поле индукцией 8 мТл со скоростью 5 м/с под углом 30° к вектору магнитной индукции
    РЕШЕНИЕ

    С какой скоростью надо перемещать проводник под углом 60° к линиям индукции магнитного поля, чтобы в проводнике возбуждалась ЭДС индукции 1 В? Индукция магнитного поля равна 0,2 Тл. Длина активной части 1 м
    РЕШЕНИЕ

    Проводник MN (рис. 105) с длиной активной части 1 м и сопротивлением 2 Ом находится в однородном магнитном поле индукцией 0,1 Тл. Проводник подключен к источнику тока с ЭДС 1 В (внутренним сопротивлением источника и сопротивлением подводящих проводников пренебречь). Какова сила тока в проводнике, если: а) проводник покоится; б) проводник движется вправо со скоростью 4 м/с; в) проводник движется влево с такой же по модулю скоростью? В каком направлении и с какой скоростью надо перемещать проводник, чтобы через него не шел ток
    РЕШЕНИЕ

    Какова индуктивность контура, если при силе тока 5 А в нем возникает магнитный поток 0,5 мВб
    РЕШЕНИЕ

    Какой магнитный поток возникает в контуре индуктивностью 0,2 мГн при силе тока 10 А
    РЕШЕНИЕ

    Найти индуктивность проводника, в котором при равномерном изменении силы тока на 2 А в течение 0,25 с возбуждается ЭДС самоиндукции 20 мВ
    РЕШЕНИЕ

    Какая ЭДС самоиндукции возбуждается в обмотке электромагнита индуктивностью 0,4 Гн при равномерном изменении силы тока в ней на 5 А за 0,02 с
    РЕШЕНИЕ

    Почему отключение от питающей сети мощных электродвигателей производят плавно и медленно при помощи реостата
    РЕШЕНИЕ

    Последовательно с катушкой школьного трансформатора, надетой на разомкнутый сердечник, включена лампочка от карманного фонаря. В цепь подано такое напряжение, что лампочка горит в полный накал. Как изменяется яркость лампочки, если: а) сердечник замкнуть ярмом; б) некоторое время держать ярмо неподвижным; в) вынуть ярмо? При возможности проверить на опыте, положив на сердечник спичку (иначе ярмо трудно оторвать от сердечника)
    РЕШЕНИЕ

    В катушке индуктивностью 0,6 Гн сила тока равна 20 А. Какова энергия магнитного поля этой катушки? Как изменится энергия поля, если сила тока уменьшится вдвое?
    РЕШЕНИЕ

    Какой должна быть сила тока в обмотке дросселя индуктивностью 0,5 Гн, чтобы энергия поля оказалась равной 1 Дж
    РЕШЕНИЕ

    Найти энергию магнитного поля соленоида, в котором при силе тока 10 А возникает магнитный поток 0,5 Вб
    РЕШЕНИЕ

    На катушке сопротивлением 8,2 Ом и индуктивностью 25 мГн поддерживается постоянное напряжение 55 В. Сколько энергии выделится при размыкании цепи? Какая средняя ЭДС самоиндукции появится при этом в катушке, если энергия будет выделяться в течение 12 мс
    РЕШЕНИЕ

    За какое время в катушке с индуктивностью 240 мГн происходит возрастание силы тока от 0 до 11,4 А, если при этом возникает средняя ЭДС самоиндукции, равная 30 В? Сколько энергии выделяется за это время в катушке
    РЕШЕНИЕ

    Задачи по физике — это просто!

    Не забываем, что решать задачи надо всегда в системе СИ!

    А теперь к задачам!

    Элементарные задачи из курса школьной физики на вычисление ЭДС индукции.

    Задача 1

    За время 5 мс в соленоиде, содержащем 500 витков провода, магнитный поток равномерно убывает от 7 мВб до 3 мВб.
    Найдите ЭДС индукции в соленоиде.


    Задача 2

    Какой магнитный поток пронизывает каждый виток катушки, имеющей 1000 витков, если при равномерном исчезновении магнитного поля в течение 0,1 с в катушке индуцируется ЭДС равная 10 В ?

    Задача 3

    Виток проводника площадью 2 см 2 расположен перпендикулярно вектору магнитной индукции.
    Чему равна ЭДС индукции в витке, если за время 0,05 секунд магнитная индукция равномерно убывает с 0,5 Тл до 0,1 Тл?


    Задача 4

    В однородном магнитном поле перпендикулярно к направлению вектора индукции , модуль которого 0,1 Тл, движется провод длиной 2 метра со скоростью 5 м/с, перпендикулярной проводнику.
    Какая ЭДС индуцируется в этом проводнике?

    Задача 5

    Перпендикулярно вектору магнитной индукции перемещается проводник длиной 1,8 метра со скоростью 6 м/c. ЭДС индукции равна 1,44 В.
    Найти магнитную индукцию магнитного поля.


    Задача 6

    Самолет имеет размах крыльев 15 метров. Горизонтальная скорость полета равна720 км/час.
    Определить разность потенциалов, возникающих между концами крыльев. Вертикальная составляющая магнитной индукции (перпендикулярно поверхности Земли) равна 50 мкТл.

    Задача 7

    Магнитный поток через контур проводника сопротивлением 0,03 Ом за 2 секунды изменился на 0,012 Вб.
    Найдите силу тока в проводнике если изменение потока происходило равномерно.

    Задача 8

    В однородном магнитном поле находится плоский виток площадью 10 см 2 , расположенный перпендикулярно вектору магнитной индукции.
    Какой ток течет по витку, если поле будет убывать с постоянной скоростью 0,5 Тл/с?

    Задача 9

    Сопротивление замкнутого контура равно 0,5 Ом. При перемещении кольца в магнитном поле магнитный поток через кольцо изменился на 5×10 -3 Вб.
    Какой за это время прошел заряд через поперечное сечение проводника?

    Motional Emf — Университетская физика, том 2

    Цели обучения

    К концу этого раздела вы сможете:

    • Определить величину наведенной ЭДС в проводе, движущемся с постоянной скоростью через магнитное поле
    • Обсудите примеры, использующие двигательную ЭДС, например, рельсовую пушку и привязанный спутник.

    Магнитный поток зависит от трех факторов: силы магнитного поля, площади, через которую проходят силовые линии, и ориентации поля с площадью поверхности.Если какая-либо из этих величин изменяется, происходит соответствующее изменение магнитного потока. До сих пор мы рассматривали только изменения потока из-за изменяющегося поля. Теперь мы рассмотрим другую возможность: изменение области, через которую проходят силовые линии, включая изменение ориентации области.

    Два примера этого типа изменения потока представлены на (Рисунок). В части (а) поток через прямоугольную петлю увеличивается по мере того, как она движется в магнитное поле, а в части (b) поток через вращающуюся катушку изменяется в зависимости от угла.

    (a) Магнитный поток изменяется, когда петля движется в магнитное поле; (б) магнитный поток изменяется при вращении петли в магнитном поле.

    Интересно отметить, что то, что мы воспринимаем как причину определенного изменения потока, на самом деле зависит от выбранной нами системы отсчета. Например, если вы находитесь в состоянии покоя относительно движущихся катушек (Рисунок), вы увидите, что поток изменяется из-за изменения магнитного поля — в части (а) поле перемещается слева направо в вашей системе отсчета, и в части (б) поле вращается.Часто можно описать изменение потока через катушку, которая движется в одной конкретной системе отсчета, в терминах изменяющегося магнитного поля во второй системе отсчета, где катушка неподвижна. Однако вопросы системы отсчета, связанные с магнитным потоком, выходят за рамки этого учебника. Мы избежим таких сложностей, всегда работая в кадре в состоянии покоя относительно лаборатории и объясняя вариации потока как следствие либо изменяющегося поля, либо изменяющейся области.

    Теперь давайте посмотрим на проводящий стержень, включенный в цепь, изменяющую магнитный поток.Площадь, ограниченная схемой «MNOP» (Рисунок), составляет лк и перпендикулярна магнитному полю, поэтому мы можем упростить интеграцию (Рисунок) в умножение магнитного поля и площади. Следовательно, магнитный поток через открытую поверхность составляет

    Поскольку B и l постоянны, а скорость стержня равна, мы можем переформулировать закон Фарадея (рисунок) для величины ЭДС, выраженной в движущемся проводящем стержне, как

    Ток, наведенный в цепи, равен ЭДС, деленной на сопротивление, или

    Кроме того, направление наведенной ЭДС удовлетворяет закону Ленца, что вы можете проверить, посмотрев на рисунок.

    Этот расчет ЭДС, вызванной движением, не ограничивается перемещением стержня по проводящим рельсам. В качестве отправной точки можно показать, что справедливо для любого изменения магнитного потока, вызванного движением проводника. Мы видели в законе Фарадея, что ЭДС, индуцированная изменяющимся во времени магнитным полем, подчиняется той же зависимости, которая является законом Фарадея. Таким образом, закон Фарадея выполняется для всех изменений магнитного потока , независимо от того, вызваны ли они изменяющимся магнитным полем, движением или их комбинацией.

    Проводящий стержень толкается вправо с постоянной скоростью. Результирующее изменение магнитного потока вызывает в цепи ток.

    С точки зрения энергии производит мощность, а резистор ее рассеивает. Поскольку стержень движется с постоянной скоростью, приложенная сила должна уравновешивать магнитную силу на стержне, когда он пропускает наведенный ток I . Таким образом, произведенная мощность составляет

    единиц.

    Рассеиваемая мощность

    В соответствии с принципом сохранения энергии производимая и рассеиваемая мощности равны.

    Этот принцип можно увидеть в работе рельсовой пушки. Рельсовая пушка — это электромагнитная пусковая установка для снарядов, в которой используется устройство, подобное (Рисунок), и схематично показано на (Рисунок). Проводящий стержень заменяется выстрелом или оружием. До сих пор мы слышали только о том, как движение вызывает ЭДС. В рельсовой пушке оптимальное отключение / уменьшение магнитного поля уменьшает поток между рельсами, вызывая протекание тока в стержне (якорь), удерживающем снаряд.Этот ток через якорь испытывает магнитную силу и продвигается вперед. Однако рельсовые пушки не используются широко в вооруженных силах из-за высокой стоимости производства и больших токов: для выработки энергии, достаточной для того, чтобы рельсовая пушка была эффективным оружием, требуется около миллиона ампер.

    Ток через две рельсы движет токопроводящий снаряд вперед за счет создаваемой магнитной силы.

    Мы можем вычислить ЭДС, индуцированную движением, с помощью закона Фарадея , даже когда фактически замкнутый контур отсутствует .Мы просто представляем замкнутую область, граница которой включает движущийся проводник, вычисляем, а затем находим ЭДС по закону Фарадея. Например, мы можем позволить движущемуся стержню (Рисунок) быть одной стороной воображаемой прямоугольной области, представленной пунктирными линиями. Площадь прямоугольника составляет лк , поэтому магнитный поток через него равен. Дифференцируя это уравнение, получаем

    , что соответствует разности потенциалов между концами стержня, которую мы определили ранее.

    С показанным воображаемым прямоугольником мы можем использовать закон Фарадея для расчета наведенной ЭДС в движущемся стержне.

    ЭДС движения в слабом магнитном поле Земли обычно не очень велики, иначе мы заметили бы напряжение на металлических стержнях, таких как отвертка, во время обычных движений. Например, простой расчет ЭДС движения стержня длиной 1,0 м, движущегося со скоростью 3,0 м / с перпендикулярно полю Земли, дает

    Это небольшое значение согласуется с опытом.Однако есть впечатляющее исключение. В 1992 и 1996 годах с космическим челноком были предприняты попытки создать большие двигательные ЭДС. Привязанный спутник должен был быть выпущен на проводе длиной 20 км, как показано на (Рисунок), для создания ЭДС 5 кВ за счет движения с орбитальной скоростью через поле Земли. Эту ЭДС можно было бы использовать для преобразования некоторой кинетической и потенциальной энергии шаттла в электрическую, если бы можно было создать полную схему. Чтобы замкнуть цепь, неподвижная ионосфера должна была обеспечить обратный путь, по которому мог течь ток.(Ионосфера — это разреженная и частично ионизированная атмосфера на орбитальных высотах. Она проводит из-за ионизации. Ионосфера выполняет ту же функцию, что и стационарные рельсы и соединительный резистор на (Рисунок), без которых не было бы полной цепи.) Затягивание тока в кабеле из-за магнитной силы выполняет работу, которая уменьшает кинетическую и потенциальную энергию шаттла и позволяет преобразовывать ее в электрическую. Оба теста не увенчались успехом. В первом случае кабель завис, и его можно было протянуть только на пару сотен метров; во втором трос оборвался при почти полном растяжении.(Рисунок) указывает на выполнимость в принципе.

    ЭДС движения как преобразование электроэнергии для космического корабля многоразового использования была мотивацией для эксперимента с привязанным спутником. Было предсказано, что ЭДС 5 кВ будет индуцироваться в 20-километровом тросе при движении с орбитальной скоростью в магнитном поле Земли. Цепь замыкается обратным трактом через неподвижную ионосферу.

    Металлический стержень, вращающийся в магнитном поле На части (a) (Рисунок) показан металлический стержень OS , который вращается в горизонтальной плоскости вокруг точки O .Стержень скользит по проволоке, которая образует дугу окружности PST радиуса r . Система находится в постоянном магнитном поле, направленном за пределы страницы. (а) Если вы вращаете стержень с постоянной угловой скоростью, каков ток I в замкнутом контуре OPSO ? Предположим, что резистор R обеспечивает все сопротивление в замкнутом контуре. (b) Рассчитайте работу за единицу времени, которую вы делаете при вращении стержня, и покажите, что она равна мощности, рассеиваемой в резисторе.

    Стратегия Магнитный поток — это магнитное поле, умноженное на площадь четверти круга. умножить на угловую скорость. Крутящий момент рассчитывается исходя из силы, действующей на стержень, и ее интегрирования по длине стержня.

    Решение

    1. Исходя из геометрии, площадь контура OPSO равна Следовательно, магнитный поток через контур равен


      Дифференцируя по времени и использованию, получаем


      При делении на сопротивление контура R получается величина индуцированного тока


      По мере увеличения увеличивается и поток через петлю. Чтобы противодействовать этому увеличению, магнитное поле из-за индуцированного тока должно быть направлено на страницу в области, ограниченной петлей.Следовательно, как показано в части (b) (Рисунок), ток циркулирует по часовой стрелке.

    2. Вы вращаете стержень, прилагая к нему крутящий момент. Поскольку стержень вращается с постоянной угловой скоростью, этот крутящий момент равен и противоположен крутящему моменту, приложенному к току в стержне исходным магнитным полем. Магнитная сила на бесконечно малом сегменте длиной dx , показанном в части (c) (Рисунок), такова, что магнитный момент на этом сегменте равен
      .


      Чистый магнитный крутящий момент на стержне равен


      Крутящий момент, который вы прикладываете к стержню, равен и противоположен ему, а работа, которую вы выполняете, когда стержень поворачивается на угол, равна Следовательно, работа на единицу времени, которую вы выполняете на стержне, равна


      , где мы заменили I .Мощность, рассеиваемая в резисторе, может быть записана как


      Следовательно, мы видим, что


      Следовательно, мощность, рассеиваемая в резисторе, равна работе в единицу времени, совершаемой при вращении стержня.

    Значение. Альтернативный способ взглянуть на индуцированную ЭДС из закона Фарадея — интегрировать в пространстве, а не во времени. Решение, однако, будет таким же. Двигательная ЭДС

    Скорость может быть записана как угловая скорость, умноженная на радиус, а дифференциальная длина — как dr .Следовательно,

    , это то же самое решение, что и раньше.

    Проверьте свое понимание Стержень длиной 10 см движется со скоростью 10 м / с перпендикулярно через магнитное поле напряжением 1,5 Тл. Какая разница потенциалов между концами стержня?

    Сводка

    • Взаимосвязь между наведенной ЭДС в проводе, движущемся с постоянной скоростью v через магнитное поле B , определяется как
    • Индуцированная ЭДС по закону Фарадея создается ЭДС движения, которая противодействует изменению потока.

    Концептуальные вопросы

    Стержневой магнит падает под действием силы тяжести вдоль оси длинной медной трубки. Если сопротивление воздуха незначительно, появится ли сила, препятствующая спуску магнита? Если да, достигнет ли магнит предельной скорости?

    Вокруг географического Северного полюса (или южного магнитного полюса) магнитное поле Земли почти вертикальное. Если в этой области самолет летит на север, какая сторона крыла заряжена положительно, а какая отрицательно?

    Положительные заряды на крыльях будут к западу или слева от пилота, а отрицательные заряды будут тянуться к востоку или справа от пилота.Таким образом, кончики левых крыльев будут положительными, а кончики правых — отрицательными.

    Проволочная петля движется поступательно (без вращения) в однородном магнитном поле. В шлейфе наведена ЭДС?

    Глоссарий

    двигательная ЭДС
    напряжение, создаваемое движением проводящего провода в магнитном поле

    Электромагнетизм — Проблема наведенной ЭДС

    Рассмотрим простейший случай однородного магнитного поля и круглого провода, перпендикулярного магнитному полю.Вы знаете, что изменяющееся магнитное поле индуцирует электрическое поле. Поскольку магнитное поле однородно, а провод перпендикулярен, электрическое поле имеет одинаковую величину во всех точках провода. Вы можете найти полную ЭДС по изменению потока. Но помните определение ЭДС для замкнутого контура. $$ emf = \ oint_C \ vec {E} \ cdot dl $$ Это также равно $ — \ frac {d \ Phi_B} {dt} $ для того же замкнутого цикла. Теперь предположим, что вы рассчитали по магнитному потоку, что наведенная ЭДС составляет 6 В. Это также означает, что если взять линейный интеграл поля $ \ vec {E} $ вокруг всего замкнутого контура, получится 6V.Теперь вместо того, чтобы обходить весь цикл, объедините примерно половину цикла. Наше поле $ \ vec {E} $ имеет одинаковую величину во всех точках, поэтому мы получаем половину ЭДС, 3В. Если мы возьмем две точки, которые составляют всего 10% от общей окружности петли, то между этими двумя точками будет ЭДС 0,6 В.

    Другими словами, в отличие от батареи, в которой ЭДС ограничена одной точкой в ​​цепи, в случае наведенной ЭДС существует усиление напряжения в любых двух точках цепи, где $ \ int_A ^ B \ vec {E } \ cdot dl $ отличен от нуля.B \ vec {E} \ cdot dl $ и от резисторов. Вы знаете, что наведенная ЭДС равна падению напряжения на резисторах. Таким образом, если ЭДС составляет 6 В, напряжение на обоих резисторах составляет -6 В. Итак, давайте снова воспользуемся простейшим примером, где поле имеет одинаковую величину в каждой точке. Выберите две точки так, чтобы между ними было два резистора и 25% провода. В части петли без резисторов и на 75% проволочной петли вы увидите усиление 4,5 В. На участке с 25% провода и обоими резисторами вы увидите 1.5 В — 6 В = усиление -4,5 В.

    Абсолютное напряжение в любой точке было бы трудно вычислить без точки отсчета. Вам, по крайней мере, нужно знать, был ли провод изначально на 0 В или какой-то другой $ V_0 $, и тогда, я думаю, вы сможете его вычислить. Но обычно в таких случаях вы заземляете часть цепи.

    Часть 6: Электромагнитная индукция | ITACA

    Мы видели, что всякий раз, когда электрический ток течет через проводник, создается магнитное поле (раздел 4.3). Обратное также может происходить, и при определенных условиях магнитное поле может быть ответственным за протекание электрического тока. Это явление может происходить двумя способами:

    1. Dynamic Induction — проводник движется в магнитном поле.
    2. Статическая индукция — магнитное поле вокруг проводника изменяется.

    В обоих случаях магнитное поле изменяется относительно проводника.

    6.1 Динамическая индукция

    Если концы металлической проволоки подсоединены к чувствительному индикаторному прибору, стрелка будет подпрыгивать, когда проволока внезапно проходит через полюса магнита (рисунок 6.1а). Движение иглы происходит из-за ЭДС, индуцируемой в проводе, когда он движется через магнитное поле, эта ЭДС вызывает протекание тока, если провод образует замкнутую цепь. Когда движение проволоки прекращается, стрелка возвращается к нулю, поскольку ЭДС больше не индуцируется (рисунок 6.1b). Если проволоку протянуть обратно через магнитное поле, будет видно, что потребности прыгают в противоположном направлении (рисунок 6.1c).

    Рис. 6.1: (а) движущаяся проволока, наводящая ЭДС; б) когда провод неподвижен, ЭДС прекращается; (c) если направление движения провода меняется, то меняется и направление наведенной ЭДС.

    Можно обнаружить, что величина наведенной ЭДС пропорциональна силе магнита, длине проводника в магнитном поле и скорости провода, таким образом;

    где:
    e = мгновенная наведенная ЭДС (В)
    B = плотность потока магнитного файла (Т)
    l = длина проводника в поле (м)
    v = скорость проводника ( м / с) (обратите внимание, что скорость — это скорость в заданном направлении).

    Следовательно, e будет постоянным значением, если проводник движется с постоянной скоростью через однородное поле.Однако в неоднородном поле или если скорость проводника непостоянна, наведенная ЭДС будет изменяться со временем, и e используется для обозначения мгновенной ЭДС, то есть ЭДС в данный момент.

    Индуцированная ЭДС также может быть определена в терминах скорости, с которой магнитные линии потока «перерезаются» проводом. Если проводник отсекает один поток потока за одну секунду, наведенная ЭДС будет составлять один вольт. Таким образом:

    где:
    Φ = общий магнитный поток «отсечка» (Вт)
    t = время, (с)

    Направление наведенной ЭДС (очевидно, что направление тока в замкнутой цепи зависит от направления наведенной ЭДС) зависит от направления движения и направления магнитного поля (помните, что направление поля принято с севера на юг).Направление ЭДС, движения или поля можно найти с помощью правила для правой руки (генератора) Флеминга (рисунок 6.2).

    Рисунок 6.2: Правило правой руки Флеминга. Убедитесь, что ток отображается в правильном направлении для указанного движения и поля на рисунке 6.1.

    6.2 Простой генератор

    Рассмотрим петлю из проволоки, вращающуюся между полюсами магнита, как показано на рисунке 9a. Соединения с концами проволоки выполняются с помощью щеток и контактных колец, так что петля может свободно вращаться.На рис. 6.3b показан вид вниз сечения петли AB в различных положениях, когда петля вращается в магнитном поле. Когда AB находится в любом положении 1 и 5, он движется по линиям потока, но не разрезает их, поэтому ЭДС не индуцируется. В положениях 2, 4, 6 и 8 AB срезает силовые линии под углом, так что возникает ЭДС. В положениях 3 и 7 проводник движется прямо поперек силовых линий, разрезая его с максимальной скоростью, и индуцируется максимальная ЭДС. Правило правой руки Флеминга можно использовать для определения направления наведенной ЭДС и, следовательно, направления, в котором течет ток (рисунок 6.3б). Обратите внимание, что направление наведенной ЭДС (и тока), когда катушка движется слева направо через поле, противоположно направлению ЭДС, индуцированной при движении катушки справа налево. Величина и направление наведенной ЭДС показаны в зависимости от времени на рисунке 6.3c, это переменная ЭДС, которая попеременно действует в разных направлениях.

    Рис. 6.3: (а) проводящая петля, вращающаяся в магнитном поле; (b) ток, протекающий в секции AB, когда он вращается через поле, крестик указывает, что ток течет на страницу, а точка указывает, что ток течет со страницы; (c) наведенная ЭДС в зависимости от положения петли.

    Обратите внимание, что ток, текущий в стороне петли AB в любой точке цикла, будет иметь направление, противоположное току, текущему в стороне CD. Следовательно, ток будет течь вниз по одной стороне петли и подниматься вверх по другой, уменьшаясь до нуля, когда петля находится в горизонтальном положении, прежде чем течь обратно по петле в противоположном направлении.

    6.3 Статическая индикация

    Если катушка с проволокой остается неподвижной, внутри нее все еще может быть наведена ЭДС, изменяя проходящее через нее магнитное поле.Следовательно, линии магнитного потока «разрезают» проводник, а не проводник «разрезают» линии магнитного потока. Когда происходит изменение магнитного потока, это часто называют изменением «потокосцепления», указывающим на то, что произошло изменение величины магнитного потока, «связанного» с проводником или катушкой или проходящего через них.

    Если к катушке присоединен детектор, это будет означать, что ЭДС была наведена, когда внешнее магнитное поле увеличивается или уменьшается через катушку (рисунок 6.4б). Когда поле выключено или остается постоянным, индуктор покажет, что ЭДС не индуцируется (рисунок 6.4a). Величина наведенной ЭДС зависит от скорости изменения магнитного потока, а не от величины изменения магнитного потока: небольшое изменение магнитного потока может вызвать большую ЭДС, чем большое изменение потока, происходящее медленно.

    Рис. 6.4: (а) катушка, через которую не проходит поток; (б) изменяющийся магнитный поток, создающий наведенную ЭДС в катушке.

    ЭДС в один вольт будет индуцироваться в катушке потоком, изменяющимся со скоростью один Вебер в секунду. Отсюда:

    где:
    e = средняя наведенная ЭДС (В)
    Φ 1 = конечное значение магнитного потока (Wb)
    Φ 2 = начальное значение магнитного потока (Wb)
    t = время, затраченное на поток изменится с Φ 1 на Φ 2 (с).
    N = количество витков на катушке.

    Чтобы избежать вычислений, это уравнение предполагает, что изменение потока происходит с постоянной скоростью или, по крайней мере, любые колебания в изменении игнорируются, следовательно, e — это средняя ЭДС, индуцированная за период изменения потока.

    Вышеприведенное уравнение предназначено для катушки, реагирующей на внешнее магнитное поле. Если катушка не находится во внешнем магнитном поле, а подключена через переключатель к источнику постоянного тока, она будет создавать собственное магнитное поле, поскольку будет действовать как соленоид (раздел 4.4). Как только переключатель замкнут, катушка начнет создавать магнитное поле. Магнитный поток будет проходить через саму катушку, и, поскольку этот поток увеличивается в момент нажатия переключателя, он вызывает ЭДС внутри катушки.Если бы эта наведенная ЭДС имела то же направление, что и источник питания, ток бесконтрольно увеличивался бы до тех пор, пока катушка не расплавилась. Следовательно, в этом случае наведенная ЭДС должна действовать против приложенной ЭДС, поскольку мы знаем, что катушки не плавятся самопроизвольно. Это явление известно как закон Ленца:

    Направление индуцированной ЭДС всегда таково, чтобы противодействовать вызывающему ее эффекту.

    Таким образом, ЭДС, вызванная изменением магнитного потока, в свою очередь, вызванная увеличением тока, будет противодействовать току и пытаться предотвратить его протекание.Точно так же ЭДС, индуцированная уменьшающимся током, будет пытаться поддерживать ток. Индуцированная ЭДС не может остановить изменение тока, поскольку ток в конечном итоге достигнет постоянного значения, и магнитный поток также станет постоянным, однако он замедляет его до тех пор, пока нет других изменений.

    6.4 Взаимная индуктивность

    Взаимная индукция возникает между двумя катушками, одна из которых создает магнитный поток по мере протекания через нее тока, а другая имеет ЭДС, индуцированную этим потоком в ней.

    Рассмотрим две катушки, расположенные рядом друг с другом (рисунок 6.5a). Левая катушка подключена к источнику постоянного тока, а правая катушка подключена к детектору центрального нуля. Пока ток не течет в левой катушке, это не повлияет на правую катушку, и датчик остается на нуле (рисунок 6.5a). Когда переключатель замкнут, ток начинает течь через левую катушку, однако он не достигает постоянного значения сразу, а постепенно увеличивается из-за закона Ленца.В то время как ток увеличивается в левой катушке, магнитный поток, создаваемый ею, также увеличивается, часть этого потока проходит через правую катушку. Поскольку этот поток увеличивается, он вызывает ЭДС в правой катушке, и стрелка детектора перемещается от нуля (рисунок 6.5b).

    Как только ток в левой катушке достигнет постоянного значения, поток, который он производит, также становится постоянным, и, следовательно, поток, проходящий через правую катушку, больше не изменяется. Следовательно, в правой катушке не возникает ЭДС, и детектор возвращается к нулю (рисунок 6.5в).

    Если переключатель разомкнут, ток в левой катушке уменьшится до нуля, но из-за закона Ленца ток будет постепенно уменьшаться, таким образом, поток, проходящий через правую катушку, также уменьшается. Этот изменяющийся поток индуцирует ЭДС в правой катушке, и детектор перемещается от нуля (рисунок 6.5d). Обратите внимание, что отклонение стрелки детектора будет в направлении, противоположном тому, которое произошло при увеличении потока.

    Как только ток в правой катушке достигает нуля, катушка больше не создает магнитный поток, и поэтому поток не проходит через правую катушку, и детектор возвращается к нулю.

    Рисунок 6.5: Взаимная индукция между двумя катушками. См текст для деталей.

    Это явление, при котором ЭДС индуцируется в одной катушке при изменении тока в другой катушке, называется взаимной индукцией и считается, что между катушками существует взаимная индукция (M). Обратите внимание, что индукция происходит по мере увеличения или уменьшения тока, но между двумя катушками существует определенная индуктивность, даже если ток постоянный. Единицей взаимной индуктивности является генри (Гн), и ее можно определить двумя способами —

    .

    (a) Взаимная индуктивность между двумя катушками составляет один генри, когда ток, изменяющийся в первой катушке со скоростью один ампер в секунду, индуцирует ЭДС в один вольт во второй катушке.Таким образом:

    где:

    e = средняя ЭДС, индуцированная во второй катушке (В)
    M = взаимная индуктивность между катушками (H)
    I1 = начальный ток в первом токе (A)
    I2 = конечный ток в конечном токе (A)
    t = время изменения (с)

    Обратите внимание, что снова e — это средняя ЭДС, индуцированная, чтобы избежать образования камней.
    (b) Взаимная индуктивность между двумя катушками равна одному генри, когда ток в один ампер в первой катушке создает магнитосцепление в один виток Вебера во второй катушке.Виток Вебера — это поток, проходящий через катушку, умноженный на количество витков в катушке. Таким образом:

    где:
    M = взаимная индуктивность между катушками (H)
    Φ = величина магнитного потока, создаваемого первой катушкой, которая проходит через вторую (Wb)
    N2 = количество витков во второй катушке
    I1 = ток на второй катушке (А)

    На практике на взаимную индуктивность катушек влияет ряд факторов. Всего:

    1. количество витков в катушках — чем больше витков, тем выше взаимная индуктивность;
      расстояние между катушками — чем больше расстояние, тем меньше взаимная индуктивность;
    2. положение одной катушки относительно другой — если катушки расположены под прямым углом, не большая часть магнитного потока, создаваемого первой катушкой, будет проходить через вторую, и взаимная индуктивность будет низкой.
    3. наличие магнитной цепи — если обе катушки намотаны на железный сердечник, это не только увеличивает создаваемый магнитный поток, но и направляет почти весь его через вторую катушку, значительно увеличивая взаимную индуктивность.

    Обозначения, используемые для взаимных индукторов, показаны на рисунке 6.6.

    Рисунок 6.6: Обозначения для (a) взаимной индуктивности с воздушным сердечником и (b) взаимной индуктивности с металлическим сердечником.

    Пример

    Две катушки с воздушным сердечником имеют взаимную индуктивность 0.4H, и одна катушка на 1200 витков имеет потокосцепление 5nWb, когда в другой протекает постоянный ток, вычислите значение установившегося тока.

    Когда ток в системе катушек увеличивается до 25 А, в катушке на 1200 витков индуцируется средняя ЭДС 80 В. Рассчитайте время, необходимое для изменения силы тока.

    Для индукторов с железным сердечником величина установленного магнитного потока не будет прямо пропорциональна току намагничивания из-за явления, называемого насыщением. Об этом будет рассказано позже.

    6.5 Собственная индуктивность

    Когда мы рассматривали взаимную индукцию, мы увидели, что изменение тока, протекающего в одной катушке, создает изменяющийся магнитный поток, который индуцирует ЭДС во второй катушке. Однако первая катушка также испытывает изменяющийся магнитный поток, потому что поток проходит через обе катушки (рисунки 11b, c и d), поэтому ЭДС должна быть индуцирована и в первой катушке.

    Индукция ЭДС в изолированной катушке из-за изменения тока внутри нее, которое приводит к изменению магнитной индукции, называется самоиндукцией (L).Единицей самоиндукции, как и взаимной индуктивности, является генри (H). Самостоятельную индуктивность можно определить двумя способами —

    (a) Собственная индуктивность катушки равна одному генри, когда скорость изменения тока в катушке один ампер в секунду индуцирует в ней ЭДС в один вольт. Таким образом:

    где:
    e = средняя наведенная ЭДС (В)
    L = собственная индуктивность системы (H)
    I 1 = начальный ток в катушке (A)
    I 2 = конечный ток в катушке (A)
    t = время, необходимое для тока от I 1 до I 2 , (с)

    (b) Собственная индуктивность катушки равна одному генри, если ток в один ампер в катушке создает в ней магнитосцепление в один виток Вебера.Виток Вебера — это поток, проходящий через катушку, умноженный на количество витков в катушке. Таким образом:

    где:
    L = собственная индуктивность катушки (H)
    Φ = магнитная индукция катушки (Wb)
    N = количество витков
    I = ток, переносимый системой (A)

    Факторы, влияющие на самоиндукцию проводника:

    1. количество витков — чем больше витков, тем выше собственная индуктивность;
    2. расположение витков — короткая толстая катушка будет иметь большую собственную индуктивность, чем длинная тонкая;
    3. наличие магнитной цепи — если катушка намотана на железный сердечник, тот же ток создаст больший магнитный поток и собственная индуктивность будет выше.

    Рисунок 6.7: Обозначения для самоиндукторов: (а) с железным сердечником; (б) порошковая

    Обозначение индуктора показано на рисунке 6.7.

    Пример

    Катушка

    A имеет собственную индуктивность 3H и сопротивление 8 Ом. Рассчитайте ток, когда катушка подключена к источнику постоянного тока 12 В. Питание отключается, и ток падает до нуля за 0,2 с, рассчитайте среднюю наведенную ЭДС.


    Отрицательный знак означает, что действует закон Ленца и индуцированная ЭДС будет противодействовать вызывающему ее изменению.В этом случае связующий поток уменьшается, потому что уменьшается ток, поэтому наведенная ЭДС пытается сохранить ток. Чем больше собственная индуктивность катушки, тем больше времени потребуется, чтобы ток снизился до нуля. Обратите внимание, что собственная индуктивность не влияет на установившийся ток, протекающий в системе постоянного тока, и I = U / R все еще можно использовать для определения установившегося тока (то есть, когда ток является установившимся и не меняется, катушка действует как плоский резистор) . Поскольку ЭДС индуктивности противодействует любым изменениям тока, она называется обратной ЭДС . Он всегда будет противостоять переменам, но никогда не помешает им. Обратные ЭДС вызывают медленные изменения тока в системах постоянного тока с высокой индуктивностью. Катушки с железным сердечником и высокой самоиндукцией называют дросселями, потому что они сглаживают и подавляют колебания тока.

    6.6 Энергия, запасенная в магнитном поле

    Идеальный индуктор — это катушка с проводом, у которой есть индуктивность, но нет сопротивления. На практике это невозможно, так как катушки индуктивности сделаны из спиральной проволоки, а провода имеют сопротивление. Однако игнорирование сопротивления может помочь нам изучить изменения энергии, происходящие в индуктивности.Для создания магнитного поля требуется энергия, и эта энергия не теряется в виде тепла, а сохраняется в магнитном поле. Когда ток, вызывающий магнитное поле, отключается, поле коллапсирует, и накопленная энергия выделяется в цепь в виде наведенной обратной ЭДС.

    Энергия, запасенная в магнитном поле, создаваемом током, протекающим в катушке, определяется выражением:

    где:
    W = энергия, запасенная в магнитном поле (Дж)
    L = собственная индуктивность (H)
    I = конечный ток, протекающий через катушку (A)

    На практике индуктор будет состоять из катушки с проволокой, имеющей сопротивление.Это сопротивление может быть показано отдельно от индуктивности, как показано на рисунке 6.8.

    Рисунок 6.8: Сопротивление и индуктивность катушки.

    6.7 Кривые роста и спада

    Для схемы, показанной на рисунке 6.8, как только переключатель замыкается, через катушку начинает течь постоянный ток. Начальная скорость нарастания тока будет высокой, что приведет к большой наведенной ЭДС. Согласно закону Ленца, эта ЭДС будет противодействовать току, и скорость увеличения тока будет снижаться до тех пор, пока не будет достигнут установившийся ток, с этого момента катушка действует как резистор.Следовательно, увеличение тока в зависимости от времени (рисунок 6.9) будет кривой, которая выравнивается при значении I = ER; где E — напряжение питания, R — сопротивление катушки.

    Рисунок 6.9: График мгновенного значения тока (i) в зависимости от времени (t) для катушки индуктивности с сопротивлением (R) при подключении к источнику постоянного тока (с напряжением E). Сила тока стабилизируется (I).

    Рисунок 6.10: График мгновенного значения тока (i) в зависимости от времени (t) для катушки индуктивности с сопротивлением (R) при отключении от источника постоянного тока (с напряжением E).Сила тока стабилизируется (I). τ — постоянная времени.

    Когда переключатель на рисунке 6.9 размыкается, возникает затухающий ток, сначала быстрый, а затем более медленный, поскольку наведенная ЭДС препятствует изменению. График зависимости мгновенного тока от времени (рисунок 6.10) обратен предыдущей кривой.

    Отношение L / R называется постоянной времени (τ), которая может быть определена как время, за которое напряжение или ток достигнет своего конечного значения, если оно продолжит изменяться с начальной скоростью и:

    При включении источника питания мгновенный ток (т.е.е. ток в любой момент времени, т ) определяется по:

    При отключении источника питания мгновенный ток равен:

    где:
    E = ЭДС питания (В)
    R = сопротивление катушки (Ом)
    t = время (с)

    Два предыдущих уравнения включены для полноты картины, но не беспокойтесь о математике, если вы не знакомы с экспоненциальными функциями.

    6.8 Переключение индуктивных цепей

    Когда источник питания, подключенный к индуктивности, переключается, ток, поддерживающий магнитное поле, прекращается, и магнитный поток исчезает.Энергия в магнитном поле больше не может накапливаться и должна быть возвращена в цепь в виде тока, вызванного наведенной ЭДС. Таким образом, если источник питания просто выключен, внезапное высвобождение энергии может вызвать большую наведенную ЭДС, которая может вызвать искру на переключателе или расплавить изоляцию на проводе катушки. Это означает, что схема, показанная на рисунке 6.8, может быть довольно опасной. Чтобы избежать повреждений, схемы спроектированы таким образом, чтобы наведенная ЭДС могла управлять током в цепи разряда.На рисунке 6.11 показаны три возможных метода, которые можно использовать для рассеивания энергии, накопленной в магнитном поле индуктора.

    На рисунке 6.11a, когда переключатель разомкнут, наведенная ЭДС пропускает ток через R D , а не через переключатель. Однако значение R D должно быть достаточно высоким, чтобы ток не протекал через него, когда переключатель замкнут, но достаточно низким, чтобы при разомкнутом переключателе через него протекал значительный ток, а не возникала дуга через переключатель.

    На рисунке 6.11b показана улучшенная схема, содержащая чувствительный к напряжению резистор. При нормальном напряжении питания эти резисторы имеют очень высокое сопротивление, и через них будет протекать небольшой ток. Однако, когда переключатель разомкнут, наведенная ЭДС вызывает падение сопротивления специального резистора и обеспечивает путь разряда для тока.

    Третий вариант, показанный на рисунке 6.11c, заключается в установке диода вместо разрядного резистора. Диоды — это полупроводниковые устройства, которые имеют очень низкое сопротивление токам, текущим в направлении стрелки и блокирующим ток, протекающий в другом направлении.Следовательно, при замкнутом переключателе ток не может проходить через разрядную цепь. Когда переключатель разомкнут, индуцированная ЭДС проталкивает ток по цепи в противоположном направлении, а диод позволяет ему течь через разрядную цепь.

    Рисунок 6.11: Цепи разряда (а) нормальный резистор; (б) резистор, чувствительный к напряжению; (c) диод.

    Базовая электротехника

    Как рассчитать наведенную ЭДС катушки? — Мворганизация.org

    Как рассчитать наведенную ЭДС катушки?

    Расчет наведенной ЭДС Закон Фарадея гласит: Индуцированная ЭДС равна скорости изменения магнитного потока. Магнитный поток = Напряженность магнитного поля x Площадь = BA. Следовательно… Индуцированная ЭДС = (изменение плотности магнитного потока x площадь) / изменение во времени.

    Что такое индуцированная ЭДС в физике?

    Его можно определить как возникновение разности потенциалов в катушке из-за изменений проходящего через нее магнитного потока.Проще говоря, электродвижущая сила или ЭДС, как говорят, индуцируются, когда поток, связывающийся с проводником или катушкой, изменяется.

    Как найти наведенную ЭДС в петле?

    Рассчитайте производную потока по времени dΦB / dt. Величина наведенной ЭДС, | εind | равно | dΦB / dt |. Если вам нужно найти ток, вы часто можете найти его, используя закон Ома, I = ε / R, где R — сопротивление провода.

    Как рассчитать максимальную наведенную ЭДС?

    эдс = 2Bℓw2ωsinωt = (ℓw) Bωsinωt эдс = 2 B ℓ w 2 ω sin ⁡ ω t = (ℓ w) B ω sin ⁡.- максимальная (пиковая) ЭДС. Обратите внимание, что частота колебаний f = ω / 2π, а период T = 1 / f = 2π / ω.

    Какой пример для динамически индуцированной ЭДС?

    ДИНАМИЧЕСКИ ИНДУЦИРОВАННАЯ ЭДС Таким образом, следуя одному из двух процессов, проводник пересекает магнитное поле, и в катушке индуцируется ЭДС. Это явление имеет место в электрогенераторах и противо-ЭДС двигателей, а также в трансформаторах.

    Что такое наведенная ЭДС в цепи?

    Чтобы быть полностью точным, если магнитный поток, проходящий через катушку, изменится, будет произведено напряжение.Это напряжение известно как наведенная ЭДС. Магнитный поток — это мера количества силовых линий магнитного поля, проходящих через область.

    Какова основная причина наведенной ЭДС?

    Самая основная причина наведенной ЭДС — изменение магнитного потока. Размещение катушки с током, которая постоянно движется в стабильном и статическом магнитном поле. Это вызовет изменение вектора площади и, следовательно, будет сгенерирована ЭДС.

    Что такое самоиндуцированная ЭДС?

    Определение: Самоиндуцированная ЭДС — это эл.m.f индуцируется в катушке из-за изменения магнитного потока, возникающего при соединении ее с ее собственными витками. Это явление самоиндуцированной ЭДС можно дополнительно понять на следующем примере, приведенном ниже: Рассмотрим катушку с числом витков N, как показано на рисунке выше.

    Может у вас отрицательная ЭДС?

    Напряжение всегда не отрицательное. Отрицательный знак в законе Фарадея (закон Ленца) не означает, что ЭДС (или ток) всегда указывает в каком-то «отрицательном» направлении. Это означает, что ток всегда течет так, чтобы противодействовать изменению потока, что хорошо показано в этом видеоклипе.

    Что такое двигательная ЭДС?

    ЭДС, индуцированная движением относительно магнитного поля B, называется ЭДС движения и определяется выражением. emf = Bℓv (B, ℓ и v перпендикулярно), где ℓ — длина объекта, движущегося со скоростью v относительно поля.

    Как сделать ЭДС?

    ЭДС индуцируется в катушке или проводнике всякий раз, когда происходит изменение магнитных связей. В зависимости от того, каким образом происходят изменения, существует два типа: когда проводник перемещается в стационарном магнитном поле, чтобы вызвать изменение магнитной связи, ЭДС индуцируется статически.

    Как определить полярность наведенной ЭДС?

    Когда ЭДС генерируется изменением магнитного потока в соответствии с законом Фарадея, полярность наведенной эдс задается законом Ленца.

    Можно ли навести ЭДС в разомкнутой цепи?

    Да, ЭДС может существовать без тока. Рассмотрим закон Ома: V = IR; V = Индуцированная ЭДС Для разомкнутой цепи, R-> бесконечность Следовательно, I = V / бесконечность = 0 даже для некоторого ненулевого значения V. Это то же самое, что и изолированная ячейка. ЭДС будет индуцироваться в проводе, только если он проходит через изменяющееся магнитное поле.

    Какая полярность наведенной ЭДС?

    Иногда бывает сложно определить полярность наведенной ЭДС. Закон Ленца гласит, что индуцированная ЭДС, возникающая в результате изменения магнитного потока, имеет полярность, которая приводит к индуцированному току, направление которого таково, что индуцированное магнитное поле противодействует первоначальному изменению потока.

    Что подразумевается под полярностью наведенной ЭДС?

    Когда ЭДС генерируется изменением магнитного потока в соответствии с законом Фарадея, полярность наведенной ЭДС такова, что она создает ток, магнитное поле которого противодействует изменению, которое его вызывает.Если оно уменьшается, индуцированное поле действует в направлении приложенного поля, пытаясь сохранить его постоянным.

    Что такое собственная индуктивность L?

    В электрической цепи, когда ЭДС индуцируется в той же цепи, в которой изменяется ток, этот эффект называется самоиндукцией (L), но иногда его обычно называют обратной ЭДС, поскольку ее полярность противоположна приложенное напряжение.

    Какой закон определяет направление наведенной ЭДС?

    Закон Ленца используется для определения направления индуцированного тока.Закон электромагнитной индукции Ленца гласит, что направление индуцированного тока в данном магнитном поле таково, что он противодействует индуцированному изменению путем изменения магнитного поля.

    Что подразумевается под полярностью?

    1: качество или состояние, присущее телу, которое проявляет противоположные свойства или силы в противоположных частях или направлениях или которое проявляет противоположные свойства или силы в противоположных частях или направлениях: состояние наличия полюсов.

    Что такое полярное и неполярное?

    Полярные молекулы возникают, когда между связанными атомами существует разность электроотрицательности.Неполярные молекулы возникают, когда электроны делятся равными между атомами двухатомной молекулы или когда полярные связи в более крупной молекуле нейтрализуют друг друга.

    Какие два типа полярности?

    1,7 Полярность. Полярность относится к электрическим условиям, определяющим направление тока относительно электрода. Полярность электродов бывает двух типов: (1) прямая полярность и (2) обратная полярность.

    Что такое полярное и неполярное соединение?

    Полярная ковалентная связь — это тип ковалентной связи, образованной между двумя неидентичными атомами.Неполярная ковалентная связь — это ковалентная связь, в которой разница в электроотрицательности между связанными атомами составляет менее 0,5.

    Как узнать, полярная или неполярная молекула?

    1. Если расположение симметрично и стрелки одинаковой длины, молекула неполярна.
    2. Если стрелки имеют разную длину и не уравновешивают друг друга, молекула полярна.
    3. Если расположение асимметрично, молекула полярна.

    Как определить полярность соединения или неполярность?

    Взгляните еще раз на воду. Вода связана с двумя атомами водорода, а также имеет два неподеленных электрона. Имеет четырехгранную изогнутую форму. Чтобы определить, полярна ли молекула, вы должны посмотреть на векторы парциальных зарядов на двух связях в молекуле.

    Что такое неполярное соединение?

    соединение, состоящее из молекул, обладающих симметричным распределением заряда, так что не существует положительных или отрицательных полюсов и которые не ионизируются в растворе, например, в углеводородах.

    В чем разница между полярной и неполярной ковалентной связью?

    Частично ионные связи называются полярными ковалентными связями. Неполярные ковалентные связи с равным разделением электронов связи возникают, когда электроотрицательности двух атомов равны.

    CO2 полярный или неполярный?

    И CO2, и h3O имеют две полярные связи. Однако диполи в линейной молекуле CO2 нейтрализуют друг друга, что означает, что молекула CO2 неполярна.

    Почему CO полярный, а CO2 неполярный?

    Он содержит две полярные связи, расположенные симметрично.Углерод образует двойную связь с каждым атомом кислорода. Но геометрия CO2 линейна, так что два дипольных момента связи сокращаются, и нет никакого чистого молекулярного дипольного момента. Таким образом, молекула неполярна.

    Какой тип молекулы CO2?

    ковалентный

    Является ли CCl4 полярной или неполярной молекулой?

    CCl4, который представляет собой четыреххлористый углерод, неполярен, потому что все четыре связи симметричны и вытянуты во всех направлениях.

    Почему CCl4 неполярная молекула?

    Молекула CCl4 неполярна по природе из-за ее симметричной тетраэдрической структуры.Однако связь C-Cl является полярной ковалентной связью, но четыре связи отменяют полярность друг друга и образуют неполярную молекулу CCl4.

    [PDF] Глава 27 Электромагнитная индукция

    1 Нам, которые восприняли идеи Фарадея, так сказать, с молоком матери, трудно оценить их величие и …

    Глава 27 Электромагнитная индукция «Нам, которые восприняли идеи Фарадея, так сказать, с молоком матери, трудно оценить их величие и смелость.”Альберт Эйнштейн

    27.1 Введение

    Поскольку ток в проводе создает магнитное поле, логично спросить, возможен ли обратный процесс. То есть есть ли способ, которым магнитное поле может производить ток? Ответ на вопрос — да, и два человека, ответственные за открытие, — это Майкл Фарадей (1791-1867), английский физик, и Джозеф Генри (1797-1878), американский физик. Процесс, при котором магнитное поле может производить ток, называется электромагнитной индукцией.

    27.2 ЭДС движения и закон электромагнитной индукции Фарадея

    Давайте рассмотрим следующий эксперимент, показанный на рисунке 27.1 (a). Две параллельные

    I

    OI

    G

    ∆x ∆AI

    Металлическая рейка

    I

    q I

    I

    L

    Проволока

    N

    v M

    v M ) Рисунок 27.1 Движущая ЭДС.

    l

    E

    B v (b)

    металлические направляющие разделены расстоянием l.На две направляющие опирается металлическая проволока. Однородное магнитное поле B прикладывается таким образом, чтобы его направление было внутрь бумаги, как показано. (Напомним, что на рисунке обозначают конец стрелки, представляющей вектор, идущий в плоскость.) Гальванометр G подсоединен поперек двух рельсов. Гальванометр показывает ноль, что указывает на отсутствие тока в цепи, состоящей из шин и проводов, то есть цепь представляет собой электрический путь, обозначенный LMNOL на рисунке 27.1 (a). Металлический провод MN теперь протягивается по рельсам со скоростью v вправо.Гальванометр теперь показывает, что в цепи течет ток. Каким-то образом движение провода через магнитное поле генерировало электрический ток. Давайте теперь проанализируем причину этого тока. Когда провод MN перемещается вправо, любой заряд q внутри провода испытывает силу F = qv  B (26.1)

    Глава 27 Электромагнитная индукция, как было показано в разделе 26.1. Если обе части уравнения 26.1 разделить на q, мы получим F = vB (27.1) q Но электростатическое поле изначально было определено как E = F qo

    (21.1)

    , где F — сила, действующая на испытательный заряд qo, покоящийся в электростатическом поле. Поэтому разумно теперь определить индуцированное электрическое поле E с помощью уравнения 27.1 как F (27.2) E = = v × B q Это другой тип поля, чем электростатическое поле. Индуцированное электрическое поле существует только тогда, когда заряд движется со скоростью v. Когда v = 0, индуцированное электрическое поле также будет равно нулю, как видно из уравнения 27.2. Индуцированное электрическое поле является причиной тока в проводе.Перекрестное произведение v  B показывает направление индуцированного электрического поля на рисунке 27.1 (b) и, следовательно, направление, в котором будет перемещаться положительный заряд q внутри провода. Следовательно, направление тока будет в направлении M  N  O  L  M, как показано на рисунке 27.1 (a). Величина индуцированного электрического поля находится из уравнения 27.2 и определения перекрестного произведения как E = vB sin  (27.3) Для конкретной задачи, рассматриваемой здесь, угол между v и B равен 900, а sin900 = 1.Следовательно, индуцированное электрическое поле равно E = vB

    (27,4)

    В главе 21 было показано, что для однородного электрического поля E = V / d, где V — разность потенциалов между двумя точками, а d — расстояние между ними. Для соединительного провода MN индуцированное электрическое поле внутри провода можно считать однородным, а индуцированный потенциал V между M и N будет обозначаться буквой E и называться индуцированной ЭДС. Расстояние d между M и N — это длина l провода. Следовательно, индуцированное электрическое поле можно записать как

    27-2

    Глава 27 Электромагнитная индукция

    Уравнивание 27.От 5 до 27,4 дает

    E = E l

    (27,5)

    E = vB l

    Следовательно, наведенная ЭДС в проводе E равна E = vBl

    (27,6)

    Если цепь имеет сопротивление R, тогда в цепи возникает индуцированный ток, определяемый законом Ома, как I = ER. Это ток, который регистрируется гальванометром.

    Пример 27.1 Индуцированная ЭДС. Проволока MN на рисунке 27.1 (a) движется вправо со скоростью 50,0 см / с. Если l = 25,0 см, B = 0,250 Тл, а полное электрическое сопротивление цепи равно 35.0 Ом, найдите (а) наведенную ЭДС в цепи, (б) ток в цепи и (в) направление тока.

    Решение а. Индуцированная ЭДС в цепи определяется из уравнения 27.6 и составляет E = vBl = (0,500 м) (0,250T) (0,250 м) (Н / (А · м)) s (T) −2 E = 3,13  10 м Н (Дж) с A (Н · м) −2 E = 3,13 10 Дж / с (AV) A (Дж / с) E = 3,13  10−2 В Обратите внимание, как манипулируют единицами измерения, чтобы получить правильные единицы для окончательного отвечать. б. Ток, протекающий в цепи, определяется по закону Ома как I = E = 3.13 10−2 В = 8,94  10−4 A R 35,0 Ом

    27-3

    Глава 27 Электромагнитная индукция c. Ток в цепи идет против часовой стрелки, как показано на рисунке 27.1 (а).

    Чтобы перейти к этому интерактивному примеру, щелкните это предложение.

    Дальнейшее понимание этой наведенной ЭДС можно получить, отметив, что скорость v провода равна v = dx / dt, где dx — расстояние, на которое провод перемещается вправо за время dt. Произведение v и l в уравнении 27.6 может быть записано как

    But

    vl = dx l dt

    (27.7)

    (dx) l = dA

    (27,8)

    — это площадь контура, смещаемая при перемещении провода вправо, как показано на рисунке 27.1 (a). Замена уравнения 27.8 в уравнение 27.7 дает vl = dA dt

    (27.9)

    И индуцированная ЭДС, уравнение 27.6, становится E = B dA dt

    (27.10)

    Но помните, что магнитный поток ΦΜ был определен как ΦΜ = B  A

    (26.101)

    Для постоянного магнитного поля B единственный способ изменения магнитного потока в нашей задаче — это изменение области A.То есть изменение магнитного потока составляет dΦΜ = B  dA

    (27.11)

    Скорость, с которой магнитный поток изменяется со временем, равна dΦΜ = B  dA dt dt

    (27.12)

    На рисунке 27.1 (a), B находится в бумаге, в то время как dA, изменение вектора площади, отсутствует, следовательно, угол между B и dA равен 1800. Используя этот факт в уравнении 27.12, мы получаем

    27-4

    Глава 27 Электромагнитная индукция

    Следовательно,

    dΦΜ = B dA cos1800 = — B dA dt dt dt

    (27.13)

    B dA = — dΦΜ dt dt

    (27.14)

    Объединение уравнений 27.10 и 27.14 дает чрезвычайно важное соотношение, известное как закон Фарадея, а именно E = — dΦm (27.15) dt Закон электромагнитной индукции Фарадея, уравнение 27.15 , утверждает, что всякий раз, когда магнитный поток изменяется со временем, возникает наведенная ЭДС. В рассматриваемом случае B была постоянной величиной, а площадь изменялась со временем. Также можно поддерживать постоянную площадь, но со временем изменять магнитное поле B.То есть, если провод MN остается закрепленным на рисунке 27.1 (a), но магнитное поле изменяется со временем, на гальванометре будет отображаться ток. Причина этого в том, что магнитный поток ΦΜ, определяемый уравнением 8-96, также может измениться при изменении B. То есть (27.16) dΦΜ = дБ A В общем случае изменение магнитного потока, вызванное изменением площади dA и изменением магнитного поля, дБ, равно dΦΜ = B  dA + A  дБ

    (27,17)

    и закон Фарадея, уравнение 27.15, также можно записать как

    E = —

    dΦ m dA dB = — B — A dt dt dt

    (27.18)

    Закон Фарадея в этой форме гласит, что ЭДС может быть вызвана либо изменением площади проволочной петли во времени, в то время как магнитное поле остается постоянным, либо сохранением постоянной площади проволочной петли и изменением магнитное поле со временем. Поскольку поток задается скалярным произведением, также можно сохранить величины B и A постоянными, но изменить угол между ними. Это также вызовет изменение магнитного потока и, следовательно, наведенную ЭДС. Этот случай будет рассмотрен позже в этой главе.Рассмотрим четыре возможных случая, связанных с этой наведенной ЭДС. Случай 1. Площадь со временем увеличивается. Это только что рассмотренный случай, показанный на рисунке 27.2 (a). Обратите внимание, что по мере увеличения A от Ai до Af изменение площади dA = Af — Ai составляет

    27-5

    Глава 27 Электромагнитная индукция

    Af Ai

    I

    v

    I

    d A = Af — Ai

    v

    II

    B

    B

    +

    B

    (a) Площадь увеличивается со временем, B — константа Ai

    v

    I + Af v

    I

    I

    I

    B

    B

    B

    d A = Af — Ai

    (b) Площадь, уменьшающаяся со временем, B — константа A

    A

    A

    II

    II

    Bi

    +

    Bf

    (c) B увеличивается со временем, A постоянная величина AA +

    d B = Bf — B i

    A

    II

    II

    Bi

    — Bf (d) B уменьшается со временем, A — константа

    Figur д 27.2 Изменение магнитного потока во времени.

    27-6

    d B = Bf — B i

    Глава 27 Электромагнитная индукция положительна и направлена ​​вверх на рисунке. Поскольку B направлен вниз, угол между векторами B и dA равен 1800, а индуцированная ЭДС равна E = — B  dA = — B dA cos1800 = B dA dt dt dt, как и было найдено ранее. На схеме также показаны полярность наведенной ЭДС E и направление тока. Случай II: Площадь со временем уменьшается. Этот случай показан на рисунке 27.2 (б). Обратите внимание, что A уменьшается от Ai до Af. Следовательно, изменение площади dA = Af — Ai отрицательно и на рисунке указывает вниз. Угол между B и dA равен 00, и тогда наведенная ЭДС равна E = — B  dA = — B dA cos00 = — B dA dt dt dt В этом случае ЭДС имеет отрицательный знак, что означает, что полярность E была обратный, и ток теперь течет по часовой стрелке. Это изменение направления тока также можно было обнаружить, заметив, что вектор скорости v теперь находится слева на рисунке.Следовательно, v  B, индуцированное электрическое поле, теперь имеет направление, противоположное тому, которое обнаружено в случае I, и, следовательно, ток должен быть обратным по сравнению со случаем I. Случай III: Магнитное поле увеличивается со временем. Этот случай показан на рисунке 27.2 (c). Площадь A остается постоянной, но магнитное поле увеличивается от Bi до Bf. Следовательно, изменение магнитного поля, дБ = Bf — Bi, указывает в том же направлении, что и Bi и Bf, что противоположно направлению A. Таким образом, угол между A и дБ равен 1800, а наведенная ЭДС равна E = — A  дБ = — A дБ cos1800 = A дБ dt dt dt Но это положительная ЭДС, как и в случае I.Следовательно, увеличивающееся магнитное поле вызывает ток против часовой стрелки, который имеет то же направление, что и ток, вызванный увеличивающейся площадью. Случай IV: Магнитное поле со временем уменьшается. Этот случай показан на рисунке 27.2 (d). Поскольку B уменьшается, изменение магнитного поля, дБ = Bf — Bi, происходит в направлении, противоположном направлению B, и указывает вверх в том же направлении, что и A. Таким образом, угол между A и дБ равен 00, а наведенная ЭДС составляет E = — A  дБ = — A дБ cos00 = — A дБ dt dt dt Обратите внимание, что это результат, противоположный тому, который обнаружен в случае III, и, следовательно, ЭДС меняется на противоположную, и ток в цепи теперь будет по часовой стрелке.27-7

    Глава 27 Электромагнитная индукция Здесь следует отметить, что хотя закон Фарадея был выведен на основе ЭДС движения, в целом он верен. Для общего случая, когда может быть N витков провода, закон индукции Фарадея принимает вид

    E = −N

    dΦ m dt

    (27,19)

    Пример 27.2. Возбуждение ЭДС путем изменения магнитного поля во времени. Провод MN на рисунке 27.1 (a) закреплен на расстоянии 10,0 см от провода гальванометра OL. Магнитное поле изменяется от 0 до 0.500 Тл за время 2,00  10−3 с. Если сопротивление цепи составляет 35,0 Ом, найдите (а) наведенную ЭДС, (б) ток в цепи при изменении магнитного поля со временем и (в) наведенную ЭДС, если магнитное поле остается постоянным. 0,500 т.

    Решение а. Индуцированная ЭДС находится из закона Фарадея, уравнение 27.18, как E = — dΦΜ = — B  dA — A  дБ dt dt dt

    (27.18)

    Поскольку провод MN закреплен, площадь контура не изменяется. со временем, т.е.е., dA = 0. Однако B изменяется со временем, и поэтому наведенная ЭДС составляет,

    Изменяющееся магнитное поле составляет

    E = — A  дБ dt

    (27.20)

    дБ = Bf — Bi

    Поскольку начальное магнитное поле Bi равно нулю, дБ имеет направление Bf, перпендикулярное бумаге и внутрь нее. Таким образом, угол между A и дБ равен 1800. Следовательно, E = — A дБ cos1800 = A дБ (27,21) dt dt Индуцированная ЭДС находится из уравнения 27.21 как E = A дБ = (0,100 м) (0.250 м) (0,500 Тл — 0 Тл) dt 2,00  10−3 с E = 6,25 Тл · м2 (Н / (А · м)) с (Тл)

    27-8

    Глава 27 Электромагнитная индукция E = 6,25 Н · м (Дж) (А) s A (Н · м) (Кл / с) E = 6,25 Дж (В) C (Дж / Кл) E = 6,25 В Обратите внимание, как были преобразованы единицы измерения. б. Индуцированный ток находится по закону Ома как I = E = R

    6,25 В = 0,179 А 35,0 Ом

    c. Когда магнитное поле остается постоянным на уровне 0,500 Тл, магнитное поле не изменяется, дБ = 0, и отсутствует наведенная ЭДС. То есть E = A dB = 0 dt Таким образом, для проволочной петли постоянной площади единственная наведенная ЭДС возникает, когда магнитное поле изменяется во времени.

    Чтобы перейти к этому интерактивному примеру, щелкните это предложение.

    27.3 Закон Ленца

    Закон электромагнитной индукции Фарадея был получен в разделе 27.2 как E = — dΦΜ dt

    (27.15)

    Знак минус в законе Фарадея очень важен. Он был получен автоматически при выводе с использованием угла между направлением векторов B и dA. Эффект этого знака минус порождает соотношение, известное как закон Ленца, и формулируется следующим образом: направление индуцированной ЭДС таково, что любой ток, который он производит, всегда противодействует через магнитное поле индуцированного тока изменению, вызывающему ЭДС.В качестве примера рассмотрим рисунок 27.2 (c). Это пример увеличения магнитного поля B со временем. Индуцированная ЭДС и ток, как показано. Посмотрите на индуцированный ток и вспомните, что всякий раз, когда ток течет по проводу, вокруг этого провода будет обнаруживаться магнитное поле. Следовательно, внутри петли будет индуцированное магнитное поле, которое направлено вверх, противодействуя возрастающему магнитному полю, направленному вниз. Наведенное магнитное поле, направленное вверх, противостоит возрастающему магнитному полю, направленному вниз, которое было причиной наведенной ЭДС, индуцированного тока и индуцированного магнитного поля.Следовательно, эффект — индуцированное магнитное поле, направленное вверх, противодействует причине — возрастающему магнитному полю, направленному вниз. Закон Ленца также является утверждением закона сохранения энергии. Если индуцированное магнитное поле находится в том же направлении, что и изменяющееся магнитное поле, общее магнитное поле увеличится еще больше, что, в свою очередь, вызовет большее магнитное поле, что снова приведет к увеличению общего магнитного поля. Процесс будет продолжаться вечно, постоянно накапливая энергию без какой-либо работы со стороны источника, что, конечно, невозможно по закону сохранения энергии.Закон Ленца — удобный инструмент, потому что он позволяет нам довольно просто определить направление индуцированного потока тока, когда конфигурация изменяющегося магнитного поля непроста. В качестве примера рассмотрим проволочную петлю на рисунке 27.3 (а). Он подключен к гальванометру, и когда стержневой магнит возле катушки неподвижен, гальванометр показывает ноль. Когда северный полюс стержневого магнита проталкивается через петлю, магнитное поле внутри петли изменяется со временем, и по закону Фарадея внутри петли индуцируется ЭДС, которая вызывает ток.Направление этого течения легко определяется законом Ленца. Поскольку причиной наведенной ЭДС является движение северного магнитного полюса к петле, индуцированный ток внутри петли вызовет магнитное поле, которое будет стремиться противодействовать этому северному магнитному полюсу. Следовательно, индуцированное магнитное поле будет выглядеть как поле северного полюса, противоположного северному полюсу стержневого магнита. (Вспомните, что противоположные магнитные полюса притягиваются, в то время как аналогичные магнитные полюса отталкиваются.) Поскольку магнитное поле исходит от северного полюса и входит в южный полюс, индуцированное магнитное поле Bi должно выходить из петли к стержневому магниту.Если правая рука обхватить любую часть проволочной петли так, что пальцы будут указывать в направлении индуцированного магнитного поля, большой палец будет указывать в направлении тока. Таким образом, направление индуцированного тока в контуре показано на диаграмме на рисунке 27.3 (a). Если стержневой магнит отводится от петли, как на рисунке 27.3 (b), то индуцированное магнитное поле должно иметь тенденцию препятствовать оттягиванию магнита. Следовательно, поверхность петли будет вести себя как южный магнитный полюс, притягивая стержневой магнит.Магнитное поле индуцированного южного полюса Bi будет направлено внутрь петли. Используя правило для правой руки и обхватив рукой провод, пальцы должны указывать в направлении индуцированного магнитного поля Bi, а большой палец указывает в направлении тока, как показано на рисунке 27.3 (b). Обратите внимание, что когда магнит подталкивают к петле, направление индуцированного тока в петле — по часовой стрелке; когда магнит отводят от петли, ток направляется против часовой стрелки.Случай перемещения южного магнитного полюса к петле показан на рис. 27.3 (c), а отведение его от петли показано на рис. 27.3 (d). Направление индуцированного тока легко найти с помощью анализа закона Ленца.

    27-10

    Глава 27 Электромагнитная индукция

    S

    (b) Северный полюс отошел от петли.

    N

    N

    S

    S

    Bi

    Bi

    N

    движение

    движение

    S

    (a) Северный полюс перемещается в сторону петли.

    N

    Bi S

    Bi

    N

    N

    S

    движение

    движение

    (d) Южный полюс отошел от петли (c) Южный полюс сместился в сторону петли. Рис. 27.3. Определение направления индуцированного тока по закону Ленца.

    27.4 Индуцированная ЭДС во вращающейся петле провода в магнитном поле — переменная ЭДС и генератор переменного тока

    В главе 26 мы видели, что когда токоведущая петля провода помещается во внешнее магнитное поле, крутящий момент образуется на петле, заставляя ее вращаться в магнитном поле.Напомним, в этом заключалась суть электродвигателя. Давайте теперь посмотрим на обратную задачу и спросим, ​​что произойдет, если проволочную петлю механически повернуть во внешнем магнитном поле? Может ли в петле индуцироваться ток? Конечно, да, и описываемое устройство будет электрическим генератором. Рассмотрим проволочную петлю, показанную на рис. 27.4 (a) и (b). Петля приводится во вращение с угловой скоростью ω. Часть проволочной петли ab движется с

    ω

    B

    c

    b

    EN v

    B d

    E

    v S

    N

    B a

    v

    r θ r

    v

    S

    θ

    B

    (b) Вид сверху

    (a) Вид сбоку

    Рисунок 27.4 Вращающаяся петля из проволоки во внешнем магнитном поле. скорость v через магнитное поле B. В проводе ab будет индуцироваться электрическое поле, заданное уравнением 27.2 как E = vB (27.2) Направление E определяется перекрестным произведением v  B и показано указывающим вниз на рис. 27.4 (а). Индуцированный ток будет течь в этом направлении. Величина индуцированного электрического поля определяется уравнением 27.3 как E = vB sinθ

    (27.3)

    , где θ — угол между вектором скорости v и магнитным полем B, как показано на рисунке 27.4 (б). Обратите внимание, что θ — это также угол, через который проходит петля. Индуцированное электрическое поле в проводе ab определяется уравнением 27.5 как E = E l

    27-12

    (27,5)

    Глава 27 Электромагнитная индукция где l — длина провода от a до b. Объединение уравнений 27.5 и 27.3 дает для наведенной ЭДС в проводе ab (27.22) Eab = vBl sinθ Проволока cd будет иметь индуцированное электрическое поле, направленное вверх, как показано на рисунке 27.4 (a). Следовательно, индуцированный ток в проводе cd будет течь вверх в направлении электрического поля.Индуцированная ЭДС в проводе cd аналогична проводу ab и определяется выражением (27.23) Ecd = vBl sinθ Вдоль проводов bc и da индуцированное электрическое поле, E = v  B, перпендикулярно самому проводу и не может вызвать никакого тока. протечь по длине проводов bc или da. Таким образом, ЭДС Ebc = Eda = 0. Следовательно, полная ЭДС вокруг петли является суммой наведенной ЭДС вдоль ab, Eab и наведенной ЭДС вдоль cd, Ecd. То есть E = Eab + Ecd

    (27.24)

    Замена уравнений 27.22 и 27.23 в уравнение 27.24 дает или

    E = vBl sinθ + vBl sinθ E = 2vBl sinθ

    (27.25)

    Катушка вращается с угловой скоростью ω, а линейная скорость v провода ab и cd определяется выражением v = ωr (27.26) Подстановка v из уравнения 27.26 обратно в уравнение 27.25 дает для наведенной ЭДС в катушке E = 2 (ωr) Bl sinθ (27.27) Но 2r — это длина провода bc, а поскольку l — длина провода ab, произведение 2rl — это площадь контура, то есть A = 2rl (27.28) Таким образом, полная наведенная ЭДС в катушке составляет E = ωAB s в θ

    (27.29)

    Поскольку катушка вращается с угловой скоростью ω, угол θ, который можно повернуть в любой момент, равен θ = ωt. Суммарная ЭДС, индуцированная в катушке в любой момент времени, составляет 27-13

    Глава 27 Электромагнитная индукция E = ωAB sinωt

    (27.30)

    Уравнение 27.30 говорит, что для получения большой наведенной ЭДС в катушке: катушка должна двигаться с высокой угловой скоростью ω; имеют относительно большую площадь А; и быть помещенным в сильное магнитное поле B. Уравнение 27.30 также указывает, что наведенная ЭДС E синусоидально изменяется со временем, как показано на рисунке 27.4 (в). Максимальная ЭДС возникает, когда sinωt равна 1 и обозначается Emax, где Emax = ωAB (27.31) Индуцированная ЭДС может быть записана как E = Emax sinωt

    (27.32)

    Если катушка подключена к цепи который имеет сопротивление R, ток, который будет течь от катушки к цепи, определяется законом Ома как i = E = Emax sinωt RR

    (27.33)

    Уравнение 27.33 показывает, что ток в цепи также будет изменяться синусоидально со временем, и это показано на рисунке 27.4 (г). Ток начинается с нуля и увеличивается до максимального значения, определяемого как imax = Emax R

    (27,34)

    Таким образом, изменение тока во времени определяется выражением i = i max sin t

    (27,35)

    Изменение этого тока легче понять, изучив рисунок 27.5. На рис. 27.5 (а) показан индуцированный ток как функция θ, угла поворота петли. На рисунке 27.5 (b) показано положение вращающейся катушки в различных положениях. В позиции 0 на рисунке 27.5 (б), скорость v параллельна магнитному полю B, θ = 0, и, следовательно, индуцированное электрическое поле (E = vB sin θ) равно нулю. Таким образом, наведенные ЭДС и ток в катушке в этом положении равны нулю. В положении 1 катушка повернулась на 450. Электрическое поле направлено вниз в проводе ab и вверх в проводе cd. Следовательно, ток течет в катушке в направлении от b к a к d к c. Величина тока в позиции 1 составляет 0,707 imax. В позиции 2 катушка повернулась на 900. Направления электрических полей в проводах такие же, как в позиции 1, и, следовательно, ток течет в том же направлении.Ток, протекающий в катушке, теперь имеет максимальное значение. В положении 3 катушка повернулась на угол 1350. Направление электрических полей и, следовательно, токи остались прежними. Однако теперь ток уменьшается до i3 = imax sin 1350 = 0,707 imax. В положении 4 катушка повернулась на угол 1800. Теперь v антипараллельно B, θ = 1800, и, следовательно, (vB sin 1800) равно нулю. Следовательно, в этом положении электрическое поле внутри провода равно нулю, как и ток.В позиции 5 катушка повернулась на угол 2250. Теперь E в проводе ab указывает вверх, изменяя направление электрического поля и, следовательно, направление тока. Это показано на рисунке 27.5 (a) как отрицательный ток -0,707 imax. Отрицательный ток означает, что направление тока было изменено с первоначального на противоположное. В позиции 6 катушка повернулась на угол 2700. Здесь v перпендикулярно B, что дает максимальный ток в отрицательном направлении.В положении 7 катушка повернулась на угол 3150, и ток снизился до -0,707 мах. Когда катушка вращает последние 450, она возвращается в положение 0; электрическое поле и ток снова равны нулю. Цикл повторяется, когда катушка вращается, и получается ток, который меняется по направлению и величине. Эта вращающаяся проволочная петля во внешнем магнитном поле называется генератором переменного тока. AC означает переменный ток, потому что ток синусоидально изменяется от одного направления к другому.Ток изменяется синусоидально, потому что наведенная ЭДС в катушке изменяется синусоидально. Помимо изменения направления, величина переменного тока постоянно изменяется. Это сильно отличается от постоянного тока, величина которого остается постоянной. Обратите внимание, что в генераторе переменного тока величины векторов B и A постоянны, но направление между двумя векторами постоянно меняется со временем. Поскольку магнитный поток равен BA cos θ, если угол θ изменяется со временем, также происходит изменение магнитного потока и, следовательно, индуцированная ЭДС.Таким образом, генератор переменного тока основан на том факте, что угол θ между вектором скорости v и вектором магнитного поля B изменяется со временем, вызывая изменяющийся поток, который создает индуцированное электрическое поле, потенциал и ток.

    27-15

    Глава 27 Электромагнитная индукция

    27-16

    Глава 27 Электромагнитная индукция

    Рисунок 27.5 Ток во вращающейся катушке, соответствующий различным положениям катушки. Результаты для генератора переменного тока можно было бы получить как чисто математический результат, отметив, что, согласно закону Фарадея, наведенная ЭДС определяется выражением E = — dΦΜ dt

    или

    E = — Поскольку B и A являются константами E = — БА

    (27.15)

    d (BA cos θ) dt

    d (cos θ) dt

    = — BA (- sin θ)

    dθ dt

    d =  Но dt, угловая скорость вращающейся катушки. Следовательно, наведенная ЭДС во вращающейся катушке становится E = ω AB sin θ, что является тем же результатом, что и в уравнении 27.30. Как видите, это очень простой вывод наведенной ЭДС для генератора переменного тока, но часть физической картины теряется в этом чисто математическом процессе. С практической точки зрения обычно желательно получить от генератора переменного тока еще большую ЭДС, чем полученная из уравнения 27.32. Этого можно достичь, увеличив количество витков, составляющих катушку, до N витков. В примере 27-17

    Глава 27 Электромагнитная индукция, если используются два витка, напряжение и ток увеличиваются вдвое, если используются десять витков, напряжение и ток увеличиваются в десять раз. Для катушки из N витков уравнения 27.32 и 27.33 умножаются на коэффициент N, и напряжение и ток увеличиваются в N раз.

    Пример 27.3 Генератор переменного тока. Катушка из 100 витков провода с площадью поперечного сечения 100 см2 вращается с угловой скоростью 377 рад / с во внешнем магнитном поле 0.450 Т. Найдите (а) максимальную наведенную ЭДС, (б) частоту переменной ЭДС, (в) если сопротивление цепи равно 100 Ом, найдите ток и максимальный ток.

    Решение а. Индуцированная ЭДС находится из уравнения 27.30 как E = NωAB sinωt

    (27.30)

    Emax = NωAB Emax = (100) (377рад) (100 см2) (1 м2) (0,450 Тл) с (104 см2) 2 Emax = 170 T m (N / (A m)) (A) (J) (V) s (T (C / s) (N m) (J / C) Emax = 170 V

    (27.31)

    и максимальная ЭДС составляет

    Обратите внимание, как коэффициенты пересчета единиц отменяются, оставляя правильную единицу для ответа.б. Частота вращения связана с угловой скоростью контура уравнением 13.9 как ω = 2πf f = ω 2π или f = 377 рад / с = 60,0 циклов / с = 60,0 Гц 2π Таким образом, частота 60,0 Гц получается из генератор путем вращения катушки с угловой скоростью 377 рад / с. c. Ток определяется из уравнения 27.33 как

    27-18

    Глава 27 Электромагнитная индукция i = Emax sinωt R i = 170 В sinωt 100 Ом i = (1,70 A) sinωt, а максимальный ток составляет 1,70 A.

    To go к этому интерактивному примеру щелкните это предложение.

    27,5 Взаимная индукция

    Рассмотрим два коаксиальных соленоида одинакового размера, показанные на рисунке 27.6. (Один синий провод и один красный провод намотаны на полый рулон картона.) Если

    Рисунок 27.6 Два коаксиальных соленоида. ток i1 в катушке 1 изменяется со временем, это со временем изменяет магнитное поле B1 соленоида 1. Но соленоид 2 находится в магнитном поле катушки 1, и любое изменение B1 вызовет наведенную ЭДС в катушке 2. Из закона индукции Фарадея это можно сформулировать как E2 = — N2 dΦΜ dt E2 = — N2 d (B1A) dt E2 = — N2A дБ1 (27.36) dt Магнитное поле внутри соленоида 1 находится из уравнения 8-58 как B1 = µon1i1 B1 = µo N1 i1 l 27-19

    (27,37)

    Глава 27 Электромагнитная индукция Когда i1 изменяется со временем, B1 изменяется как dB1 = µo N1 di1 l

    (27.38)

    Замена уравнения 27.38 на уравнение 27.36 дает

    или Определим величину

    E2 = — N2A (µo N1 di1) l dt E2 = — µoA N1N2µo1 l dtN = M l

    (27,39)

    — коэффициент взаимной индукции коаксиальных соленоидов.Теперь закон Фарадея принимает вид E2 = — M di1 (27.40) dt Уравнение 27.40 говорит, что изменение тока i1 в катушке 1 со временем индуцирует ЭДС в катушке 2. Обратите внимание, что M в уравнении 27.39 зависит только от геометрии соленоидов 1 и 2, и является константой для конкретного набора выбранных соленоидов. Для других конфигураций, кроме коаксиальных соленоидов, очень трудно, а иногда почти невозможно определить M таким способом. Однако даже в этих сложных случаях M всегда можно найти, измерив величины di1 / dt и E2.Затем M находится из уравнения 27.40 как M = — E2 (27.41) di1 / dt Следовательно, уравнение 27.40 можно использовать для определения наведенной ЭДС в катушке 2, создаваемой изменяющимся током в катушке 1, независимо от фактической геометрической конфигурации. Если ЭДС в уравнении 27.41 выражается в вольтах, а ток и время — в амперах и секундах, соответственно, то единица измерения взаимной индуктивности M называется генри и выражается как 1 генри = вольт-ампер / секунда. Это будет сокращенно 1H. = ВА / с

    27-20

    Глава 27 Электромагнитная индукция Используя аналогичную аналогию, можно изменять ток в катушке 2, и в катушке 1 будет индуцироваться ЭДС, определяемая как

    Где и

    E1 = — N1 dΦΜ = — N1A дБ2 dt dt дБ2 = µo N2 di2 l E1 = — N1AµoN2 di2 l dt E1 = — µo AN1N2 di2 l dt

    (27.42)

    Обратите внимание, что коэффициент di2 / dt в уравнении 27.42 — это коэффициент взаимной индукции, найденный в уравнении 27.39. Следовательно, наведенная ЭДС в катушке 1, вызванная изменением тока в катушке 2, определяется выражением E1 = — M di2 dt

    (27,43)

    Пример 27.4 Коэффициент взаимной индуктивности коаксиальных соленоидов. Найдите взаимную индуктивность двух коаксиальных соленоидов радиусом 5,00 см и длиной 30,0 см, если одна катушка имеет 10 витков, а вторая — 1000 витков.

    Решение Площадь соленоида находится как A = πr2 = π (0.0500 м) 2 = 7,85 10−3 м2 Коэффициент взаимной индуктивности находится из уравнения 27.39 как

    и

    M = µoA N1N2 l −7 M = (4π  10 Тл м) (7,85  10−3 м2 ) (10) (1000) A 0,300 м M = 3,29 10−4 Тл м2 (Н / (Кл м / с)) A (T) −4 M = 3,29  10 Нм (Дж) (В) AC / с (Н · м) (Дж / К) 27-21

    Глава 27 Электромагнитная индукция M = 3,29 10−4 В (H) A / s (В / (A / s)) M = 3,29  10−4 H Примечание как используются коэффициенты пересчета единиц, чтобы указать правильную единицу для ответа.

    Чтобы перейти к этому интерактивному примеру, щелкните это предложение.

    Пример 27.5 Индуцированная ЭДС во второй катушке. Если ток в первой катушке 27,4 изменяется на 2,00 А за 0,001 секунды, какова наведенная ЭДС во второй катушке?

    Решение Индуцированная ЭДС во второй катушке, определяемая из уравнения 27.40, равна E2 = — M di1 = — (3,29  10−4 H) (2,00 A) dt 0,001 с −1 E2 = — 6,58  10 H (V / (A / s)) A (H) s E2 = — 0,658 В

    Чтобы перейти к этому интерактивному примеру, щелкните это предложение.

    27.6 Самоиндукция

    Изменяющийся магнитный поток в одной катушке не только индуцирует ЭДС в соседней катушке, но и сам по себе.То есть, если ток изменяется в одном соленоиде, изменяющийся ток будет создавать изменяющееся магнитное поле, а изменяющееся магнитное поле будет индуцировать ЭДС в самом одиночном соленоиде. Этот процесс называется самоиндукцией и может быть объяснен законом электромагнитной индукции Фарадея. Один соленоид показан на рисунке 27.7. Если два конца катушки соленоида присоединены к генератору переменного тока секции 27.4 (показан в виде круга с синусоидальной волной на рисунке 27.7), переменный ток в катушке вызовет наличие переменного магнитного поля в катушке. .Поскольку магнитное поле соленоида определяется выражением B = µoni (27,44)

    27-22

    Глава 27 Электромагнитная индукция

    I

    Рисунок 27.7 Самоиндукция. изменение тока i вызывает изменение магнитного поля на dB = µondi

    (27.45)

    Индуцированная ЭДС определяется законом Фарадея как E = — N dΦΜ = — N d (BA) dt dt Поскольку площадь A катушка не изменяется, это становится E = — NA дБ dt

    (27.46)

    Изменяющееся магнитное поле внутри соленоида представлено в уравнении 27.45 и подставив его в уравнение 27.46, получим E = — NAµon di dt

    (27.47)

    Общее количество витков N связано с n числом витков на единицу длины следующим образом: N = nl, где l — длина соленоида. . Замена этого в уравнении 27.47 дает E = — (µoAln2) di dt. Обратите внимание, что коэффициент di / dt является константой, которая зависит только от геометрии катушки соленоида. Эта постоянная называется самоиндукцией соленоидной катушки и обозначается буквой L, где

    27-23

    Глава 27 Электромагнитная индукция Самоиндуцированная ЭДС теперь равна

    L = µoAln2

    (27.48)

    E = — L di dt

    (27.49)

    Уравнение 27.49 говорит, что изменение тока в катушке вызовет в катушке ЭДС, а знак минус означает, что индуцированная ЭДС будет действовать, чтобы противодействовать причине. наведенной ЭДС. То есть, если приложенная ЭДС положительна, самоиндуцированная ЭДС отрицательна; если приложенная ЭДС отрицательна, самоиндуцированная ЭДС положительна. Другими словами, если ток в катушке увеличивается, направление наведенной ЭДС противоположно направлению тока.Если ток уменьшается, наведенная ЭДС имеет то же направление, что и исходный ток. Во многих случаях будет очень трудно, если не невозможно, вывести L, как мы это делали для частного случая соленоида в уравнении 27.48. Однако во всех случаях уравнение 27.49 выполняется и может использоваться для определения самоиндукции как L = —

    E di / dt

    (27,50)

    E и di / dt могут быть измерены экспериментально для любой конфигурации катушки и L можно определить из уравнения 27.50. L, самоиндукция, обычно называется индуктивностью катушки и измеряется в генри, как и взаимная индуктивность, т.е.е. 1H = 1 В А / с. Элемент схемы, в котором самоиндуцированная ЭДС сопровождает изменяющийся ток, называется индуктором. L — индуктивность катушки индуктивности, представленная на принципиальной схеме символом катушки.

    Пример 27.6 Индуктивность катушки. Батарея подключается через переключатель к катушке соленоида. Катушка имеет 50 витков на сантиметр, диаметр 10,0 см и длину 50,0 см. Когда переключатель замкнут, ток изменяется от 0 до максимального значения 3,00 A в 0.002 секунды. Найдите индуктивность катушки и наведенную ЭДС в катушке за этот период.

    Решение Площадь поперечного сечения катушки A = πd2 = π (0,100 м) 2 = 7,85  10−3 м2 4 4 27-24

    Глава 27 Электромагнитная индукция и 50 витков на сантиметр катушки равны 5000 витков на метр. Индуктивность катушки соленоида определяется из уравнения 27.48 как L = µoAln2 L = (4π  10 Тл) (7,85  10−3 м2) (0,500 м) (5000) 2 A (м2) 2 L = 0,123 Гн — 7

    Индуцированная ЭДС в катушке находится из уравнения 27.49 как E = — L di dt E = — (0,123 H) (3,00 A — 0 A) 0,002 с E = — 185 В Знак минус на E указывает, что напряжение противоположно напряжению батареи V.

    Чтобы перейти к этот Интерактивный пример щелкните это предложение.

    Пример 27.7 Индуцированная ЭДС при постоянном токе. Какова наведенная ЭДС в 27,6 через 0,002 секунды?

    Решение Поскольку максимальный ток в цепи, 3,00 А, достигается через 0,002 с, изменения тока во времени больше не происходит, т. Е. Di / dt = 0.Следовательно, индуцированная ЭДС определяется выражением E = — L di = — (0,123 Гн) (0) = 0 dt. Когда ток является установившимся, в цепи больше нет самоиндуцированной ЭДС.

    Чтобы перейти к этому интерактивному примеру, щелкните это предложение.

    27-25

    Глава 27 Электромагнитная индукция

    Пример 27.8 Индуцированная ЭДС при затухании тока. Переключатель в приведенных выше примерах теперь открыт. Если ток спадает с той же скоростью, с которой он нарастал в цепи, найдите наведенную ЭДС в цепи.

    Решение Индуцированная ЭДС определяется уравнением 27.49 как E = — L di dt E = — (0,123 H) (0 — 3,00 A) 0,002 с E = + 185 В Обратите внимание, что E теперь положительно, потому что ток идет от От 3,00 A до 0 A, следовательно, di отрицательно. Эта положительная наведенная ЭДС препятствует спаду тока в цепи.

    Чтобы перейти к этому интерактивному примеру, щелкните это предложение.

    27.7 Энергия, запасенная в магнитном поле индуктора

    В разделе 25.5 было показано, что энергия может храниться в электрическом поле между пластинами конденсатора.Подобным образом энергия может храниться в магнитном поле катушек индуктора. Когда переключатель на рисунке 27.8 (a) замкнут, всего

    с

    L

    V

    (a)

    с V

    (b) Рисунок 27.8 Самоиндукция

    27-26

    L

    E

    Глава 27 Электромагнитная индукция Приложенное напряжение V подается на концы катушки соленоида. Начальный ток i равен нулю. Чтобы ток увеличился, необходимо снять заряд dq с положительной стороны батареи и направить его против индуцированной ЭДС E в катушке.Объем работы, выполняемой аккумулятором при перемещении этого небольшого количества заряда, равен dW = (dq) E (27,51). Но величина наведенной ЭДС определяется уравнением 27.49 как E = L di dt

    (27,49)

    Следовательно, небольшой объем проделанной работы равен dW = dq L di dt

    После перестановки терминов

    dW = dq L di dt dW = iL di

    (27,52)

    , где использовался тот факт, что dq / dt = i, текущий. Однако этот ток i не является постоянным, а изменяется со временем.Следовательно, небольшой объем выполненной работы не является постоянным, а зависит от тока i в цепи. Общая проделанная работа равна сумме или интегралу всех этих dW, то есть = W

    dW ∫ ∫ =

    I

    0

    Lidi =

    W = 12 LI 2

    1 2

    Li 2

    I 0

    (27,53) (27,54)

    Эта работа W, выполняемая аккумулятором при заряде, отображается как потенциальная энергия заряда. Считается, что эта энергия находится в магнитном поле катушки, рис. 27.8 (b) и обозначается как UΜ. Таким образом, энергия, запасенная в катушке индуктивности, определяется выражением

    UM = 12 LI 2

    (27,55)

    Эту накопленную энергию также можно выразить через магнитное поле B, вспомнив, что для соленоида L = µoAln2 (27,48 ) и (27.44) B = µonI Решая уравнение 27.44 для тока I, мы получаем

    27-27

    Глава 27 Электромагнитная индукция I = B µon

    (27,56)

    Заменяя уравнения 27.48 и 27.56 в уравнение 27.55 дает UΜ = 1/2 LI2 = 1/2 (µoAln2) (B) 2 (µon) 2 UΜ = 1/2 B2Al µo

    (27,57)

    Уравнение 27.57 дает энергию, запасенную в магнитном поле соленоида. . Плотность энергии определяется как энергия на единицу объема и может быть представлена ​​как uΜ = UΜ = 1/2 (B2Al) / µo VV Но объем соленоида равен V = Al Следовательно, 2 u M = 12 B o

    (27.58)

    Уравнение 27.58 дает плотность магнитной энергии или энергию на единицу объема, которая хранится в магнитном поле.Хотя уравнения 27.54 и 27.58 были выведены для соленоида, они являются совершенно общими и применимы к любой катушке индуктивности. Обратите внимание на сходство с плотностью энергии электрического поля. uE = 1/2 εoE2

    (25,57)

    Пример 27.9 Энергия, запасенная в магнитном поле индуктора. Какая энергия хранится в магнитном поле в 27,6?

    Решение В этом примере индуктивность оказалась равной L = 0,123 Гн, а конечный ток — 3,00 А. Следовательно, энергия, запасенная в магнитном поле, находится из уравнения 27.55 как UΜ = 1/2 LI2 UΜ = 1/2 (0,123 H) (3,00 A) 2 UΜ = 0,554 H A2 (В / (А / с)) (H) UΜ = 0,554 CV · с (Дж / C) · с (В) UΜ = 0,554 Дж 27-28

    Глава 27 Электромагнитная индукция

    Чтобы перейти к этому интерактивному примеру, щелкните это предложение.

    27.8 Сравнение электростатического поля и индуцированного электрического поля

    При изучении потенциала точечного заряда в главе 23 мы показали, что разность потенциалов между двумя точками A и B в электрическом поле определяется выражением VB — VA = — ∫ E  dl

    (23.21)

    Уравнение 23.21 говорит, что если мы начнем с позиции A, где потенциал равен VA, и перейдем в позицию B, где потенциал равен VB, то разность потенциалов VA — VB будет равна интегралу от E  дл. Но что произойдет, если после достижения B мы вернемся к A, какова же тогда разница в потенциале? Из уравнения 23.21 видно, что разность потенциалов в этом случае будет 0 = — ∫ Edl VA — VA =

    (27.59)

    Обратите внимание, что теперь знак интеграла обведен кружком, что означает, что интегрирование находится вокруг закрытой тропы, мы вернулись туда, откуда начали.Запишем уравнение 27.59 в стандартной форме (27.60) Edl = 0 Уравнение 27.60 является характеристикой всех электростатических полей. Это говорит о том, что электростатическое поле — консервативное поле. Это означает, что работа, совершаемая при перемещении заряда в электростатическом поле, не зависит от пройденного пути. Характерной чертой консервативного поля является наличие потенциала, и мы изучали этот электростатический потенциал в главе 23. Мы скоро увидим, что это не то же самое для индуцированного электрического поля.Давайте теперь вернемся к проблеме двигательной ЭДС, изученной в разделе 27.1 и проиллюстрированной на рисунке 27.1. Вычислим интеграл от E  dl для цикла MNOLM на рисунке 27.1. Поскольку интеграл охватывает весь цикл, мы можем разбить интеграл на две части: первая — по пути MN, а вторая — по пути NOLM, т. Е. (27.61)  = ∫ Edl ∫ Edl + ∫ Edl MN

    NOLM

    Но на пути NOLM нет электрического поля E, тогда как электрическое поле E вдоль пути MN представляет собой индуцированное электрическое поле, определяемое уравнением 27.2 как 27-29

    Глава 27 Электромагнитная индукция E = vB

    (27.2)

    , которая вызвана движением провода в магнитном поле B. Уравнение 27.61 принимает вид (27.62) ∫ Edl = ∫ (v × B) dl + 0 MN

    ∫ Edl = ∫ (v × B) dl = ∫ vB sin 90 dl cos0 0

    MN

    0

    = vBl

    (27,63)

    MN

    , но мы показали в уравнении 27.6, что

    E = vBl

    (27,6)

    Объединение уравнений 27.6 с 27,63 дает E = ∫ Edl

    (27,64)

    Уравнение 27.64 говорит, что интеграл от E  dl вокруг замкнутого контура равен наведенной ЭДС E в контуре. Сравните уравнения 27.60 и 27.64, они показывают основное различие между электростатическим полем и индуцированным электрическим полем. Для электростатического поля интеграл от E  dl вокруг замкнутого контура равен нулю, в то время как для индуцированного электрического поля интеграл от E  dl вокруг замкнутого контура равен наведенной ЭДС E в контуре.Следовательно, электростатические поля сильно отличаются от индуцированных электрических полей. Как мы вскоре покажем, электростатическое поле постоянно во времени, тогда как индуцированное электрическое поле изменяется со временем.

    27.9 Обобщение закона электромагнитной индукции Фарадея

    Мы показали, что закон Фарадея можно записать в виде E = —

    dΦ m dt

    (27.15)

    Мы использовали для магнитного потока соотношение ΦM = B  A

    (26.101)

    Уравнение 26.101 использовался для описания величины магнитного потока, проходящего через плоскую поверхность A. Но в целом поверхность могла иметь любую форму. Чтобы справиться с этим случаем, поверхность разбивается на большое количество бесконечно малых

    27-30

    Глава 27 Электромагнитная индукция площадей dA, и вычисляется бесконечно малое количество потока dΦM через каждую из этих маленьких площадей, т. Е. (27,65 ) dΦM = BdA Полный поток через поверхность становится суммой или интегралом всех бесконечно малых потоков dΦM через все бесконечно малые области dA, то есть ΦM =

    ∫ dΦ

    M

    =

    ∫ B  dA

    (27.66)

    Мы можем подставить уравнение 27.66 в уравнение 27.15, чтобы получить dΦ md E = — = — ∫ BdA dt dt

    (27.67)

    Но мы также обнаружили в уравнении 27.64, что индуцированная ЭДС также может быть задана как E = ∫ Edl

    (27,64)

    Приравнивая уравнение 27.64 к уравнению 27.67, получаем

    ∫ Edl =

    d BdA dt ∫

    (27,68)

    Уравнение 27.68 является интегральной формой обобщения закона электромагнитной индукции Фарадея.

    Язык физики Закон электромагнитной индукции Фарадея Каждый раз, когда магнитный поток, проходящий через катушку, изменяется со временем, в катушке индуцируется ЭДС. Магнитный поток можно изменить, изменив магнитное поле B, площадь A петли или направление между магнитным полем и вектором площади (p). Закон Ленца. Направление индуцированной ЭДС таково, что любой ток, который она производит, всегда противодействует через магнитное поле индуцированного тока изменению, вызывающему ЭДС (p).Генератор переменного тока Устройство, в котором катушка с проволокой вручную вращается во внешнем магнитном поле. Вращающаяся катушка имеет переменную, синусоидально изменяющуюся ЭДС и ток, индуцированный в катушке. Генератор переменного тока является источником переменного тока (p). 27-31

    Глава 27 Электромагнитная индукция Взаимная индукция Изменение магнитного потока в одной катушке индуцирует ЭДС в соседней катушке (p). Самоиндукция Изменение магнитного потока в катушке вызывает в катушке ЭДС. Индуцированная ЭДС противодействует изменению магнитного потока (p).Индуктор. Элемент схемы, в котором самоиндуцированная ЭДС сопровождает изменяющийся ток (p).

    Сводка важных уравнений Индуцированное электрическое поле

    E = F / q = v  B

    Величина индуцированного электрического поля Закон индукции Фарадея Закон Фарадея для контуров N Индуцированная ЭДС во вращающейся катушке

    E = vB sinθ E = — dΦΜ = — B  dA — A  дБ dt dt dt E = — N dΦΜ dt E = ωAB sinωt

    Переменная ЭДС

    Индуктивность соленоида Самоиндуцированная ЭДС катушки

    (27.18) (27,19) (27,30) (27,32)

    i = imax sinωt

    (27,35)

    ЭДС, индуцированная в катушке 2 путем изменения тока в катушке 1 M = —

    (27,3)

    E = Emax sinωt

    Переменный ток

    Взаимная индуктивность

    (27,2)

    E2 = — M di1 dt

    (27,40)

    E2 di1 / dt L = µoAln2

    (27,41)

    dt =

    (27,49)

    Индуктивность

    L = — E di / dt

    Энергия, запасенная в магнитном поле индуктора Энергия, запасенная в магнитном поле соленоида

    27-32

    UΜ = 1/2 LI2 UΜ = 1 / 2 B2Al µo

    (27.48)

    (27,50) (27,55) (27,57)

    Глава 27 Электромагнитная индукция Плотность магнитной энергии

    uΜ = 1/2 B2 µo

    Характеристика всех электростатических полей

    ∫ Edl = 0

    Характеристика индуцированных электрических полей

    E =  ∫ Edl

    Интегральная форма закона электромагнитной индукции Фарадея d ∫ Edl = — dt ∫ BdA

    (27,58) (27,60) (27,64)

    ( 27.68)

    Вопросы к главе 27 * 1. Если изменение магнитного потока во времени индуцирует электрическое поле, вызывает ли изменение электрического потока со временем магнитное поле? 2.Если бы металлическую проволоку MN на рисунке 27.1 заменить деревянной палкой, как это повлияло бы на эксперимент? 3. Покажите, что если бы вектор площади на рисунке 27.2 был определен в противоположном направлении, анализ не соответствовал бы закону Ленца. Таким образом, покажите, что выбор направления для вектора площади A не может быть произвольным. 4. Можно ли изменить площадь петли и магнитное поле, проходящее через петлю, и при этом в петле не будет наведенной ЭДС? 5. Обсудите закон Ленца.6. Можно ли использовать электродвигатель для привода генератора переменного тока, при этом выходная мощность генератора используется для управления двигателем? 7. Обсудите, как энергия сохраняется в магнитном поле катушки. Сравните это с тем, как энергия сохраняется в электрическом поле конденсатора. * 8. Если изменение электрического поля во времени создает магнитное поле, а изменение магнитного поля во времени создает электрическое поле, возможно ли, чтобы эти изменяющиеся поля соединялись вместе и распространялись в пространстве?

    Проблемы для главы 27 27.2 ЭДС движения и закон электромагнитной индукции Фарадея 1. Магнитный поток, проходящий через катушку из 10 витков, изменяется от 5,00  10–4 Вт до 5,00 10–3 Вт за 1,00  10–2 с. Найти наведенную ЭДС в катушке. 2. Круглая катушка диаметром 6,00 см помещается в однородное магнитное поле 0,500 Тл. Если B упадет до 0 за 0,002 с, найдите максимальную наведенную ЭДС в катушке. 3. Круглая катушка с 25 витками диаметром 6,00 см помещается перпендикулярно магнитному полю, изменяющемуся со скоростью 2,50  10-2 Тл / с.(a) Найдите наведенную ЭДС в катушке. (b) Если сопротивление катушки составляет 25,0 Ом, найдите наведенный ток в катушке. 4. Круглая катушка диаметром 5,00 см имеет сопротивление 2,00 Ом. Если в катушке должен протекать ток 4,00 А, с какой скоростью должно изменяться магнитное поле со временем, если (а) катушка перпендикулярна магнитному полю и (б) если катушка образует угол 300 с полем ? 5. Провод МН на схеме рисунка 27.1 (а) движется вправо со скоростью 25,0 см / с. Если l = 20,0 см, B = 0,300 Тл, а полное сопротивление цепи составляет 50,0 Ом, найдите (а) наведенную ЭДС в проводе MN, (б) ток, протекающий в цепи, и (в) направление электрический ток. 6. Повторите задачу 5, если проволока MN движется влево со скоростью 25,0 см / с. 7. Если проволока MN задачи 5 движется со скоростью 50,0 м / с вправо, с какой скоростью должна изменяться величина B, когда проволока находится на расстоянии 5,00 см от левого конца петли, так что будет нет наведенной ЭДС в цепи? 8.Проволока MN (l = 25,0 см) на рисунке 27.1 (a) закреплена на расстоянии 20,0 см от провода OL гальванометра. Сопротивление цепи 50,0 Ом. (а) Если магнитное поле изменяется от 0 до 0,350 Тл за время 0,030 с, найдите наведенные ЭДС и ток в цепи за это время. (b) Если магнитное поле остается постоянным на уровне 0,350 Тл, найдите наведенные ЭДС и ток в катушке. (c) Если магнитное поле спадает с 0,350 Тл до 0 Тл за 0,0200 с, найдите наведенную ЭДС, ток и его направление в проводе.9. Самолет летит со скоростью 200 узлов через область, где вертикальная составляющая магнитного поля Земли составляет 3,50  10−5 T. Если размах крыла составляет 12,0 м, какова разница потенциалов между концом крыла и концом крыла? Можно ли использовать эту разность потенциалов в качестве источника тока для работы оборудования самолета? 10. Поезд движется со скоростью 40,0 км / ч в месте, где вертикальная составляющая магнитного поля Земли составляет 3,00  10-5 Тл. Если оси колес находятся на расстоянии 1,50 м друг от друга, какова наведенная разность потенциалов через ось? 11.Катушка из 10 витков и площадью 35,0 см2 находится в перпендикулярном магнитном поле 0,0500 Тл. Затем катушка полностью выводится из поля за 0,100 с. Найдите наведенную ЭДС в катушке, когда она выдергивается из поля. 12. Желательно определить величину магнитного поля между полюсами большого магнита. Прямоугольная катушка из 10 витков размером 5,00 см на 8,00 см расположена перпендикулярно магнитному полю. Затем катушка выводится из поля за 0,0500 с и наведенная ЭДС равна 0.В катушке наблюдается 0250 В. Какова величина магнитного поля между полюсами? 13. Желательно определить магнитное поле стержневого магнита. Стержневой магнит проталкивается через круглую катушку диаметром 10,0 см за 2,50  10–3 с, и получается ЭДС 0,750 В. Найдите магнитное поле стержневого магнита. 14. Поток через 20-витковую катушку с сопротивлением 20,0 Ом изменяется на 5,00 2 Вт / м за время 0,0200 с. Найдите наведенный ток в катушке. 15. Покажите, что всякий раз, когда поток через катушку с сопротивлением R изменяется на ∆ΦM, в контуре всегда будет индуцированный заряд, указанный в 27-34

    Глава 27 Электромагнитная индукция ∆q = −N∆ΦM R Этот индуцированный заряд становится индуцированным током в катушке.27.3 Закон Ленца 16. Каково направление индуцированного тока в соленоиде, если северный магнитный полюс перемещается к соленоиду на диаграмме?

    Диаграмма для проблемы 16.

    Диаграмма для задачи 17.

    17. Каково направление индуцированного тока в соленоиде, если южный магнитный полюс перемещается к соленоиду на диаграмме? 18. Найдите направление тока во втором соленоиде, когда переключатель S1 в цепи соленоида 1 замкнут (а) и разомкнут (б).

    Схема для задачи 18. 27.4 Индуцированная ЭДС во вращающейся петле провода в магнитном поле — переменная ЭДС и генератор переменного тока 19. Круглая катушка в генераторе имеет 50 витков, диаметр 10,0 см, и вращается в поле 0,500 Т. Какой должна быть угловая скорость катушки, если она должна генерировать максимальное напряжение 50,0 В? 20. Круглая катушка с сопротивлением 50,0 Ом, состоящая из 20 витков и диаметром 5,00 см, вращается с угловой скоростью 377 рад / с во внешнем магнитном поле, равном 2.00 T. Найдите (а) максимальную наведенную ЭДС, (б) частоту переменной ЭДС, (в) максимальный ток в катушке и (г) мгновенный ток при 5,00 с. 21. Генератор переменного тока состоит из катушки на 200 витков диаметром 10,0 см. Если катушка вращается со скоростью 500 об / мин в магнитном поле 0,250 Тл, найдите максимальную наведенную ЭДС. 22. Вас просят разработать генератор переменного тока, который будет обеспечивать выходное напряжение максимум 156 В при частоте 60,0 Гц. (a) Найдите произведение N, количества витков в катушке, B, магнитного поля и A, площади катушки, которое даст это значение напряжения.(b) Выберите разумный набор значений для N, B и A, чтобы удовлетворить требованию. 27-35

    Глава 27 Электромагнитная индукция 23. С какой угловой скоростью следует вращать катушку площадью 1,00 м2 в магнитном поле Земли в области, где B составляет 3,50  10−5 Тл, чтобы создать максимальную ЭДС 1,50 В? 24. Круглая катушка с проволокой площадью 25,0 см2 помещается в магнитное поле 0,0500 Тл. Какой будет наведенная ЭДС в катушке, если катушка вращается со скоростью 20,0 рад / с (а) вокруг оси, которая выровнена. с магнитным полем и (б) вокруг оси, перпендикулярной магнитному полю? 25.Генератор переменного тока спроектирован с круглым поперечным сечением радиусом 1,25 см для выработки 120 В. Каким должен быть радиус катушки генератора, если он должен вырабатывать 240 В, если предположить, что магнитное поле, частота и число оборотов остается постоянным? 26. Генератор переменного тока вырабатывает 10,0 В, когда катушка вращается со скоростью 500 об / мин. Найдите ЭДС, когда катушка вращается со скоростью 1500 об / мин. 27.5 Взаимная индукция 27. ЭДС 0,800 В наблюдается в цепи, когда в соседней цепи изменяется ток со скоростью 200 А / с.Какова взаимная индуктивность цепей? 28. Найдите взаимную индуктивность двух коаксиальных соленоидов радиусом 10,0 см и длиной 20,0 см с 120 витками в катушке 1 и 200 витками в катушке 2. 29. Две катушки имеют взаимную индуктивность 5,00 мГн. Если ток в первой катушке изменится на 3,00 А за 0,0200 с, какова наведенная ЭДС во второй катушке? 27.6 Самоиндукция 30. Какова самоиндуцированная ЭДС в катушке 5,00 Гн, если ток через нее изменяется со скоростью 150 А / с? 31. Катушка имеет индуктивность 5.00 мГн. С какой скоростью должен изменяться ток в катушке, чтобы наведенная ЭДС составляла 100 В? 32. Ток через катушку индуктивности 10,0 мГн изменяется со скоростью 250 А / с. Найдите изменение магнитного потока в катушке. 27.7 Энергия, запасенная в магнитном поле индуктора 33. Сколько энергии хранится в магнитном поле катушки с индуктивностью 5,00 мГн, несущей ток 5,00 А? 34. Соленоид имеет 2500 витков / м, диаметр 10,0 см и длину 20,0 см. Если ток изменяется от 0 до 10,0 А, сколько энергии хранится в магнитном поле соленоида? 35.Если в катушке индуктивности накапливается 80,0 Дж энергии при изменении тока с 5,00 А до 15,0 А, найдите индуктивность катушки индуктивности.

    Дополнительные проблемы * 36. Стержневой магнит диаметром 10,0 см имеет значение B = 2,50  10-3 Тл. Он находится на 30,0 см выше круглой катушки с радиусом 5,00 см и 20 витками. Магнит падает из упора. 27-36

    Глава 27 Электромагнитная индукция. Найдите (а) скорость северного полюса стержневого магнита, когда он достигает катушки, (б) время, необходимое переднему краю стержневого магнита, чтобы пройти через 1.00 мм толщиной катушки, (c) наведенная ЭДС в катушке, когда передний край стержневого магнита входит в катушку, (d) ЭДС в катушке, когда весь стержневой магнит проходит через катушку, и (e ) ЭДС в катушке, когда задний край стержневого магнита проходит через катушку. Сформулируйте предположения, которые вы делаете при решении проблемы.

    Диаграмма для задачи 36.

    Диаграмма для задачи 37.

    37. На диаграмме две катушки намотаны в противоположных направлениях вокруг общего сердечника.Резистор r переменный. Найдите направление тока в резисторе R, когда (а) r увеличивается и (б) r уменьшается. * 38. Стержень длиной 25,0 см вращается с угловой скоростью 20,0 рад / с в однородном магнитном поле 4,5  10-3 Тл, направленном внутрь страницы. Найдите наведенную ЭДС от конца до конца стержня.

    Схема для задачи 38. 39. Круглая петля радиусом 5,00 см помещена в однородное внешнее магнитное поле 0,0400 Тл, направленное внутрь страницы, как показано на схеме.Круговой контур соединен проводами с резистором R = 10,0 Ом. Теперь петле позволено схлопнуться до тех пор, пока ее площадь не станет фактически нулевой за 0,010 с. Найдите величину и направление индуцированного тока в резисторе R. 40. Найдите наведенную ЭДС во внутреннем контуре, когда ток изменяется во внешнем контуре со скоростью ∆i / ∆t.

    27-37

    Глава 27 Электромагнитная индукция

    Диаграмма для задачи 39.

    Диаграмма для задачи 40.

    41. Квадратная петля из проволоки, 2.00 мм с каждой стороны, помещается в центре плоской катушки с 10 витками и радиусом 10,0 см. Ток в катушке увеличивается с 0 до 1,00 А за 2,50 с. Если сопротивление контура 0,15 Ом, найдите ЭДС, индуцированную в контуре прямоугольного сечения. * 42. Вращающаяся катушка с проволокой в ​​магнитном поле используется для создания искрового устройства. Катушка соединена щетками с двумя проводами C и D, концы которых разнесены на 1,00 мм. (a) Найдите необходимую ЭДС между точками C и D, которая позволит диэлектрический пробой воздушного зазора между этими точками.(b) Если N = 200 витков, длина катушки 10,0 см, ее ширина 8,00 см и магнитное поле 8,50  10−1 Тл, найдите угловую скорость ω катушки, чтобы получить необходимую наведенную ЭДС. для искрения.

    Схема для задачи 42. 43. Круглая петля из проволоки диаметром 7,50 см изначально ориентирована так, что внешнее магнитное поле 4,00 Тл перпендикулярно плоскости петли. За 2,00 с петля поворачивается вокруг оси в своей собственной плоскости так, что в конечном итоге она составляет угол 450 с полем.За те же 2,00 с величина внешнего поля возрастает до 5,66 Тл. Определить ЭДС, наведенную в петле.

    27-38

    Глава 27 Электромагнитная индукция 44. Ток через катушку изменяется от 300 мА до 150 мА за 5,00  10–3 с. Получена наведенная ЭДС 2,00  10−2 В. Найдите (а) индуктивность катушки, (б) начальную энергию в поле и (в) конечную энергию в поле. 45. Воздушный соленоид имеет индуктивность 5,00 мГн. Найдите его индуктивность, если в соленоид поместить кусок железа (µ = 800 µ0).46. ​​Катушка индуктивности 5,00 мГн и резистор 50,0 Ом последовательно подключены к батарее на 24,0 В. Найдите (а) конечный ток в цепи и (б) энергию, запасенную в магнитном поле индуктора. * 47. Катушка диаметром 5,00 см размещена внутри соленоида на 1000 витков / м. (а) Если ток в соленоиде возрастает до 10,0 А за 2,00  10–2 с, найдите наведенную ЭДС во внутренней катушке. (b) Каков магнитный поток через внутреннюю катушку при постоянном токе в соленоиде 10,0 А? (c) Если внутренняя катушка теперь повернута на 20.0 рад / с, какова наведенная ЭДС в катушке? (d) Если внутренняя катушка подключена к цепи с сопротивлением 30,0 Ом, найдите максимальный ток, который будет протекать в цепи. * 48. Конденсатор емкостью 5,00 мкФ заряжается до потенциала 100 В и затем подключается к катушке с индуктивностью 7,00 мГн в точках A и B на схеме. (а) Найдите исходную энергию в конденсаторе. (б) Каков максимальный ток в катушке при разряде конденсатора? (c) Когда конденсатор разряжается, куда уходит энергия в конденсаторе? (d) Что произойдет после полной разрядки конденсатора?

    Схема задачи 47.

    Схема для задачи 48.

    * 49. При выводе уравнения 27.18 наведенная ЭДС в проводе cd была выражена как Ecd = vBl sinθ. Используя рисунок 27.5, найдите угол между v и B для провода cd. Покажите, что синус этого угла сводится к синусу угла θ. * 50. На рисунке 27.1 (a) показано, что когда провод MN перемещается вправо, в проводе от M к N. индуцируется ток. Покажите, что этот провод является проводящим ток во внешнем магнитном поле и, как таковой, испытывает сила.Покажите, что эта сила противодействует исходному движению и имеет тенденцию замедлять движение проволоки и является проявлением закона Ленца. * 51. В качестве разновидности ЭДС движения, рассмотренной на рисунке 27.2, пусть рельсы будут размещены на наклонной плоскости, как показано. Найдите, как наведенная ЭДС меняется со временем.

    27-39

    Глава 27 Электромагнитная индукция

    Диаграмма для задачи 51. Интерактивные учебные пособия 52. Закон Фарадея. Катушка площадью A = 0,035 м2 подключена к гальванометру.Магнитное поле изменяется от начального значения Bi = 0,200 Тл до конечного значения Bf = 0,500 Тл за время 1,50  10−3 с. Начальное и конечное значения вектора магнитного поля указывают из области в направлении вектора площади A. Если сопротивление цепи составляет 20,5 Ом, найдите (а) наведенную ЭДС в цепи и (б) ток в цепи, в то время как магнитное поле увеличивается со временем. 53. Генератор переменного тока. Катушка с N = 200 витками проволоки с площадью поперечного сечения A = 0,015 м2 вращается с угловой скоростью ω = 377 рад / с во внешнем магнитном поле B = 0.225 Т. Найдите (а) максимальную наведенную ЭДС Emax, (б) частоту f переменной ЭДС и (в), если сопротивление цепи R = 100 Ом, найдите максимальный ток imax. (d) Напишите уравнение для наведенной переменной ЭДС E и наведенного переменного тока i и постройте график обоих.

    Чтобы перейти к этим интерактивным учебным материалам, щелкните это предложение. Чтобы перейти к другой главе, вернитесь к оглавлению, щелкнув это предложение.

    27-40

    Что такое противо-ЭДС в двигателе постоянного тока?

    Когда проводник с током помещен в магнитное поле, крутящий момент индуцирует на проводнике крутящий момент, вращающий проводник, который отсекает поток магнитного поля.Согласно феномену электромагнитной индукции «когда проводник разрезает магнитное поле, в проводнике индуцируется ЭДС» .

    Правило правой руки Флеминга определяет направление наведенной ЭДС.

    Согласно правилу Флеминга для правой руки, если мы держим большой, средний и указательный пальцы правой руки под углом 90 °, то указательный палец представляет направление магнитного поля. Большой палец показывает направление движения проводника, а средний палец представляет собой индукцию ЭДС на проводнике.

    При применении правила правой руки на рисунке, показанном ниже, видно, что направление наведенной ЭДС противоположно приложенному напряжению. Таким образом, ЭДС называется противодействующей ЭДС или обратной ЭДС .

    Обратная ЭДС развивается последовательно с приложенным напряжением, но противоположно по направлению, т. Е. Обратная ЭДС противодействует току, который ее вызывает.

    Величина обратной ЭДС определяется таким же выражением, как показано ниже:

    Где E b — наведенная ЭДС двигателя, известная как обратная ЭДС, A — количество параллельных путей через якорь между щетками противоположной полярности.P — количество полюсов, N — скорость, Z — общее количество проводников в якоре, а ϕ — полезный поток на полюс.

    Простая условная принципиальная схема машины, работающей как двигатель, показана на схеме ниже:

    В этом случае величина обратной ЭДС всегда меньше приложенного напряжения. Разница между ними почти одинакова, когда двигатель работает в нормальных условиях.

    Ток, наведенный на двигатель из-за основного источника питания.Соотношение между основным питанием, обратной ЭДС и током якоря определяется как E b = V — I a R a .

    Преимущества обратной ЭДС в двигателе постоянного тока

    1. Обратная ЭДС противодействует питающему напряжению. Напряжение питания индуцирует ток в катушке, которая вращает якорь. Электрическая работа, необходимая двигателю для создания тока против обратной ЭДС, преобразуется в механическую энергию. И эта энергия индуцируется в якоре двигателя.Таким образом, можно сказать, что преобразование энергии в двигателе постоянного тока возможно только за счет обратной ЭДС.

    Механическая энергия, индуцированная в двигателе, является произведением обратной ЭДС и тока якоря, то есть E b I a .

    2. Обратная ЭДС делает двигатель постоянного тока саморегулирующимся, т.е. обратная ЭДС развивает ток якоря в соответствии с потребностями двигателя. Ток якоря двигателя рассчитывается как:

    Давайте разберемся, как обратная ЭДС заставляет двигатель саморегулироваться.

    • Считайте, что двигатель работает без нагрузки. На холостом ходу двигателю постоянного тока требуется небольшой крутящий момент для контроля потерь на трение и сопротивление воздуха. Двигатель потребляет меньше тока. Поскольку обратная ЭДС зависит от тока, их величина также уменьшается. Величина обратной ЭДС практически равна напряжению питания.
    • Если к двигателю прилагается внезапная нагрузка, двигатель замедляется. По мере уменьшения скорости двигателя величина их обратной ЭДС также падает.Малая задняя ЭДС отбирает сильный ток от источника питания. Большой ток якоря вызывает в якоре большой крутящий момент, необходимый для двигателя. Таким образом, двигатель непрерывно движется с новой скоростью.
    • Если нагрузка на двигатель внезапно снижается, крутящий момент двигателя больше, чем момент нагрузки. Приводной крутящий момент увеличивает скорость двигателя, что также увеличивает их обратную ЭДС. Высокое значение обратной ЭДС снижает ток якоря. Малая величина тока якоря приводит к меньшему крутящему моменту, равному крутящему моменту нагрузки.И мотор будет равномерно вращаться с новой скоростью.

    соотношение между механической мощностью (Pm), напряжением питания (Vt) и обратной ЭДС (Eb)

    Обратная ЭДС в двигателе постоянного тока выражается как:

    Где E b — Обратная ЭДС
    I a — Ток якоря
    В t — Напряжение на клеммах
    R a — Сопротивление якоря

    Максимальная мощность, развиваемая двигателем, выражается в

    .

    Продифференцируя приведенное выше уравнение, мы получаем

    Из уравнения обратной ЭДС получаем

    Подставив I на R на в приведенное выше уравнение, мы получим

    Приведенное выше уравнение показывает, что максимальная мощность в двигателе вырабатывается, когда противоэдс равна половине напряжения питания.

    Лаборатория 7 — Цепи LR

    Введение

    Английский физик Майкл Фарадей в 1831 году обнаружил, что при изменении тока через катушку катушка создает изменяющееся магнитное поле (в дополнение к полю изменяющегося тока), которое индуцирует электродвижущую силу («ЭДС») в самой катушке. . В 1834 году немецкий физик Генрих Ленц уточнил это, показав, что индуцированный ток, вызываемый этой ЭДС, будет в направлении, противоположном изменению исходного тока.Мы называем это явление самоиндукцией, а катушки — индукторами. Когда Фарадей объявил о своем открытии, его спросили, как можно использовать это знание? Его ответ был: «Какая польза от новорожденного ребенка?» Как и многие, казалось бы, загадочные открытия, исследования индукции Фарадеем привели к созданию нескольких распространенных и полезных современных электрических устройств. Катушки индуктивности, как и конденсаторы, влияют на временные характеристики цепи переменного тока (переменного тока) и поэтому используются для настройки радиосхем, фильтрации нежелательных шумов и т. Д.В телефонной трубке используется индуктор, а также в стереосистемах и микрофонах. В этой лабораторной работе вы изучите влияние индуктора на ток и напряжение в простой цепи.

    Обсуждение принципов

    Индуктивность цепи, обычно обозначаемая цифрой L и измеряемая в генри (Гн), представляет собой тенденцию цепи противодействовать любым изменениям тока. Это противодействие изменению тока проявляется как замедление роста или падения тока в цепях.Индуктивность — это свойство электрических устройств. Устройства, обладающие этим свойством, называются индукторами , . Индуктивность устройства, как сопротивление и емкость, зависит от геометрических факторов, таких как размер устройства и от материала, из которого оно изготовлено. Это не зависит от силы тока в устройстве. Рассмотрим простую схему, состоящую из переключателя, резистора R и батареи. Когда переключатель замкнут, ток I в цепи будет очень быстро увеличиваться до устойчивого значения, заданного законом Ома,

    I = ΔV / R,

    , где

    ΔV

    — это напряжение или ЭДС батареи.Рассмотрим ту же схему с добавлением катушки индуктивности, как показано на рис.1.

    Рисунок 1 : Контур LR

    Когда переключатель находится в положении 1, как показано на рис. 1 (а), нарастающий ток создает нарастающий магнитный поток в катушке индуктивности. Этот индуцированный магнитный поток создает электродвижущую силу (ЭДС), полярность которой противоположна полярности батареи, что приводит к индуцированному току, противоположному току батареи.Следовательно, ток нарастает медленнее, чем в противном случае без индуктора. Насколько медленнее увеличивается ток, зависит от величины индуктивности и величины сопротивления. Несмотря на то, что время, необходимое току для повышения до его конечного значения, больше, фактическое конечное значение такое же, как то, которое было бы достигнуто без катушки индуктивности. Рассмотрим ситуацию, когда переключатель какое-то время находился в положении 1 и в цепи течет постоянный ток.Если переключатель теперь перемещен в положение 2, как на рис. 1 (b), батарея больше не является частью цепи. Катушка индуктивности предотвратит мгновенное падение тока до нуля. Подобно тому, как индуктор сопротивлялся быстрому увеличению тока, он также сопротивлялся быстрому уменьшению тока. Для схемы, показанной на рис. 1 (а), уравнение петли Кирхгофа можно записать как Решение уравнения. (1)

    — это

    (2)

    I = I f
    1 — e (−R / L) t
    где I f представляет конечный установившийся ток, протекающий в цепи после бесконечного промежутка времени, R — сопротивление цепи, а L — индуктивность катушки.Из этого выражения видно, что ток увеличивается экспоненциально. Это показано на рис. 2 (а). Когда переключатель перемещается в положение 2, для схемы, показанной на рис. 1 (b), уравнение петли Кирхгофа теперь имеет вид Решение уравнения. (3)

    — это где I 0 представляет собой начальный максимальный ток в цепи при t = 0. Из этого выражения видно, что ток уменьшается экспоненциально. См. Рис. 2 (b).

    Рисунок 2 : График зависимости тока от времени

    Постоянная времени

    Математический анализ простой цепи LR аналогичен анализу простой RC-цепи (цепи, состоящей из последовательно соединенных резистора и конденсатора).В цепи RL постоянная времени

    τ

    определяется как Постоянная времени также определяется как количество времени, за которое ток достигает 63% или

    (1 — e -1 )

    от его окончательного значения. Обратите внимание, что уравнение. (2) I = I f
    1 — e (−R / L) t
    имеет ту же форму, что и уравнение, описывающее зарядку конденсатора. Поскольку ток меняется со временем, разность потенциалов на резисторе также должна изменяться со временем.Уравнение для разности потенциалов на резисторе получается с использованием закона Ома,

    ΔV = IR,

    (6)

    ΔV R = ΔV f
    1 — e (−t / τ )
    где

    ΔV f

    — конечная или максимальная разность потенциалов на резисторе, равная ЭДС аккумулятора. Обратите внимание, что при

    t = 0, (1-e -t / τ ) = 0

    , а ток в цепи и разность потенциалов на резисторе равны нулю.Следовательно, падение потенциала полностью происходит на катушке индуктивности. При t = бесконечность (долгое время после того, как переключатель находился в положении 1),

    (1 — e −t / τ ) = 1

    и ток в цепи и разность потенциалов на резисторе максимальны при I f и

    ΔV R .

    Следовательно, падение потенциала полностью происходит на резисторе, а падение потенциала на катушке индуктивности равно нулю. Без катушки индуктивности в цепи ток в резисторе очень быстро упадет до нуля, как только переключатель будет перемещен в положение 2.Когда катушка индуктивности включена в цепь, она препятствует этому изменению тока, поэтому ток падает медленнее. Ток и напряжение на резисторе t секунд после того, как батарея была удалена из цепи путем перевода переключателя в положение 2, определяются следующим образом.

    (8)

    ΔV R = ΔV 0 e (−t / τ )

    Здесь

    τ

    — время, необходимое для уменьшения тока до 33% от исходного значения при t = 0.Рассмотрим уравнение. (7)

    I = I 0 e (−t / τ )

    и уравнение. (8)

    ΔV R = ΔV 0 e (−t / τ )

    . При

    t = 0, e −t / τ = 1,

    и ток в цепи и разность потенциалов на резисторе максимальны. Следовательно, падение потенциала полностью происходит на резисторе, а падение потенциала на катушке индуктивности равно нулю. При t = бесконечность (долгое время после того, как переключатель находится в положении 2),

    e −t / τ = 0,

    и ток в цепи и разность потенциалов на резисторе равны нулю.Следовательно, падение потенциала полностью происходит на катушке индуктивности, а падение потенциала на резисторе равно нулю. В любой момент времени t сумма падений потенциала на резисторе и катушке индуктивности будет равна ЭДС батареи.

    (9)

    ЭДС аккумулятор = ΔV R + ΔV L

    Разность потенциалов

    ΔV R

    на резисторе как функция времени показана здесь на Рис. 3 и Рис.4 показывает падение напряжения на катушке индуктивности как функцию времени. Обратите внимание, что когда напряжение на резисторе является максимальным, напряжение на катушке индуктивности равно нулю и наоборот, как обсуждалось ранее.

    Рисунок 3 : Напряжение на резисторе как функция времени

    Рисунок 4 : Напряжение на катушке индуктивности как функция времени

    Уравнение (6) ΔV R = ΔV f
    1 — e (−t / τ )
    можно алгебраически переставить как: где

    τ

    заменено на L / R .Вычисляя натуральный логарифм обеих частей этого уравнения и умножая на –1, получаем

    (11)

    −ln
    = t.

    Если вы рассматриваете левую часть уравнения как одну переменную, скажем, y , тогда уравнение принимает вид

    y = (R / L) t,

    , что является линейным уравнением формы

    y = mx.

    Индуктивность можно определить по наклону этой прямой. Точно так же уравнение.(8)

    ΔV R = ΔV 0 e (−t / τ )

    можно записать как

    (12)

    −ln
    = t.

    График зависимости −ln
    от времени t для убывающего тока (вскоре после размыкания переключателя) даст прямую линию с наклоном R / L , из которой индуктивность может быть определена.

    Использование прямоугольной волны для имитации роли коммутатора

    В этом эксперименте вместо переключателя мы будем использовать генератор сигналов, который может генерировать периодические волновые формы различной формы, такие как синусоидальная волна, треугольная волна и прямоугольная волна. Также можно регулировать как частоту, так и амплитуду формы волны. Здесь мы будем использовать генератор сигналов для создания изменяющегося во времени напряжения прямоугольной формы на катушке индуктивности, аналогичного показанному на рисунке 5.

    Рисунок 5 : Прямоугольная волна с периодом T

    Выходное напряжение генератора сигналов изменяется назад и вперед от постоянного положительного значения до постоянного нуля вольт через равные интервалы времени t .Время

    T = 2t

    — это период прямоугольной волны. В течение первой половины цикла, когда напряжение положительное, это похоже на то, что переключатель находится в положении 1. Во второй половине цикла, когда напряжение равно нулю, это то же самое, что переключатель находится в положении 2. Таким образом, прямоугольная волна, представляющая собой постоянное напряжение, которое периодически включается и выключается, служит одновременно аккумулятором и переключателем в схеме, показанной на рис. Генератор сигналов позволяет выполнять это переключение многократно, и можно оптимизировать сбор данных, регулируя частоту повторения.Эта частота будет зависеть от постоянной времени цепи RL. Когда время t больше постоянной времени

    τ

    цепи RL, ток в цепи будет иметь достаточно времени, чтобы достичь установившегося состояния, и напряжение на катушке индуктивности будет таким, как показано на рис. .

    Цель

    Цель этого эксперимента — изучить динамическое поведение цепи LR с помощью осциллографа, чтобы визуализировать напряжение на резисторе как при нарастании, так и при уменьшении тока.Вы также определите постоянную времени и индуктивность катушки.

    Оборудование

    • Печатная плата PASCO
    • Программное обеспечение Capstone
    • Сигнальный интерфейс с выходной мощностью
    • Соединительные провода
    • Мультиметр

    Процедура

    Распечатайте лист для этой лабораторной работы.Этот лист понадобится вам для записи ваших данных.

    Настройка контура LR

    Печатная плата RLC, которую вы будете использовать, состоит из трех резисторов и одной катушки индуктивности среди других элементов. Значение индуктивности можно изменить, вставив в катушку железный сердечник. См. Рис. 6 ниже. Теоретически вы можете использовать разные комбинации резисторов и конденсаторов. В этом эксперименте вы будете использовать резистор 10 Ом и катушку индуктивности.

    Рисунок 6 : Печатная плата RLC

    1

    Подключите крайнюю правую выходную клемму сигнального интерфейса к катушке индуктивности в точке 9.

    2

    Подключите точку 1 ко второй выходной клемме сигнального интерфейса, чтобы замкнуть цепь.

    3

    Подключите пробник напряжения к аналоговому каналу A.

    4

    Чтобы измерить напряжение на резисторе, подключите один вывод датчика напряжения к точке 8, а другой — к точке 1. Убедитесь, что земля интерфейса (вывод «-») подключена к той же стороне резистора, что и земля генератора сигналов (выход мощности). Подключение вашей схемы должно выглядеть так, как показано на рис.7.

    Рисунок 7 : Принципиальная схема

    Контрольная точка 1:
    Попросите своего технического специалиста проверить ваши связи, прежде чем продолжить.

    Процедура A: Определение

    L по постоянной времени Компьютер будет работать как осциллограф для записи

    ΔV R

    и как генератор сигналов.

    5

    Откройте файл Capstone, связанный с этой лабораторной работой.Отобразится экран, аналогичный показанному на рис. 8.

    Рисунок 8 : Начальный экран файла схемы LR

    6

    Файл должен открываться с помощью генератора сигналов, чтобы получить положительную прямоугольную волну.

    7

    Если еще не установлено, установите напряжение на амплитуду 7 В с частотой от 120 до 180 Гц и установите смещение напряжения на 7 В.

    8

    Включите генератор сигналов, щелкнув ON в окне генератора сигналов, и проследите за сигналом, щелкнув МОНИТОР в главном окне.Должна быть трасса сигнала, подобная показанной на рис. 9. Это позволит вам наблюдать, как напряжение на резисторе изменяется в зависимости от времени. Нажмите СТОП через несколько секунд. Данные будут оставаться в окне осциллографа до следующего нажатия кнопки START .

    Рисунок 9 : Пример сигнала

    9

    Отрегулируйте напряжение (разность потенциалов) и шкалы времени так, чтобы в окне осциллографа отображалась примерно одна длина волны, поместив курсор на значения каждой шкалы и перетащив влево-вправо или вверх-вниз, в зависимости от ситуации.

    10

    Нажмите кнопку Показать координаты из кнопок над графиком. См. Рис.10.

    11

    Используя Показать координаты , определите время начала (то есть, когда разность потенциалов начинает увеличиваться с 0 вольт) и запишите его в рабочий лист.

    12

    Вычислите 63% максимальной разности потенциалов (0,63

    ΔV f ).

    13

    Используйте Показать координаты , чтобы определить время, в которое возникает эта разность потенциалов.Запишите это время на листе.

    Рисунок 10 : Показать координаты

    14

    Из двух значений времени, полученных на шагах 11 и 13, определите и запишите время, необходимое для перехода сигнала от

    ΔV R = 0 до ΔV R = 0,63 ΔV f .

    Это ваше экспериментальное значение постоянной времени

    τ .

    15

    Используйте мультиметр для измерения общего сопротивления катушки и резистора последовательно.Это полное сопротивление цепи. См. Приложение К. Для этого отсоедините все остальные провода от печатной платы PASCO, а затем подключите мультиметр к комбинации резистора и индуктора, как показано на рис.11.

    Рисунок 11 : Физическая проводка для измерения общего сопротивления цепи RL

    16

    Рассчитайте экспериментальное значение индуктивности по формуле. (5)

    τ = L / R.

    и экспериментальные значения

    τ

    и R .Запишите это значение на листе.

    17

    Используйте значение индуктивности, напечатанное рядом с катушкой индуктивности на печатной плате PASCO, как допустимое значение и запишите его в рабочий лист.

    18

    Вычислите погрешность в процентах между экспериментальным и принятым значениями индуктивности и запишите ее в рабочий лист. См. Приложение Б.

    Контрольная точка 2:
    Попросите своего технического специалиста проверить ваши данные и расчеты.

    Процедура B: Измерение напряжения для увеличения тока

    19

    По записанной осциллограмме измерьте напряжение

    ΔV R

    на резисторе и время t для шести точек на восходящей части кривой.Запишите эти значения в Таблицу данных 1.

    20

    Исходя из окончательной разности потенциалов и значений

    ΔV R

    , которые вы только что записали, вычислите количества для оставшихся двух столбцов в таблице данных 1.

    21

    Используйте Excel для построения графика

    −ln [(ΔV f — ΔV R ) / ΔV f ]

    против t для шести точек. См. Приложение G.

    22

    Используя опцию линии тренда в Excel, чтобы нарисовать линию, наиболее подходящую для ваших данных, определите наклон линии.См. Приложение H. Запишите это на листе.

    23

    Используйте значение крутизны, чтобы найти индуктивность и записать ее в рабочий лист.

    24

    Вычислите погрешность в процентах между принятым значением индуктивности и значением, полученным из наклона графика. Запишите это значение на листе.

    Контрольная точка 3:
    Попросите своего технического специалиста проверить ваши данные, график Excel и расчеты.

    Процедура C: Измерение напряжения для уменьшения тока

    25

    По записанной осциллограмме измерьте напряжение

    ΔV R

    на резисторе и время t для шести точек на спадающей части кривой.Запишите эти значения в Таблицу данных 2. Обратите внимание, что

    ΔV 0

    для падающей части кривой совпадает с

    ΔV f

    для восходящей части кривой.

    26

    Исходя из начальной разности потенциалов

    ΔV 0

    и значений

    ΔV R

    , которые вы только что записали, вычислите количества для оставшихся двух столбцов в таблице данных 2.

    27

    Используйте Excel, чтобы построить график

    −ln [(ΔV R ) / ΔV 0 ]

    против t для ваших шести точек.

    28

    Используя параметр линии тренда в Excel, чтобы нарисовать линию, наиболее подходящую для ваших данных, определите наклон линии и запишите это значение на листе.

    29

    По значению наклона найдите индуктивность и запишите ее в рабочий лист.

    30

    Вычислите погрешность в процентах между принятым значением индуктивности и значением, полученным из наклона

    −ln [(ΔV R ) / ΔV 0 ]

    по сравнению с графиком t .

    Добавить комментарий

    Ваш адрес email не будет опубликован. Обязательные поля помечены *